Sie sind auf Seite 1von 89

GRAND PHYSIOLOGY B FINALS REVIEWER

Page 1 of 89 By: Ayesha Bea Federizo Batch 2017


GRAND PHYSIOLOGY B FINALS REVIEWER

CARDIOVASCULAR PHYSIOLOGY 1 Remember this: MA SISI DSSD


MITRAL VALVE STENOSIS DIASTOLE
*Blood flow from the Heart* INCOMPETENCE SYSTOLE
AORTIC VALVE STENOSIS SYSTOLE
INCOMPETENCE DIASTOLE
Question: When is Mitral Stenosis murmur best heard?
Answer: Diastole
th
Question: You heard a murmur in the 5 ICS Mid Clavicular
Line during Systole, what kind of lesion is this? What valve is
affected? Incompetence @ Mitral valve

*Myocardial cells*
o Automatic
A. SA Node/ Node of Keith and Flack
Atria: Primer pumps
-found in junction of SVC and RA
Ventricle: Major pumps
-Primary pacemaker
Left ventricle is thicker
- highest frequency of AP generation
*Cardiac Valves*
B. AV Node/ Node of Kent and Tawara
 Atrioventricular valve: -found in posterior R side of interatrial septum;
o Mitral- 2 cusps; (+) chordae tendinae for better Ventricular filling
o Tricuspid- 3 cusps; (+) chordae tendinae -small fiber diameter and few gap junctions
- most common site for heart block= Nodal zone
 Semilunar Valves: of AV node
o Pulmonary- 3 cusps C. Purkinje system
o Aortic- 3 cusps -Fastest AP conduction
-large fiber diameter kase
#Chordae tendinae + Papillary muscle = prevent
overbulging of AV valves into atria

#Valve closure is due to pressure difference *SA NODE*


*Heart Sounds*

 Phase 4/RMP
Less K channels are open
More Na leak channels open
Kaya RMP is less negative
 Appointment from 2pm – 5pm  Phase 0 (depo)
APTM Voltage gated Ca channels open
Aortic 2nd ics Right sternal border slowly (L type > T type)
nd
Pulmonic 2 ics left sternal border Ca and some Na Influx
Tricuspid 5th ics left sternal border  Phase 1 2 3
th
Mitral= 5 ics MCL Ca influx K efflux= slow repo
st
 AV valve closure= 1 heart sound; simultaneous Midway- Ca channel closes
K channels open continuously

nd
SL valve closure: 2 heart sound; simultaneous or not #Action potential= Ca Influx
; due to physiologic splitting s/t Inspiration - ,mauuna
mag close si Aortic valve
rd
 3 heart sound= normal in children; Ventricular
gallop; due to rapid inflow of blood from Atria to
ventricle upon opening of the AV valve

 4th heart sound=atrial gallop; due to atrial contraction


or systole

*Cardiac Valve Lesion*


 Stenosis- narrowing = dec blow flow
 Incompetence/Insufficiency= regurgitation or
backflow of blood
 Murmurs- heart sound due to lesions

Page 2 of 89 By: Ayesha Bea Federizo Batch 2017


GRAND PHYSIOLOGY B FINALS REVIEWER

*VENTRICULAR MUSCLE*

*Ion channels in Cardiac muscle membrane:


Ca enter T tubule Some binds with troponin C forming Ca-
Troponin complex= contraction;
Some binds with ryanodine receptors- activate Ca gated
channels, Ca moves out , some binds with troponin C forming
 Phase 4(RMP) Ca-Troponin complex= contraction
High permeability to K
Slight permeability to Na O diba dami nyang source for calcium!
 Phase 0 (depo)
Na influx Calcium pump para mag relax un muscle babalik na sa
Ca influx pinanggalin nya si calcium
Na channels close
 Phase 1 #Cardiac Glycosides- Digitalis for HF
K channels open Inhibit Na-K pump para walang calcium pump para si
 Phase 2 Calcium dun lang sa loob para stronger contraction
Plateu (Pla-2, phase 2 )
K efflux and Ca influx effect:
Membrane potential ain’t moving Positive Inotropy= Inc force of contraction
 Phase 3 Negative Chronotropy- Dec HR
Ca channels close Negative Dromotropy- Dec Conduction
K channels remain open

#Ca influx is most important during plateu or Phase 2, ito un


responsible sa ST segment sa ECG
#Action Potential is d/t Na Influx mainly, secondary lang Ca
influx
#Absolute RP= Phase 0-2
#Relative RP= End of phase 2- Phase 4(RMP)

*Heart Block*
-Common sa AV Node, particularly sa Nodal Zone

1st degree
Incomplete heart block
PR interval > 0.20 sec
2nd degree
Incomplete heart block
Not all impulses are transmitted
P> QRS = P wave P wave QRS T P wave
3rd degree
Complete heart block
After 20 sec purkinje fibers take over

#Extra systole occurs only during: mid diastole and late


diastole

*Contractility properties of the heart:

#Involuntary; smaller muscle fiber


#intercalated disc- connects muscle fiber thru gap junctions
#exhibits functional syncitium
#fair amount of connective and elastic tissue
#more mitochondria and active capillaries
#Sarcoplasmic reticulum is less developed
#well developed Transverse tubular system

Page 3 of 89 By: Ayesha Bea Federizo Batch 2017


GRAND PHYSIOLOGY B FINALS REVIEWER

CARDIOVASCULAR PHYSIOLOGY 2

Tips: Pakinggan ng paulit ulit yung Narration ng Cardiac Cycle.


Pag nagets nyo nayun, madali na lang 

 Atrial Systole
#Onset of Isovolumic Contraction= 1st heart sound= AV valve
-preceded by P wave
closure
-4th heart sound
#end of Isovolumic Contraction= Open Semilunar valve
-A wave (JVP curve) d/t inc atrial
#marks the beginning of Isovolumic Relaxation= Closure of
pressure d/t atrial systole
semilunar valves=2nd heart sound
#Onset of Ventricular filling= AV valves open
 Isovolumic contraction
-C wave
AV valve Open= Atrial Pressure> Vent Pressure
- Av valve close; SL valve still close
AV valve Close= Vent Pressure> Atrial Pressure
-After onset of QRS SL valve Open= R Vent Pressure>10 mmHg
L Vent Pressure> 80mmHg
 Rapid Ejection SL valve close= Aortic Pressure> Vent Pressure
*-Ventricular pressure > than Aortic
pressure * *Structure Innervated by Sympathetic Nerve
-atrial filling begins
1. SA node
 Reduced Ejection 2. AV node
-T wave 3. Purkinje Conduction System
-atrial filling continuous 4. Atria
-Protodiastole 5. Ventricle
6.
 Isovolumic Relaxation *Structure Innervated by Paraympathetic Nerve/Vagus
-Incisura/dicrotic notch
Av valve close; SL valve still close 1. SA node
-V wave (JVP curve) d/t inc atrial 2. AV node
pressure d/t inc atrial filling 3. Proximal bundle of his
4. Atrial muscle
5. Ventricular muscle- INDIRECT LANG!
 Rapid Inflow #Why? Kasi diba parasympa, prolonged ang AV nodal
rd
-3 heart sound- open na kasi AV delay, so more filling sa ventricle, mas stretched ang
valve Ventricular wall, so increased ang Force of Ventricular
 Diastasis contraction
-SL valve is close In short pag direct innervations ang tinatanong sa
-AV valve is open Parasympathetic NS, di kasama ang Ventricular
-75% Ventricular filling muscle!

Page 4 of 89 By: Ayesha Bea Federizo Batch 2017


GRAND PHYSIOLOGY B FINALS REVIEWER

CARDIOVASCULAR PHYSIOLOGY 3 #dapat hindi Overstretched or Overdistended ang cardiac


muscle, prevented yan ng connective tissue sa heart.
Cardiodynamics
A. Heart Rate= Number of heart beats or
contraction per minute, N=60-100 (adult) Factors affecting cardiac muscle length=

o Stronger Atrial Contraction=


B. Cardiac Output=
- Diba may 25% pa sa atria, o di pag mas
-Vol/Amt/Quantity ejected by each ventricle
per MINUTE; SVxHR malakas yan mas madami pupunta sa
ventricle, mas distended ang heart, Inc EDV
mas malakas contraction, Dec ang ESV
C. Stroke volume=
-Vol/Amt/Quantity ejected by each ventricle o Inc Total Blood Volume
per CONTRACTION/CARDIAC CYCLE/ HEART BEAT - Strength in number! In this case,
amount/volume.
D. End Diastolic Volume(EDV)/Preload= o Inc Venous Tone
-Vol of blood in Ventricle after relaxation - Sustained contraction, hindi constriction!
- Sa peripheral BV to ah, so ibig sabihin ang
before contraction; vol of blood that will exert force
veins, partially contracted sila, veins are your
on ventricular wall. Ave= 110-130ml; ESV+SV
capacitance vessel diba, so kapag inc ang
venous tone decrease ang venous capacity,
# ito yung baon mo sa school, ito un motivation mo
sa pagpasok, mas malaki ang baon mas may force so Inc ang venous return and so on
o Inc Pumping of skeletal muscle
pumasok! 
- You move= compress vein= open venous
valves= inc venous return and so on
#Intrinsic Factors= Force of ventricular contraction
o Inc negative intrathoracic Pressure
#Peripheral Factors= blood vol, venous return, arterial system
- Suction effect sa heart= heart dilate= more
blood= Inc EDV so on
EDV/Preload Factors:
F. Afterload
 Venous return- Most important
- Pressure sa Aorta
-yung babalik sa heart mo; In short mas
madaming babalik na blood, mas malakas - This is different from ESV
yung contraction ng heart, inc CO - Pag ito increased di makakaflow yung blood
mula Left ventricle papuntang AORTA, so
 Effective filling time- mas mahaba ang diastole, mas bababa ang CO
madaming filling, mas madaming blood sa Ventricles
mas malakas yung contraction ng heart, inc CO; 20-
CONCLUSION: ANG EDV/PRELOAD mo, maganda yan, pag
180 bpm ok pa ang filling pero pag >180 bpm na OA
na, din a kaya magcompensate, panget na. mataas yan, mataas din ang Cardiac output (CO) mo; ANG ESV
naman masama yan, kasi yan yung natitira sa heart, pag
Bpm= beats per minute/ (HR)
mataas yan ibig sabihin hindi maganda yung pag pump ng
heart mo, so mag dedecrease ang CO mo.
 Effective Filling Pressure
-Difference of Central Venous Pressure (CVP)
Autonomics
& Intrathoracic Pressure (IP)
Sympathetic: Excitatory
CVP= Pressure sa RIGHT ATRIUM
Parasympathetic: Inhibitory, pero may indirect “excitatory”
IP= pressure outside heart.
If CVP>IP= Inc EDV effect sya sa Ventricle diba? Naalala mo pa un? Hindi na? sige
basahin mo ulit from the top.
 Myocardial Compliance
-ability to distend d/t elastic tissue; change Sympathetic:
in volume should be greater than change in pressure  Epi and Norepi binds with B1 receptor sa heart, ang
daming mangyayare daming sinasabe grrrr. Pero eto
E. End Systolic Volume (ESV) lang yan:
-Volume of blood remaining in ventricles
 Phosphorylated and Ca channel= more Ca enters=
after contraction
INC FORCE OF CONTRACTION
N=45-50ml; SV-EDV
#tira ng baon mo, naks di naubos! Balik mo sa nanay
mo yan!  Phosphorylate ang Trop I= Troponin tropomyosin will
cover active site of actin= RELAXATION
ESV Factors:
 Myocardial Contraction= MOST IMPORTANT  Phosphorylated or Inhibited ang phospholamban,
- Malamang, mas malakas contraction, mas yang phospholamban nayan, masama yan, kasi
konti matitira sa ventricle, so dec ang ESV iniinhibit nya ang Ca pump, so pag inhibited ang
phospholamban, magkakaron ng calcium pump, so
Frank Starling Law: Inc blood sa Ventricle, mas stretched ang babalik ang Ca sa Sarcoplasmic reticulum=
ventricle, Inc ang Contraction force RELAXATION
Factors
*End of autonomics
-EDV
Autonomics
Page 5 of 89 Frank starling’s law By: Ayesha Bea Federizo Batch 2017
GRAND PHYSIOLOGY B FINALS REVIEWER

Heart Rate  Point A


= Inc heart rate, Inc ang Ca influx, Inc force of - Opening of AV Valves
contraction
 Point A to B
Factors that will regulate HR -Ventricular filling
1. Autonomic or neural- MOST IMPORTANT -P wave in ECG
-Sympa/ Para! - Inc Vol of blood in the ventricles
-Diastasis
rd
2. Hormones -3 heart sound
- Thyroid & Cortisol (inc)
3. Neuroreflexes  Point B
a. Baroreceptor Chemoreceptor -Closure of AV Valves 1st heart sound
b. Bainbridge reflex-Inc blood return sa Atria= - ito na un amt ng EDV mo
stretched ang atria mo= stimulated ang
stretch receptors= SA node stimulation=Inc * para makuha ang SV/ Stroke volume, I minus mo un EDV sa
AP = Inc HR ESV mo.
4. Moderate to heavy exercise
5. Emotions  Point B to C
6. Temperature, Inc= Inc HR -Period of isovolumic contraction
Yang HR ok lang na mataas yan, wag lang lalagpas ng 180 bpm - QRS complex
kase OA na un.
 Point C
Yung mga nakalagay sa taas will either Increase or decrease o SL valve opens
your CO, depende sa anong state nila.
 Point C to D
Other factors that will influence CO: o Period of Ejection and Dec volume of blood
in the ventricles
A. Ejection fraction- percentage ng EDV na naeject ng
 Point D
ventricle or in short, Stroke volume; N=65-70% of
o Closure of Semilunar Valves dito na un sa
EDV ( measured via 2d echo)
ESV mo
B. Body surface are- more developed skeletal muscle
o 2nd heart sound
(athlete)= Inc CO
- physiologic splitting of 2nd heart sound
-Aortic pressure exceeds Ventricular pressure
With athletes pala, its okay to have Low HR, normal un.
 Point D to A
C. Cardiac Reserve
Isovolumic relaxation
-Max amount ng CO na kayang I increase ng Heart
atrial filling is increasing
o Hypereffective- Increased eg. during Exercise
o Hypoeffective- weak heart eg. (+) Heart
Test of knowledge:
Failure
San imeameasure ang CVP?
EJECTION LOOP
Kelan maririnig ang murmur pag Insufficient ang Aortic valve?
Anong phase ng Cardiac cycle mas mataas ang Ventricular
pressure as compared to Aortic Pressure? E yung 3rd heart
sound? Yung 4th?
Paki Auscultate nga ang Pulmonic valve? E ang Apex? Sige
ngaaa saan? 
San pinakamabilis ang conduction ng AP? Ano sagot mo? SA
node? WRONGGG!

HALFWAY THEREEEEE! SA CARDIO KA PALANG ANG HABA PA


GIRL. PUSH MO YAN!

Page 6 of 89 By: Ayesha Bea Federizo Batch 2017


GRAND PHYSIOLOGY B FINALS REVIEWER

CARDIOVASCULAR PHYSIOLOGY 4 *BLOOD FLOW*


Vascular System  Ohms Law- Change in Pressure/Resistance; so pag
A. Aorta mas mataas ang pressure gradient, mas maganda
-Largest ARTERYYY blood flow
-Highest presssureeeeee,
-Highest velocity of blood flow  Poiseuilles- Resistance= Length x Viscocity x 8 / nr4;
-Inc Transmural pressure in short mas mahaba, mas malapot mas mataas
-More Elastin than Smooth muscle fiber, resistance, pero pag inc ang radius ng BV dec ang
-pag nag branch out= Arteries resistance
- Blood contained= Stress blood volume
#kaya ang arteriole, pag nagconstrict sya, sobrang liit
B. Venous system ng lumen nya diba, tapos nagconstrict pa, di wala ng
-Collecting blood vessel, CAPACITANCE VESSEL Blood nadaan na blood.
reservoir
-thinner wall  Reynolds Number- tendency of blood to become
-Vena Cava= biggest radius turbulent
= lowest transmural pressure = Diameter x Velocity x density / viscosity
; Inc diameter density and velocity= turbulence inc;
C. Microcirculation pero pag Viscous and blood less turbulence, kasi
malapot masyado, so magiging mabagal ang blood
o Arterioles flow, like sa case ng polycythemia vera, kaya dec ang
-formed when arteries branch out Reynolds number.
- pinaka madaming smooth muscle, so sya
ang thickest ang muscular layer= #Types of blood flow
RESISTANCE VESSELS Laminar- Silent
Turbulent- Loud
o Capillaries Heart-Murmur
- No sympathetic innervations but responds to Blood vessel- Bruit
circulating vasoconstrictor <2000= laminar
- Smallest BV; largest Cross sectional area 2000-3000= transitional
- Thinnest >3000= turbulent
- No smooth muscle
- EXCHANGE VESSEL Venous return
- Amount of blood that will return to
o Venules heart ( okay. buti pa yung dugo
- BV that is in direct contact to tissues bumabalik sa puso, ikaw, anyare
sayo?)
 Left side of Heart Aorta Arteries Arterioles
Metarterioles  Venules  Veins Right side of
Heart

La Place Equation:
Wall tension= Distending pressure x radius  Inc total BV= Inc MCSFP= Inc Venous Return=Inc
of blood vessel CO
 Inc Resistance and Vascular Capacity= Dec CO
#Prone to aneurysm ang ARTERIES kasi they have
high pressure and they must have high wall tension. Once wall Central Venous Pressure
tension is low and pressure is high = RUPTURE -Pressure in R atrium
- diba ang lahat ay dahil sa Pressure difference? Ibig
#tunica intima= site ng atherosclerotic plaque sabihin dapat ang CVP mo mababa (N=0 mmHG) para from
#Sympa adrenergic(norepinephrine)= venous circulation pupunta sya sa Right atrium.
Bind A1= Vasoconstriction (peripheral BV)
Bind Beta 2= Coronary Vasodilation ABP/ Arterial blood pressure
#Sympa Cholinergic(acetylcholine)= -force exerted by volume of blood on arterial wall.
Bind M3 M4= Vasodilation (peripheral BV) a. Systolic Pressure(SP)- highest during ventricular
#Smooth muscle (BV)= Ca binds to calmodulin, walang systole
troponin C dun! b. Diastolic Pressure (DP)- lowest during ventricular
#myosin dephosphorylation= Relaxation diastole
# LATCH BRIDGE MECHANISM- c. Pulse pressure= SP-DP
Inc tone ng Vascular wall -> Dec ang Vascular
capacity -> Dec capacity of BV to hold blood -> Inc venous #Kelan wide ang pulse pressure? Meaning malayo ang pagitan
returnnn ng Systolic at Diastolic pressure mo
#Biggest pressure drop= From Artery to arteriole  Exercise- Sympathetic sa lahat (inc SP), pero dahil
#Lowest pressure= Vena Cava may heat production at inc metabolism= dec ang
peripheral resistance (dec DP)
 Hyperthyroidism- same with exercise

Page 7 of 89 By: Ayesha Bea Federizo Batch 2017


GRAND PHYSIOLOGY B FINALS REVIEWER

 Atherosclerosis- SP increase, pero walang Dec sa DP #kaya kapag may hypoproteinemia ka may edema ka
kasi walang nagpupull ng fluid mula interstitium mo pabalik ng
MAP or Mean Arterial Pressure Intravascular compartment
=DP x 2 + SP/3 or DP + 1/3 of Pulse Pressure
4. Interstitial Fluid Pressure or Interstitial Fluid Hydrostatic
ABP= CO x TPR (total periph resistance) Pressure (Pif)
Increased pag Inc ang blood volume, tapos ano - Force exerted by the volume of fluid in the
mangyayare pag increase ang Blood volume? Dapat interstitial space ; water pushing from Interstitium pabalik ng
alamonayan! So mag inc ang EDV/preload tapos alam mo na Intravascular compartment
talaga yan!!!
 Which Favors movement of fluid from Intravascular to
TPR (total periph resistance)- main factor is Interstitial?
VASOCONSTRICTION -Interstitial Fluid Colloid Osmotic Pressure or
Interstitial Fluid Osmotic Pressure (Пif) and
Test of knowledge: Dahil tapos ka na sa CVS 4, pag may mali -Capillary Hydrostatic Pressure (Pc)
back to start hah!
Interstitial to Intravascular?
Ano pinaka importanteng factor sa EDV? E sa ESV? -Interstitial Fluid Pressure or Interstitial Fluid
Anong nga uli yung iniinhibit ng Cardiac Glycoside? Hydrostatic Pressure (Pif)
Anong pagkakaiba ni Latch bridge at Bainbridge bukod sa -Plasma Colloid Osmotic Pressure (Пp)
spelling at pronunciation?
Anong phase sa Ventricula Muscle AP may equal conductance
ng K at Ca?
Sino responsible sa AP ng SA node? E ng Ventricular muscle?
Ang pulse pressure ba ay ang pressure ng pulse?

CARDIOVASCULAR PHYSIOLOGY 5
Microcirculation

 Left side of Heart Aorta Arteries Arterioles


Metarterioles  Venules  Veins Right side of *If the value is POSITIVE = Favors FILTRATION
Heart (occurring at the arterial end) fluid will move out
* If the value is NEGATIVE =Favors ABSORPTION (at
4 Forces Responsible for Filtration and Absorption to take the venous end) = fluid will move in
place =STARLING’S FORCES #Autoregulation

1. Capillary Hydrostatic Pressure (Pc) A.Active hyperemia- pag inc ang tissue
- force exerted by blood on capillary wall; in short metabolism-> inc oxygen consumption->
fluid pushing papuntang interstitium! Hypoxia-> vasodilation-> inc blood flow so ma wawash out
din un metabolites
Factors: B. Reactive Hyperemia- reaction pag may
A. Arteriolar Dilation- pag dilated ang arteriole inc occlusion sa BV; occlusion->Hypoxia->
ang CHP vasodilation-> inc blood flow so ma wawash out
B. Venoconstriction- Pooling of blood sa Capillary din un metabolites
so Inc ang CHP
C. Inc Venous pressure- diba ang flow dahil sa  Myogenic Theory
pressure difference? O edi pag ganyan, din a When BP inc, blood flow should also inc, only applicable
maka flow un blood, so pooling uli sa Capillary so between BP of 75-175 mmHg
Inc uli CHP If BP decrease below 75 mmHg ↓ Blood Flow
D. Inc Arterial BP- pag ganyan Inc ang blood flow, If BP rises above 175 mmHg ↑ Blood flow
so madami din dugong pupunta sa Capillary, edi
inc ang CHP Vasoconstrictors: Vasodilators:
E. Increased Total blood volume (TBV)- paulit ulit na Norepi/ Epi Prostacyclin
to kung bakit hah! Alamonayan  Endothelin Nitric Oxide
Angiotensin 2 Bradykinin
2. Interstitial Fluid Colloid Osmotic Pressure or Interstitial Vasopressin Histamine
Fluid Osmotic Pressure (Пif) Serotonin Inc K, Mg, CO2 & H+
- fluid pulling pabalik ng Interstitium Inc Calcium Acetylcholine
- not much of a problem unless sira capillary
membrane mo Mechanisms to Regulate BP

3. Plasma Colloid Osmotic Pressure (Пp) Neural Mechanisms


-force exerted by the presence of osmotically active 1. Provides Immediate BP regulation
substances in the plasma that will attract fluids a. Baroreceptor reflex- mechanical stimulus,
- fluid pulling pabalik ng Intravascular compartment i.e inc or dec in BP

Page 8 of 89 By: Ayesha Bea Federizo Batch 2017


GRAND PHYSIOLOGY B FINALS REVIEWER

2 types of receptor
a.1 Carotid Sinus
- more sensitive, stimulated by either increase or 2. Mechanisms that provide intermediate BP
decrease in BP regulation
-Impulse Gen by CN IX or Glossopharyngel nerve, a. Stress relaxation mechanism
more accurately the Herring’s nerve
a.2 Aortic Sinus
- stimulated only by increase in BP
- Impulse Gen= CN X or Vagus

b. Capillary Fluid Shift

b. CNS System Ischemic Response

c. Renal Fluid Shift

 Cushings Reaction, nangyayare to pag may


Brain tumor or pag inc ang CSF e yung fluid sa CNS.

c. Chemoreceptor reflex
- Stimulated by chemical stimulus, i.e dec O2
inc CO2, Dec pH or Inc H 3. Long term BP regulation (within several hours)
- Carotid and Aortic Receptors din

a. RAAS
Angiotensin 2:
 Potent Vasoconstrictor = Inc Tpr-> Inc ABP
 Stimulate adrenal gland to secrete Aldosterone-
so Na Reabsorption sa DCT-> Inc ang Blood Vol->
alamonayan
 Stimulate Hypothalamus to secrete ADH -> so Na
Reabsorption sa DCT-> Inc ang Blood Vol->
alamonayan

Page 9 of 89 By: Ayesha Bea Federizo Batch 2017


GRAND PHYSIOLOGY B FINALS REVIEWER

RAAS mechanism: ST segment elevation= Myocardial Ischemia


ST segment depression=Myocardial Infarction

#Bundle branch block= QRS> 0.10 secs; longer wider QRS


#Peak T waves= Hyperpataasemia (Inc K)

#Prolonged ST and QT interval= Hypocalcemia

#Cardiac Enlargement= Very tall R waves, Deep S waves


and QRS interval > 0.10 secs

#In cases of Orthostatic hypotension, ang mechanism na


nagwowork is yung BARORECEPTOR REFLEX
#
#Coronary Blood flow of the heart is BEST during
DIASTOLE, kasi hindi sya nacocompress unlike in systole

Neural Mechanisms in Regulating BP


Immediate Intermediate Long term
Baroreceptor Stress Relaxation RAAS
(angiotensin+
aldosterone)
CNS Ischemic Capillary fluid Aldosterone
Response shift
Chemoreceptor Renal Fluid Shift
Angiotensin

CARDIOVASCULAR PHYSIOLOGY 6
ECG

ECG WAVESSSSS

 P wave- Atrial Depolarization; depolarization of


whole atrial muscle
 Q wave= Interventricular Septum depo
 R wave= Apical depolarization
 S wave= Base depolarization
 PR segment- AV nodal delay
 PR interval- duration of atrial depo + AV nodal
delay
 T wave- only represent part of Vent Repo; tells
duration of phase 3
 QRS complex- Ventricular Depolarization-Phase 0
 QT interval- Ventricular muscle Action potential
or Vent muscle Depo and Repo; electrical systole
of the heart
 ST segment-after depo of ventricle; Phase 2
(remember Plateu?)
 ST interval- Ventricular Repolarization; Phase 1-3
 U wave- Uncommon; HypUUUUkalemia (dec K)

#J point- end of depo, start of repo, in isoeltric line #Most common Cardiac Vector= 59 degrees
#Left side- most common problem

Page 10 of 89 By: Ayesha Bea Federizo Batch 2017


GRAND PHYSIOLOGY B FINALS REVIEWER

TEST OF KNOWLEDGE Bago sa last topic para either ma


stress ka or maging confident ka >:)

>Represents Ventricular Repo? T wave? ANONGGG T


WAVE?????
>paki explain ang frank starlings law!
> ano mas sensitive? Carotid o Aortic Bodies?
> San nag bibind ang Ca sa Smooth muscle?
> Saan galling si Renin?
>san makikita ang C wave? Ang V wave?
>Lead I + Lead III= ?? 4?? Anong 4???
> Formula ng MAP?
> Capacitance vessel?
> Differentiate SV vs CO
>other name for EDV?
> Pareho ba ang ESV sa Afterload?
>anong pinagsasasabe ni Ohm’s law? E ni Poiseuille? Pano
ito ipronounce?
> other term for Asystole? Nice to know lang to, dali
macurious ka I google mo! 

#Einthoven’s Eq= Lead II= Lead I + Lead III

PAPASA KA!  Power of Words, Power of Faith! PRAYERS


PRAYERS!!!!! GODBLESS SATING LAHAT!

Tatabihan kita pag natulog ka.

Page 11 of 89 By: Ayesha Bea Federizo Batch 2017


GRAND PHYSIOLOGY B FINALS REVIEWER

BLOOD PHYSIOLOGY 1 2. Recipient is Rh  Rhogam-


Functions of blood: Composition of negative and donor given within
 Homeostasis blood is Rh positive 72 hours after
 Respiratory 1. Plasma giving birth to
 Nutritive 2. Formed Rh positive
 Excretory elements baby
 Endocrine a.RBC
 Immunity b.WBC
c.Platelets
Red Cell production:
 Embryo- Yolk sac  Maturation
 2nd trimester -Liver of RBC- need
spleen lymph nodes ang Vit B12
 3rd trimester to 5 y/o- at Folic Acid,
Bone marrow pag wala
 >20 y/o-Bone magkaka
marrow (flat bones) MEGALOBLA
STIC
ANEMIA

RBC Hypertonic -
-transport hgb which Crenate/shrink
carries O2
-120 days life span; 1% Isotonic- NR
replacement everyday;
destroyed in spleen Hypotonic-
Swell/burst
Sickle cell anemia- sickle
shaped RBC; abnormal beta
chain; causes hemolysis
and sickle cell crisis!
ANEMIA POLYCYTHEMIA VERA

Causes: Hemorrhage, IDA, Causes: Genetics, High


Aplastic anemia, Hemolysis, altitude
SCA etc
Normal Cardiac
Greater Cardiac Workload Workload
Rh Blood Group (1) magdedevelop si
mother ng Antibody,
Reactions Happen when: magdedevelop pa lang Major Reaction in cross matching:
1. Mother is Rh kaya yung first baby a. donor’s cell and recipient’s serum
Negative & Baby is safe pa , unless b. donor’s agglutinogen and recipient’s
Rh Positive nakagawa na sya ng agglutinin
(hemolytic disease antibody if kunwari
of the nasalinan sya ng Rh
newborn/Erythrobl Positive na blood(2).
astosis Fetalis)

Page 12 of 89 By: Ayesha Bea Federizo Batch 2017


GRAND PHYSIOLOGY B FINALS REVIEWER

BLOOD PHYSIOLOGY 2 For Acquired or Adaptive Antigen Presenting cells


Complement System Immunity: AKA
Para ma activate si C1  Interleukin 2- tagasumbong/tagatawag
Innate Immunity = C3 need mo ng antigen Stimulate ng backer kay CD8 & CD4:
Acquired or Adaptive= C1 antibody complex diba? So Cytotoxic T cell  Dendritic Cells
meaning dun nag sstart and Suppressor T  Macrophage
 C3b- Opsonization ang acquired immunity cell and T helper  B-Lymphocyte
&phagocytosis cells
C3 naman, sa innate
 C5B6789- Lysis immunity kasi diba, sa C3,  Interleukin 4,5,6 -
papatayin nya agad, for stimulates B cell
 C5a- chemotaxis opsonization agad, so stimulating factors
excited sya, kaya sa Innate
 C3a C4a C5a- immunity sya, hindi nya IgG= Gadami (most IgA= Secretions eg
Activation of mast need ng Ag-Ab complex, abundant) ; Gamommy Colostrum from Breastmilk
cells and basophils carry nya na agad. (crosses placenta) ; Gronic
(chronic) infection, first to IgE= Ellergies (allergies)
Antibody appear sa 2nd or
subsequent infestation IgD= Dimaunawaan;
 Constant portion=  vAriable Portion= attaches to B lymphocytes
Activates the where Antigen IgM= Mauuna; Malaki;
Complement Attaches first to appear sa 1st or Most abundant to least
system initial infestation (order) : G A M D E
Cytokines
For Innate Immunity: Cell Mediated Immunity Humoral Immunity ( B)
 Interleukin 12 – T helper cells= inactivated (T)
calls Natural Killer in AIDS Chimumea
cells  Transplant
Suppressor T cells and T Rejection  Chicken pox
 Interferon y- helper Cells= REGULATORY  TB  Mumps
secreted by NK CELLS  Tancer cells  Measles
cells to activate (cancer)
macrophage CD4= helper T cells  Tungi (fungi)
*yung 4 mukang H   Tiruses (virus)
 Interleukin 1 &
TNF- chemo CD8= Cytotoxic T cells
attractants, calls Active Natural= own Ab Passive Natural (maternal
neutrophils; Allergy  Primary infections Ab)
secreted by  Acute= IgE eg primary chicken  Anti Rh antibodies
activated  Chronic= CD8 pox  IgG, IgA
macrophage  Anti Rh antibodies Passive Artificial (Ab
Active Artificial= Injected Injectables)
 Immunizations  Anti Rh Ab
 Rabies vaccine  Ig Injectables
 Tetanus toxoid  Hep B
 Antitetanus Serum
Active= Takes time pero  Rabies Ab
pangmatagalan (true love)  Antivenin
Passive= Mabilis, pero
pang madaliaan (fling)

Page 13 of 89 By: Ayesha Bea Federizo Batch 2017


GRAND PHYSIOLOGY B FINALS REVIEWER

BLOOD PHYSIOLOGY 3

Blood Coagulation Rate Limiting Factor=


 FORMATION OF PROTHORMBIN
ACTIVATING FACTOR= Activated
Factor 10 / 10a
Intravascular anticoagulants

 Glycocalyx= smoothness property


 Thrombomodulin=binds to thrombin
 Thrombomodulin-Thrombin complex=
activates Protein C
 Protein C= Inactivates Factor 5 and 8

 Fibrin Fibers=Adsorbs thrombin


 Antithrombin 3= block and inactivate
thrombin
Fibrin fibers and Antithrombin 3= strong
anticoagulants
 Heparin= low concentration in blood.
Of little use alone

 Antithrombin 3+ Heparin= POWERFUL


ANTICOAGULANT; inactivates factor
9a, 10a, 11a, 12a and thrombin
 Needs Ca= Factor 2, 7, 9 & 10; guyton: all in intrinsic
pathway except factor 12 and 11
 Needs K= Factor 2, 7, 9 & 10; protein C
Others key points:
 Needs Phospholipids= Factor 10 and 2
Thrombostenin= clot retraction
 Thrombin stimulates activation of Factors 5, 8 & 13
 Intrinsic Pathway= Collagen contact, blood trauma ;Partial
Plasmin= Lysis of fibrin fibers, Factors 1,2, 5, 8
thromboplastin time(PTT); Heparin therapy
and 12 (antidote=Protamine Sulfate) (HPITT)
 Extrinsic Pathway= trauma to vascular wall and tissue
Coagulation factor present in Intrinsic and Prothrombin time (PT); Coumadin (warfarin) therapy
Extrinsic= 4 / Calcium (samplex) (antidote= Vit K) (PWET)
 Warfarin= opposes VCOR c1 (vit k epoxide reductase
Fibrinogen= naturally occurring anti coagulant complex 1)
VCOR c1= reduces back Vit K to functional form
t-PA meds like Streptokinase converts So Warfarin= No VCOR c1= No Vit K= No Coagulation
plasminogen to plasmin, it does not dissolve
clots. The PLASMIN dissolve clots.

Abruptio placenta= tissue factor source=


follows the extrinsic pathway

Page 14 of 89 By: Ayesha Bea Federizo Batch 2017


GRAND PHYSIOLOGY B FINALS REVIEWER

RENAL PHYSIOLOGY I  Constrict - Dec GCHP; kasi papunta pa lang yan ng


glomerulus, so kapag nagsara yan di di na nag flow
 For Homeostasis un blood
 State of equilibrium! Balance!  Dilate - Inc GCHP, better blood flow
 Maintenance of internal environment 8. Glomerulus
 What is more important is what you excrete! Lesser in
 High pressure capillary bed facilitating Filtration. High
what you take in.
pressure kasi un supply nya galing mismo sa
abdominal aorta (60-100 mmHg)
 Lungs – H20, CO2
9. Efferent Arteriole
 GIT – Undigested food, H20, meds, heavy metals,
 Controls pressure in peritubular capillaries or Vasa
electrolytes, etc.
recta bec it can constrict or dilate. Can also affect
 Skin – H20, electrolytes
GCHP
 Constricts - Increase GCHP kasi onti magfflow un
KIDNEYS
blood palabas ng glomerulus, pero pag nagsuper
 Main Role: Excretion of metabolic wastes and foreign
constrict na sya wala na talaga blood flow, ediba ang
chemical
blood may proteins, proteins imparts onctic pressure
 Regulates H20, electrolytes, plasma osmolarity, arterial so magkakaron ng water pulling, so maooppose ang
pressure (RAAS), acid base balance, volume and
filtration, reabsorption ang mangyayare
composition of ECF
 Dilates - Edi mas maganda blood flow so mas mag
 For excretion, secretion and metabolism of hormones fflow un blood out of the glomerulus, edi dec GCHP
 For gluconeogenesis 10. Peritubular Capillaries/Vasa recta
 Endo functions:  Give rise to venous drainage; give O2 and nutrients to
 Erythropoietin - stimulates bone marrow to produce renal tubules; low pressure capillary- 13-15 mmHg
RBC 11. Interlobular Vein
 Renin - secreted by granular cells or JG cells, targets 12. Arcuate Vein
Angiotensinogen from liver to be converted to 13. Interlobar Vein
Angiotensin 1 14. Segmental Vein
 Vit D is activated in kidneys, for calcium absorption 15. Renal Vein
16. Vena Cava
HORMONES THAT TARGET KIDNEYS:
 ADH/Vasopressin - produced in hypothalamus, released
by posterior pit or neurohophysis
 Aldosterone - from the adrenal cortex, most potent
mineralocorticoid in the Zona Glomerulosa
 ANP - from the atria
 Calcitonin, PTH, Vit D - Calcium and phosphate metab
RENAL TUBULES:
RENAL CORTEX RENAL MEDULLA 1. Bowman's Capsule
ISOTONIC HYPERTONIC  Mula glomerulus kung san ka nag filter ng blood,
yung ultrafiltrate mo mapupunta sa bowman's space
 Composed of:  Composed of:
2. PCT
 Glomerulus  Loop of Henle
 Bowman's capsule  Medullary Collecting  Where majority of substances is reabsorbed and
secreted.
 PCT Duct
 Hula Exam Tip: Basta, kung di mo na alam kung san
 DCT
 Cortical Collecting duct na sesecrete or reabsorb, PCT na agad.
3. Loop of Henle
 Renal Hilum - where renal artery enters and renal
 Thin Descending Limb, Thick and Thin Ascending
vein exits
Limb
 Nephron - Basic functional units of kidney;
4. DCT
Approximately 1 M per kidney
 Where macula densa is, for Na tonicity sensor for
filtrate, hindi plasma! Filtrate!
CHARACTERISTICS OF KIDNEY: 5. Cortical Collecting Duct
 Retroperitoneal, located around costovertebral angle; 6. Medullary Collecting Duct
weighs 150g, bean shaped
 Cilia
BLOOD SUPPLY: - Encoded by cholecyteine 1 and 2
1. Abdominal Aorta
- Important for flow dependent K secretion
2. Renal artery
- Mechanoreceptors that detect tubular flow rate
3. Segmental arteries
- Chemosensors for calcium dependent pathways
4. Interlobar arteries
5. Arcuate arteries
JG APPARATUS
6. Interlobular arteries
7. Afferent Arteriole  JG cells/Granular cells- elaborates renin & erythropoietin;
 Contains JG or granular cells (renin and erythropoietin lines the afferent arteriole
production); part of JG apparatus  Macula densa- Na Tonicity sensor
 Can modify Glomerular capillary hydrostatic pressure  Lacis /extraglomerular mesangial cells

Page 15 of 89 By: Ayesha Bea Federizo Batch 2017


GRAND PHYSIOLOGY B FINALS REVIEWER

 For elaboration of diff subs that will control renal RENAL BLOOD FLOW THROUGH VASA RECTA:
blood flow  Receives <2% of total RBF, so hypoxemic incidents is
 For supporting structure higher
 A=J M=D || afferent=jg cells macula densa = DCT  Flow is sluggish - majority of h2O filtered na sa
glomerulus so concentrated na un mapupunta sa efferent
2 TYPES OF NEPHRON: arteriole and so on so sluggish na kasi mas viscous na
Cortical Nephron Juxtamedullary
Nephron RBF = Parterial – Pvenous
BLOOD Peritubular Vasa Recta F = (Parterial – Pvenous) / Renal Vascular Resistance
SUPPLY capillary; better BP = SV x HR x TPR
perfused
TONICITY Isotonic - So filtrate Hypertonic - filtrate  Increased P arterial = Inc flow
stays in the tubule, flows from tubules to  Increased P venous = Dec flow syempre mas madami un
di pupunta sa medullary interstitium; babalik sa heart edi bababa un magfflow sa kidneys
interstitium so greater concentrating  Increases resistance = Negative yan malamang, so bababa
capability kasi mas ang flow
mataas ang osmotic
potential AUTOREGULATION
7 CN is to 1 JN 1 JN is to 7 CN  Controls amount of plasma being filtered!
 BP of 80-160 mmHg arterial pressure is CONSTANT!
RENAL BLOOD FLOW:  If <80mmHg= dec RBF and GFR syempre mahina un flow
 Receives 20-25% or 1.1 L of cardiac output kasi diba may e
direct supply sya from Abdominal aorta  If > 160mmHg= inc RBF and GFR, edi malakas, edi better
 Normal Urine Output flow
 Adult: 0.5-1 ml/kg/hr
 Pedia: 1-2 ml/kg/hr  E pag 130???? sige ngaaa!!!
 Ave UO: 1,000 - 1,500 ml/day  Better RBF 90 mmHg o 150 mmHg????
 So mas prone sa dehydration ang pedia, pati din  Constant within 80-160 mmHg padin kasi ang nagaadjust
oldies is si resistance, sina efferent at afferent, interlobar arteries,
so constrict dilate constrict dilate kaya within that range,
same pa din ang flow! AMAAAAZINGGG
Ohm's Law Pouseuilles Law
F = Δ P/R; Flow= Pressure R = 8lη/Πr4 TUBULOGLOMERULAR FEEDBACK MECHANISM:
gradient / Resistance  GFR and RBF control when BP dec
Diba lahat naglalaro sa pressure Measure of resistance!!!  So BP dec  Mag dec din GFR kasi syempre mababa
gradient, tigan mo yan in a pressure dibaaa so less filtrate is produced, so dec filtrate
positive way, si resistance flow sa tubules, pag dec ang filtrate flow ibig sabihin mas
naman in a negative syempre mabagal un flow so mas mahaba un time for the filtrate na
resistance nga e. kakaunti to be reabsorbed sa tubules papuntang
 Inc flow pag ⬆Pressure  Inc resistance pag circulation, mag fflow flow tapos pupunta ng DCT  Dec
gradient and/or ⬇ang mas mahaba ang Na conc sa DCT kasi nga nareabsorb mo na, madedetect
resistance vessel, mas viscous un ni Macula Densa, sstimulate nya si JG cells to produce
 Dec flow pag ⬇Pressure ang blood at mas Renin  RAAS activation  Angiotensin 2 activated 
gradient and/or ⬆ang maliit ang radius ng Inc BP and Efferent arteriole constricts  Inc GFR
resistance vessel (constricted)  Afferent arteriole dilates  Inc GFR

RBF MEASUREMENT: MYOGENIC MECHANISM:


 Direct Method  Regulates pressure and RBF when BP inc
 Tedious, invasive and impractical  Inc BP  Inc RBF and GFR initially  Arterioles stretches
 Indirect Method kasi Inc un circulating blood vol sa kidney  Vasodilation
 Utilizes Clearance principle- volume of plasma cleared  Mech gated Ca channels activated  Contraction 
of a substance per unit time Reflex Afferent arteriole constriction  Dec GCHP  Dec
GFR
A. Get PAH clearance (infuse PAH to patient)
B. Get Urine sample and determine PAH conc OTHER MECHANISM:
C. Get Blood sample determine plasma conc of PAH  High CHO or CHON diet
 Inc glucose conc in blood – So need mo sya i filter 
 Clearance of plasma PAH actuates Renal plasma flow; it When CHO is reabsorbed , co transport mechanism
doesn’t enter the cell, it stays in the plasma happens, So Na is also reabsorbed  So sa DCT,
 PAH is filtered and secreted madedetect nanaman ni macula densa na dec un Na
 RBF = RPF / (1 – Hct) conc ng filtrate nya - Activates tubuloglomerular
 C = UV / P (Clearance= Urince conc x Urine vol / Plasma feedback, so afferent arteriole dilates and efferent
conc) arteriole constricts  Inc GFR
 High CHON intake  All should be filtered so it again
Activates tubuloglomerular feedback - if efferent

Page 16 of 89 By: Ayesha Bea Federizo Batch 2017


GRAND PHYSIOLOGY B FINALS REVIEWER

arteriole is severely constricted, blood flow to vasa FILTRATION BARRIER:


recta or peritubular capillaries will severely dec   Bowman’s Epithelium
Ischemia  Acute tubular necrosis  (+) slit pores and podocytes
 (+) Sialoglycoproteins
 Sympathetic or Neural Innervation is not important in  Basement Membrane
URINE FORMATION  (+) proteins type 4 collagen, laminin, polyanionic
 Blood vessels constricts -> dec RBF -> Dec Filtration -> proteoglycan entactin, heparan sulphate and
dec urine volume (inc urine osmolality) mas konti mas fibronectin
concentrated  Lamina rara externa
 Lamina densa
 Severe sympa:  Lamina rara interna
 Oliguria= 300-500 ml/day of urine  Capillary endothelium
 Anuria- less than 50 ml/day of urine  (+) fenestrated , Sialoglycoproteins
VASOCONSTRICTORS VASODILATORS
Angiotensin I & II Nitric Oxide  100x more permeable ang glomerulus dahil sa mga slit
pores, fenestrations, podocytes , yung mga plasma
Epinephrine Histamine
proteins or negative particles di makapenetrate kasi may
Norepinephrine Prostaglandin (PGE2, PGI2) mga proteins din un mismong filtration apparatus, so
Endothelin Bradykinins (Produce Nitric narerepel nya un mga plasma proteins, plus big size sila
Oxide) like un mga formed elements. Plus high GCHP pa sya kasi
Anesthetics Dopamine diba direct blood supply sa Abd Aorta.
Angiotensin Receptor Pyrogens: Interleukin, Bacterial
Blockers endo and exotoxin MESANGIAL CELLS:
ACE inhibitors  antagonist  Support structure within glomerulus
ANP BNP β antagonist  Properties: contractility, phagocytosis
 Produces matrix for support of basement membrane
 No parasympathetic innervations Prostaglandin, and other important proteins
 NSAID causes Renal Failure because it decreases the
production of prostaglandins necessary for vasodilation,  Nakakapag regulate din sila ng RBF kasi may
so puro vasoconstriction  Ischemia  Renal failure! prostaglandin which has a vasodilating effect

RENAL PHYSIOLOGY II FILTERABILITY:


 Glomerular Filtration  Eg Na Glucose: F= 1.0 , meaning 100% is filtered, lahat
 Tubular Reabsorption pumunta sa renal tubule for possible excretion, but still
subject for reabsorption
 Tubular Secretion
 Eg. Myoglobin: F=0.75, meaning only 75% is filtered
 Highest to lowest filterability
1. Positively charged
2. Neutrally charged
3. Negatively charged

 Mas maliit mas madali i filter

GIBBS DONNAN EFFECT:


 Behavior of charged particles near a semi-permeable
membrane that sometimes fail to distribute evenly across
the two sides of the membrane. The usual cause is the
presence of a different charged substance that is unable to
pass through the membrane and thus creates an
uneven electrical charge.
 Eg. Nagfilter ka, matitira sa plasma mo e si Albumin,
albumin is negatively charged, so kung may mga to be
filtered pa sa plasma dina sya makakadaan papuntang
tubules kasi un mga positively charged particles na ififilter
sana e kay albumin na naattract.

EFFECTIVE FILTERING PRESSURE:


 Substance A- Filtered Only
 Substance B- Filtered, Partly reabsorbed
 Substance C - Filtered and Completely Reabsorbed
 Substance D -Filtered, None Reabsorbed + Secretions

GLOMERULAR FILTRATION
 Nonselective, as long as it is small and positively charged,
it is filtered indiscriminately

Page 17 of 89 By: Ayesha Bea Federizo Batch 2017


GRAND PHYSIOLOGY B FINALS REVIEWER

 Glomerular Hydrostatic Pressure generated by the


pumping action of the heart, hydrostatic pressure so yung
blood ipupush nya palabas papuntang renal tubules
filtrate
 Bowman’s Oncotic Pressure normally 0 in value because
proteins are not filtered. Oncotic, meaning water pulling
effect, so yung asa tubule na , hihigop sya ng water edi
dagdag filtrate
 Bowman’s Hydrostatic Pressure due to the presence of
filtrate inside the Bowman’s space, so yung fluid sa filtrate
ipupush nya pabalik ng systemic circulation MECHANISM FOR REABSORPTION:
 Glomerular Oncotic Pressure plasma proteins present in A. ACTIVE TRANSPORT - uses energy; uphill
the glomerulus, again ang proteins nagiimpart ng oncotic 1. Transport Maxima
pressure, so nag pupull yan ng water, so mula tubule  Energy and carrier dependent
hihigopin ng plasma proteins yung water pabalik sakanya,  How Glucose, amino acids, PO4, SO4, Vitamin C,
sa systemic circulation malate, lactate, aceto-acetate, βhydroxybutyrate
are handled.
FILTRATION COEFFICIENT:  In DM patients, sobrang daming nafilter na
 Permeability: glucose but since may certain amount lang na
Inc kayang i handle ang carriers mo, di nya
Permeability marereabsorb lahat kasi saturated na lahat, wala
Inc Kf  Inc ng magbabalik sa ibang glucose kaya merong
GFR; so eh nag glucosuria.
thicken ang 2. Gradient Time Limitation
membrane, edi  How Sodium and Bicarbonate are handled
mas mahirap  Inc Gradient  Inc Reabsorption
ipenetrate, dec ang Kf  Dec GFR Oliguria  Inc time of contact within epithelium  Inc
 Surface Area: Inc SA  Inc Kf  Inc GFR; so for example Reabsorption
partial nephrectomy, o edi mas konti un nephrons mo na  Syempre pag mas matagal un filtrate mo sa
makakafilter  Dec Kf  Dec GFR epithelium mas may time mag reabsorb ng Na or
 Mesangial Cell Activity: Vasodilating property via HCO3 yung tubular cells mo. Think of
prostaglandin  Inc Kf  Inc GFR tubuloglomerular feedback!
3. Pinocytosis
QUANTIFICATION OF GLOMERULAR FILTRATION:  Cell drinking
 Inulin – Gold  How PROTEINS  Amino Acids are handled
standard in
measuring GFR, B. PASSIVE TRANSPORT - no energy needed; downhill
it is only filtered  How WATER, UREA and CHLORIDE are handled
plus almost
same sya with Quantification of Reabsorption:
normal GFR, but expensive and not readily available  R= filtered – excreted
 BUN (Blood Urea Nitrogen) – Filtered and reabsorbed so  RX = (GFR x PX) – (UX x V)
it underestimates GFR  If the filtered load > excretion rate, net reabsorption has
 Creatinine – Filtered and Secreted so it is overestimated occurred

FILTRATION FRACTION: FACTORS AFFECTING TUBULAR REABSORPTION:


 Flow rate – Inc Flow rate  Dec Reabsorption
 Osmotic pressure of the filtrate  If Inc Osmotic P  Dec
Reabsorption
 Hormone influence

ADH (Inc Water absorption)


Aldosterone ( Inc Na reapbsorption)
 Inc FF= Inc Plasma proteins. ANP( Dec Na reabsorption)
 So yung plasma mo nafifilter sya, natitira un mga plasma Angiotensin II (Inc Na Reabsorption)
proteins, naiipon. PTH, (Inc Calcium Reabsorption; Dec PO4 reabsorption
Vit D (Inc Calcium and PO4 rebsorption)
 Higher Inc FF sa Efferent arteriole kasi mas increased na Calcitonin (Dec Ca and PO4 Reabsorption)
yung oncotic pressure dun kasi nakadaan na ng
glomerulus

TUBULAR REABSORPTION
 Selective, purpose is to bring back essential substances
into the body.

Page 18 of 89 By: Ayesha Bea Federizo Batch 2017


GRAND PHYSIOLOGY B FINALS REVIEWER

2. LOH

 Exposed in Hypertonic Medium


a. Thin Descending Limb
Water Reabsorption ONLY
PI – Fluid Pressure na magpupush pabalik sa systemic circ b. Thin Ascending Limb
ΠC – Oncotic or water pulling effect pabalik ulit ng systemic Passive reabsorption of Solutes ONLY; DILUTING
circ SEGMENT Diluted filtrate
PC – Fluid Pressure na magpupush mula capillary papuntang c. Thick Ascending Limb
interstitium Active reabsorption of Solutes ONLY; DILUTING
ΠI – Oncotic or water pulling effect pabalik ng interstitium SEGMENT Diluted filtrate
 Since only water is reabsorbed in Thin Desc Limb
REABSORPTION: Inc filtrate conc= concentration gradient  In thin
 Powered by Pumps Asc Limb, NaCL is passively reabsorbed
 Na-K Pump  concentration gradient between your  Claudin 16- tight junction; permits Mg or Ca passage
filtrate and your tubular cell  Power for Secondary  When Filtrate exits LOH: Hypotonic na yung Filtrate
pump  Na from your filtrate goes into cell via kasi puro solute un nireabsorb mo sa ascending limb,
secondary pump  transport substance X = every Na so halos puro water un matitira
that is pumped out, a Substance X is reabsorbed
 For anion eg Chloride, it causes electrochemical Barters’ Syndrome & Na-K-Cl Symport Defect:
gradient powering passive transpo, so didikit lang sya  K is not reabsorbed= Hypokalemia
kay Na since opposite attracts, sabay na silang  Na is not reabsorbed= Hyponatremia  Dec Circ volume
marereabsorb RAAS activation
 Back Leak  Aldosterone reabsorbs Na in exchange for H, HCO3 is left=
 Commonly happens with Potassium Metabolic Alkalosis
 Pabalik ng filtrate
 Via paracellular route or in bet cells 3. DCT & CD

SEGMENTS OF NEPHRON:
1. PCT

a. Early DCT
 Reabsorption of Solutes ONLY; DILUTING
 SITE OF OBLIGATORY REABSORPTION SEGMENT
 Reabsorbs 67% of filtered H2O, Na, Cl, K  Fluid leaving this segment is then HYPOSMOTIC
 Requires tons of Mitochondria b. Late DCT & CD
 Commensurate Water reabsorption- amt of solute =  FACULTATIVE REABSORPTION OF WATER
amt of water reabsorbed  Influenced by ADH- aquaporin formation
 Filtrate tonicity= ISOTONIC & ISOSMOTIC  Without ADH= 8% H2O reabsorption
 Solvent drag via paracellurar route- flowing water  With ADH= 17% H2O reabsorption
drags along solutes, and since changes in Na  Aldosterone- Inc Na-K pump, So Na is
reabsorption influences H2O and solute reabsorption , reabsorbed, K is excreted ;targets Principal and
pag dec ang Na conc sa filtrate  dec reabsorption Intercalated cells
din yung ibang solutes  K-H pump, K goes inside Intercallated cells, H is
excreted = alkalosis (hyperaldosteronism)
Renal Tubular Acidosis - Na-HCO3 transport defect,
no HCO3 reabsorption= acidosis Distal Renal Tubular Acidosis = Metab Acidosis,
Cystinuria - Amino acid transport defect= Hypokalemia, Hypercalciuria, nephrolithiasis
aminoaciduria Liddles Syndrome- Amiloride-Sensitive Na
Channels= Dec Na excretion Hypernatremia
Hypertension

Page 19 of 89 By: Ayesha Bea Federizo Batch 2017


GRAND PHYSIOLOGY B FINALS REVIEWER

TUBULAR SECRETION GLOMERULOTUBULAR BALANCE:


 Same with reabsorption but direction is different  If there is Inc in Tubular load  Inc GFR  Inc filtrate 
 Any substance ↑ concentration to a greater extent than Counteracted by Inc in Reabsorption rate
does inulin is secreted  Purpose:
 Prevents overloading of distal segments with an ↑ GFR
MECHANISMS FOR TUBULAR SECRETION:  Prevents large changes in fluid flow in the distal
 Na-K pump concentration gradient  Power for tubules brought about by:
Secondary pump (more often ANTIPORTS)  So for every - Changes in arterial pressure
Na reabsorbed, substance X goes out into filtrate = every - Disturbances that would create problems in the
Na that is pumped out, a Substance X is secreted into the maintenance of Na and vol homeostasis
filtrate
TEST OF KNOWLEDGE
Active Secretion:  Reabsorption of Amino Acid? PCT
 Transport maxima  Needs most Mitochondria? PCT
 Carboxylic, sulfonic acids, hippurate, creatinine,  Obligatory Reabsorption? PCT
penicillin, thiazide, glucoronides, urological contrast  Facultative Reabsorption? PCT??? Nanaman! Mali!
 Organic bases, guanidine, thiamine, choline, histamine  Diluting Segments? 3 yan!
 Gradient-time limited  Specific Mechanism of Secretion ng H? K?
 How H & K are handled  Diff Glomerulotubular balance vs Myogenic Mech vs
Tubuloglomerular feedback
Passive Secretion:  Reabsorption mechanism ng Urea? Creatinine? Glucose?
 Diffusion trapping  Inc BP  GFR will??
 Weak bases, quinine, quinacrine, procaine,  Afferent arteriole Dilates  GCHP will?
chloroquine, NH3  Efferent arteriole moderately constricts  GCHP will?
 Weak acids, salicylic acid, phenobarbital  Asan si Macula densa? Si JG cells?
- Weak bases and acids are unionized, meaning no
charge, once filtered returns to ionized form RENAL PHYSIOLOGY III
(positive and negatively charged)  if this
happens, they can no longer cross back into the REGULATION OF PLASMA AND WATER VOLUME
cell membrame, dun na sila forever, then KIDNEYS:
excreted  Maintain plasma osmolality of 282 mOsm/L
 Antidote for Aspirin poisoning = Sodium Bicarbonate  Conserve fluid and excrete solute
(it follows the diffusion trapping mech)  Normal urine osmolality= 50-1400 mOsm/L
 50- diluted || 1400- concentrated
SEGMENTS OF NEPHRON:  Normal Urine Specific gravity (measure of the
concentration of solutes in the urine)
 1.002-1.400
 1.002- diluted ||1.400- concentrated
 Plasma specific gravity= 1
BLOOD:
 5 Liters  3L plasma & 2L of formed elements

ADH
 Synthesized within hypothalamus in the supraoptic nuclei
(mainly) and paraventricular nuclei
 Posterior pituitary only functions as for storage and
release
 For water reabsorption in Late DCT and CD (aquaporin
 There is no secretion
synthesis)
in THIN DESCENDING LIMB;
 Inc in urea transporters: UT-A1, UT-A3 || NKCC2
wala din nabanggit sa Thin
transporter || NaCl symporter|| ENaC  Promotes solute
Ascending limb diba?? Di ko
reabsorption  Water goes along
alam!
 2 Stimuli
 Sa late DCT at CD,
 1% inc in plasma osmolality (more sensitive, day to
Aldosterone ulit gagalaw 
day basis)
Inc Na-K pump
 10% dec in BP and/or BV ( state of shock or
 Principal cells =
haemorrhage)
Handles K (secretes K)
 Intercalated cells =
Handles H (secretes H)  Antero-Ventral portion of 3rd ventricle- (+) osmoreceptors
 Indirectly detects plasma tonicity  If Plasma tonicity
QUANTIFICATION OF SECRETION: inc  Inc CFS (cerebrospinal fluid) tonicity 
Osmoreceptors will shrink  stimulates supraoptic and
 If the filtered load <
paraventricular nuclei  Posterior pituitary releases ADH
excretion rate, net secretion
 Targets late DCT and CD  H2O reabsorption
has occurred

Page 20 of 89 By: Ayesha Bea Federizo Batch 2017


GRAND PHYSIOLOGY B FINALS REVIEWER

 Baroreceptor and cardiopulmonary receptors- BV and BP ADH MECHANISM:


detectors  ADH attaches to V2 receptor  activated coupled
 Subfornical region and Organum vasculosum of lamina with G protein activating adenylyl cyclise  Inc cAMP
terminals - Thirst regulator  Protein kinase A activated  Aquaporin 2
(Regulated by ADH) exocytosis into luminal side ng
membrane  Water from luminal membrane will be
transferred to Circulatory system via AQP3&4 which
are always in the basolateral membrane = water is
then reabsorbed
 So kapag walang ADH, wala si AQP2, so walang
channel mula luminal side pabalik ng circulatory
system= no H2O reabsorption

Syndrome of Nephrogenic Syndrome of


Inappropriate Antidiuretic Inappropriate Antidiuresis
Hormone (SIADH)
 ADH  Water   V2 receptors 
 ADH  H2O reabsorption  Regulates plasma Retention= continuous AQP2
osmolality/tonicity  Hypoosmotic body production  even in the
 Aldosterone  Reabsorbs solute  Commensurate water fluids (diluted) Absence of ADH, (+) water
reabsorption = Isotonic fluid is reabsorbed Regulates  Hyperosmotic urine Retention=
volume (concentrated)  Hypoosmotic body
fluids (diluted)
 Hyperosmotic urine
(concentrated)

DI/DIABETES INSIPIDUS/DAMI IHI


CENTRAL DI NEPHROGENIC DI
 ⬇Neurophysin (shuttle of  (+) ADH but no
ADH going to Posterior RECEEPTORS  water is
Pituitary) =⬇ADH  water then excreted instead =
is then excreted instead = Polyuria {Hypoosmotic
Polyuria {Hypoosmotic urine (diluted)}
urine (diluted)}  Hyperosmotic body fluids
 Hyperosmotic body fluids (concentrated)
(concentrated) Polydipsia
Polydipsia

 Plasma Osmolality THIRST/UHAWWW


 <280/282 mOsm/L  ZERO ADH release  Stimulated with a:
 >280/282 mOsm/L  INC ADH release  2-3% Inc in Plasma Osmolality
 Blood Pressure/Volume  10-15% Dec in BP/BV
 Zero change or Inc in BP/BV  ZERO ADH  Thirst threshold > Threshold for ADH secretion
release  Thirst: 295 mOsm/kg H2O
 Dec BP/BV  INC ADH release  ADH: 285 mOsm/kg H2O
 Chronic Hypotension >270 mOsm/L  INC  Mas nauuna magsecrete ng ADH kesa sa feeling ng
ADH release UHAWW
 Chronic Hypertension <290 mOsm/L  ZERO
ADH release  Angiotensin II  Inc Thirst
 >290 mOsm/L --> INC ADH release  Pag uminom ka Stimulated ang Oropharyngeal &
 Chronic meaning matagal na, normally UGIT receptors mo, kaya maski hindi pa sya naaabsorb
mag rerelease ka lang ng ADH pag ng GIT  Dec Thirst agad
>280/282 mOsm/L ang plasma osmolality
mo, what happens in chronic  Coke Story: Coke contains solute Solutes Inc plasma
hypo/hypertensive condition is osmolality  Thirst threshold is reached again 
“nasasanay”yung body mo na mataas or INOM PA!
mababa yung BP, so nag aadjust sya, kaya  Alcohol Story: Alcohol contains happiness 
sa chronic HYPOTENSION >270 mOsm/L Happiness Inc plasma osmolality  Thirst threshold is
pa lang nagrerelease na sya ng ADH para reached again  INOM PA! (After hell week?
makaconserve na agad sya ng water. In GAMEE!!!)
chronic Hypertension naman
magrerelease lang ng ADH pag >290 WATER REABSORPTION
mOsm/L pa, so matagal kasi in HPN PCT
patients sanay yung body nila na conc  67% of H2O is reabsorbed (OBLIGATORY REABSORPTION)
yung plasma.

Page 21 of 89 By: Ayesha Bea Federizo Batch 2017


GRAND PHYSIOLOGY B FINALS REVIEWER

LOH: Descending Limb  Counter current exchanger (U shape kasi)- this also
 20% of H2O is reabsorbed minimizes solute wash out from medullary interstitium
LOH: Ascending Limb  PS: Actually dami nanamang sinasabi ni Guyton pero
 Solutes are reabsorbed NO H2O REABSORPTION basta, Si Vasa Recta Maintenance lang ng tonicity ng Renal
DCT Medulla, di sya nagcocontribute ng maski pisong mOsm.
 H2O permeability is ADH-dependent (late dct)  Inc Arterial Pressure Inc Renal Medullary blood flow 
(FACULTATIVE REABSORPTION) Inc wash out of solutes sa Intersitium  Dec ability of
CD urine to be concentrated
 H2O permeability is ADH-dependent (FACULTATIVE
REABSORPTION)  Absent ADH = Diluted Urine, syempre walang
 Without ADH = 8% H2O reabsorption magrereabsorb ng water edi ilalabas mo na lang yung
 With ADH = 17% H2O reabsorption water
 Present ADH = Concentrated urine, narereabsorb kasi
OBLIGATORY URINE VOLUME: water pabalik ng systemic circ, edi konti lang ieexcrete
 Minimal vol of urine that must be excreted mong water
 Serves as a dissolving and suspension medium
 Excretion: 600 mOsm/day INCREASE DECREASE INCREASE DECREASE:
 Concentrating ability : 1200 mOsm/L (renal medulla  Ganito, pag sa interstitium, pending for reabsorption yan
tonicity) ,yung vasa recta, ang role nya, ibalik yung ibang solute
 600 mOsm/day is to be excreted divide it with 1200 mula interstitium pabalik ng systemic circ para i conserve;
mOsm/L pag asa tubules, yan un pending urine
 0.5 L/day ( amount of fluid needed to remove 600  Renal medulla - hypertonic for concentration of urine
mOsm/day  Dec solute loss in interstitium mas macoconcentrate ang
urine, dadami yung pending for reabsorption
HYPERTONIC RENAL MEDULLA:
 Tonicity is 1,200-1,400 mOsm  Inc length of LOH more solute is pumped into
 NaCl- 600 (active transpo) interstitium Inc Osmotic Gradient Dec Solute loss in
 Urea-600 (Recycling of Urea) medullary interstitium
 Vasa Recta- only maintains tonicity  Inc # of Juxtamedullary nephrons  more solutes will
 Absorbs water via Osmosis with action of ADH enter medullary interstitium  Inc Osmotic Gradient
 For Urine Concentration (DCT, CD) Dec Solute loss in medullary interstitium
 With ADH  Water is reabsorbed into the Interstitium  Inc flow rate in LOH (vasodilation)  Dec Osmotic
 Vasa Recta (contains plasma proteins, imparts gradient  Inc solute loss or solute wash off
oncotic pressure or water pulling effect) returns water  Inc flow rate in vasa recta Dec Osmotic gradient  Inc
back into circulation thereby concentrating urine solute loss or solute wash off in Medullary interstitium
 Corticopapillary osmotic gradient- secondary to CC  Remember, sa vasa recta 1-2% lang ng RBF meron sya
mechanism, urea recycling and vasa recta; this will at dahil dyan decrease ang solute loss sa interstitium
minimize solute loss in medullary interstitium  Inc ADH vasoconstriction Dec RBF Dec GFR
filtrate is low filtrate flow slows down  more time for
COUNTERCURRENT MECHANISM: NaCl reabsorption (gradient time limitation to)  Inc
 ANG DAMING SINASABI NI GUYTON. Pero ito lang yan: Osmotic gradient  Decrease solute loss in Medullary
Maglalagay ka ng madaming solute sa medulla in excess interstitium
of water, in time, ma ttrap na lang dun yung mga solutes,  Inc Urea  more hypertonic ang medulla Inc osmotic
so mauulit ulit lang yung reabsorption ng NaCl by the gradient  Dec Solute loss in Medullary interstitium
thick ascending limb, tapos meron pang continuous inflow
ulit ng new NaCl galing ng PCT So paikot ikot lang FREE WATER CLEARANCE:
hanggang sa ma reach nya ang 1,200-1400

UREA RECYCLING:
 Mediated by ADH, diba nga ADH Inc urea transporters:
UT-A1, UT-A3.
 Urea is permeable only in Medullary collecting duct and
due to conc gradient babalik sya ng Ascending limb basta
paikot ikot lang din yan.
 Patients with protein malnutrition  Dec urea production
 Dec renal medullary tonicity  Dec urine concentration
= Diluted urine
 Estimates the ability to concentrate or dilute the urine
VASA RECTA:
 Rate at which solute-free water is excreted= water lang
 Medullary blood flow is low and sluggish= 1-2% lang ng
 Free H2O
RBF ang meron sya, pero sufficient na yon, plus dahil dun,
 Produced in the diluting segments of the kidney
minimized ang solute loss sa medullary interstitium para
- Thin and Thick Asc Limb & Early DCT
sakto lang yung ibabalik mo sa circulation since ang main
function nga ng kidney mo is to EXCRETE “wastes”

Page 22 of 89 By: Ayesha Bea Federizo Batch 2017


GRAND PHYSIOLOGY B FINALS REVIEWER

 +CH2O: -ADH  Positive water clearance, nilabas mo  will promote secretion of K or H


daw un water so ibig sabihin since nilabas mo ang water, Epinephrine,  Stimulated by dec in ECF volume
wala kang ADH urine is diluted norepinephrine  Inc NaCl and water reabsorption
 -CH2O: +ADH  Negative water clearance, wala kang  RAAS will potentiate sympathetic
inilabas na tubig, ibigsabihn may ADH ka urine is effects
concentrated Cortisol  Glucocorticoid produced by
 0 CH2O: +loop diuretic use (furosemide)  inhibited ang adrenal gland
NKCC2 in thick asc limb dec medullary tonicity  Will have mineralocorticoid effect
medulla becomes isotonic  water and solute stays in  Inc Na Reabsorption
tubules  urine is neither conc or diluted  BUT When filtered load >
 ADH - water conservation reabsorption rate Inc GFR 
 Diuretic - meds that promotes urination Dec Na Reabsorption
Redistribution to  More solutes will be pumped into
WATER REABSORPTION: deeper renal tissue the interstitium  Na
Reabsorption
Efferent Arteriole  Dec Hydrostatic Pressure in
Constriction Peritubular capillaries  Dec
NaCl and H2O backleak ( back
into tubular fluid)  Inc Na and
H2O reabsorption
Increase  Inc GFR  Inc Filtered load 
Glomerular plasma proteins accumulate and
Capillary imparts onctic pressure) water
Hydrostatic reabsorption
Pressure

ANTI-REABSORPTION:
ANP, BNP  Produced in Atrium (ANP) and
Ventricles (BNP)
 Will decrease NaCl and water
reabsorption in the collecting
REGULATION OF OSMOTIC EQULIBRIUM AND PLASMA ducts
IONIC BALANCE:
 Decrease the total peripheral
PRO-REABSORPTION:
resistance
 If ECF vol Dec  RAAS  decrease ADH secretion and ADH
Angiotensin II activation A II increases mediated water reabsorption
targets:
UROGUANYLIN,  produced in GIT and kidneys
 PCT: Inc NaCl reabsorption
GUANYLIN  stimulated by high salt diet
 Na-H antiport
 Targets PCT to decrease NaK pump
 Starling forces adjustment
and Na-H antiport Dec NaCl and
 LOH, DCT, CD:Inc Na reabsorption
H2O reabsorption
 Weak vasoconstrictor of afferent
 Target your DCT and CD
arteriole and strong
decreasing now your ROMK which
vasoconstrictor of efferent
is a K channel dec K
arteriole Dec GFR
reabsorption
 Mesangial cell contraction dec
surface area dec Kf  Dec GFR DOPAMINE  Stimulated by Inc in ECF volume
 Aldosterone release  Dec NaCl and water reabsorption
ADRENOMEDULLIN  Produced by kidneys
 ECF volume regulation via
 Stimulated by CHF & long
Aldosterone commensurate water
standing HPN
reabsorption
 Inc RBF Inc GFRDec NaCl &
  Na reabsorption
H2O reabsorption
 Na-Cl symport , Na-K pump,
ENaC, Sgk1, CAP1, prostatin URODILANTIN  Produced by kidneys
 LOH, DCT, CD: Inc NaCl  Stimulated by Inc ECF vol & long
reabsorption standing HPN
 DCT, CD: Inc K secretion  decreases salt and water
 Paracellular Cl reabsorption reabsorption
(sumama si Cl nung nireabsorb si
Na) ELECTROLYTES
 Inc aldosterone release if (+) SODIUM  Most abundant extracellular cation
hyperkalemia & RAAS  Determines the ECF vol- along with
 Dec aldosterone release if (+) anions (HCO3, Cl)
hypokalemia, ANP  DCT and CT reabsorption is concerned
 Will have glucocorticoid effect with acid-base balance

Page 23 of 89 By: Ayesha Bea Federizo Batch 2017


GRAND PHYSIOLOGY B FINALS REVIEWER

 high salt dietECF volume is Aldosterone also promotes water


increased Inc BP BV  counteracted reabsorption  flow rate dec  by
by inc in excretion of NaCl virtue of gradient time limitation  K
secretion goes down; so Net effect,
NA REABSORPTION constant or no change in K secretion
or excretion
 Dec aldosterone release if (+)
hypokalemia, ANP
 ADH Targets Principal cells Inc
Na uptake (hyponatremic effect)  K
secretion
 Pag pumasok kasi ang positive
ion (Na), dapat may lalabas din
na positive ion (K)

 BUT, ADH also reabsorbs water 


flow rate dec  by virtue of gradient
time limitation  K secretion goes
down; so Net effect, constant or no
change in K secretion or excretion

 Inc flow rate  K secretion Inc &


 Depends on Na Availability; for every
PKD1&2 Ca conducting channel is
POTASSIUM K excreted, Na is reabsorbed
also activated Ca goes inside cell
 Renal handling: filtered ; Reabsorbed
 K channels activated  K goes out
PCT: 67% LOH: 20% DCT,
CD:aldosterone dependent ||
 Inc Na in filtrate  Inc Na uptake
Secreted in DCT CD
pag may pumasok dapat may lalabas
 Regulation of K excretion:
 K goes out
 Aldosterone targets Principal cells 
Eg Dec Na in renal tubules  K
secretion Dec  Glucocorticoids  Inc K secretion
and also Inc GFR and other traspo
 Eg Inc H in renal tubultes 
mech
Intercalated cells are targeted K
secretion Dec
 Acidosis  Intercalated cells rather  Acidosis Dec K secretion;
loses H than K Intercalated cells rather loses H than
 Excretion and secretion is Primary K; Plus there is Dec NaK pump so Na
regulated by Plasma K goes outside cellInc Na Excretion
 aldosterone release H secretion
HYPOKALEMIC HYPERKALEMIC
EFFECT EFFECT Acute Chronic
Metabolic Metabolic
 Epinephrine: β2  Epinephrine: Acidosis Acidosis
Stimulation =  α stimulation
K uptake of cells =  K release  Na-K pump is  Mag wowork na
Inhibited + Dec si aldosterone
 Insulin- K uptake from cells, liver
of cells K mechanism 
permeability Inc Na-K pump
 Aldosterone- K
Dec K excretion and Inc K
uptake of cells &
permeability 
K excretion
Inc K excretion
 Metabolic  Metabolic Aldosterone mechanism, matagal yan,
alkalosis Acidosis
remember? Kaya sa chronic pa sya
 Respiratory  Inc Plasma magkakaron ng Inc K excretion
alkalosis Osm Then again, in acute cases, Aldosterone
 Cell lysis has no change in K excretion
 Inc Exercise
Respiratory Acidosis: NO EFFECT

REGULATION OF K REABSORPTION
AND SECRETION:
 (+) Hyperkalemia Inc K secretion
and this will also Inc Aldosterone and
will stimulates Principal cells to
Secrete K in DCT and CD || But

Page 24 of 89 By: Ayesha Bea Federizo Batch 2017


GRAND PHYSIOLOGY B FINALS REVIEWER

CALCIUM  If protein bound, not filtered pressure)  IncH2O reabsorption, then via solvent drag,
 Only Ionized is filtered solutes come along
 Low excretion rate  Balance nga e, so nangyayare to para my state ng balance,
 PTH & Vit D Inc Ca reabsorption para maski Inc GFR mag Iinc din agad reabsorption rate
 Calcitonin Dec Ca reabsorption para naman hindi lahat e iihi mo na
 are reabsorbed through Claudin 16 in
ascending limb TUBULOGLOMERULAR FEEDBACK
PHOSPHATE  Acts as buffers (Owmygad bakit may  Renal blood flow and GFR are kept constant for as long
buffers nanaman, BIOCHEM???) arterial pressure range is within autoregulatory range.
 PTH and Calcitonin Dec PO4
reabsorption  What Specific/Immediate Segments are Impermeable to
 Vit D Inc PO4 reabsorption water or has no ADH pakebells? O bakit 2 lang? 3 yan!
CHLORIDE  Associated with Na and H2O  Inc Osmotic gradient Solute loss is? Urine conc is?
handling, diba nga passively  Claudin 16 transports?
reabsorbed lang sya along with Na  Aldosterone effect in K secretion is?
and H2O
MAGNESIUM  If protein bound, not filtered
 All filtered are reabsorbed through RENAL PHYSIOLOGY IV
Claudin 16 in ascending limb
SULFATES BODY FLUIDS AND ACID-BASE BALANCE:

 Actively transported
 PS wala kasing list ng kung ano un
passively and actively
transported/reabsorbed, pero basta 3
lang yung Passively reabsorbed: Urea
Chloride at water.
AMMONIA  Acid base balance
HYDROGEN  ALL OF FILTERED HCO3 is
BICARBONATE REABSORBED
 (+) Alkalosis counteracted by Dec
in HCO3 reabsorption  Whatever goes In, must go out 
 (+) Acidosis  counteracted by Inc in
 Normally Intake=Output
HCO3 reabsorption
 The lining epithelium of the skin prevents rapid
GLUCOSE evaporation of water, kaya in cases of burns Inc Fluid
 loss  Tx= Fluid administration
 Glucose conc> renal threshold=
 Sweat loss varies in temperature and physical activity
Glucosuria
 Glucose conc< Tm = Glucosuria  Fecal water loss Inc in cases of diarrhea
 Di naman kasi parepareho ng # of
carriers ang nephron Body = 60% Water (42L)
AMINO ACIDS 2 Main compartments:
  ICF 40% (28L) (K, PO4, CHON)
VITAMIN C  Filtered, Reabsorbed and Excreted  ECF 20% (14L) (Na, Cl, HCO3)
 Inc Adrenal steroids and/or Inc  Interstitialfluid 15% (11L)
Filtered load of Na  Inc Vit C  Plasma 5% (3L)
secretion  Transcellular fluid  CSF, peritoneal fluid etc
 Tm=2 around 2L
UREA  Filtered and reabsorbed (remember Body = 7% Blood (5L)
narerecycle lang yan)  Plasma 60%
 ADH dependent  Formed Elements 40%
URIC ACID  Actively Secreted
 Tm=15 INDICATOR DILUTION PRINCIPLE
CREATINE  Filtered and reabsorbed
 No Tm
 Essential source of energy of muscle
CREATININE  Byproduct of creatine
 Secreted actively
 Tm 16
 Allows one to measure the compartments in the body
 3 criteria forindicator:
GLOMERULOTUBULAR BALANCE  Disperses evenly
 Filtered load = GFR x Na in filtrate  Stays in the compartment
 Inc GFR  Inc filtered load Inc reabsorption of Na  Not metabolized
and H2O .Why?  Eg of Indicators
 Inc GFR  Inc filtered load Inc Filtration Fraction  Tritium/Deuterium – heavy water  will go to all
(plasma protein concentration that imparts oncotic compartments for total body water measurement

Page 25 of 89 By: Ayesha Bea Federizo Batch 2017


GRAND PHYSIOLOGY B FINALS REVIEWER

 Inulin & Na Isotope ECF measurement  MASKI AKO DI KO GETS TO. AT AYOKO ALAMIN.
 Radiolabeled albumin  Plasma measurement NAKAKAINIS BAKIT MAY MATH! GRRR

OSMOTIC PRESSURE
 ICF- 5423; Interstitum-5423; Plasma-5443
 Plasma has higher osmotic Pressure as compared to
ICF and Interstitium
OSMOTIC EQUILIBRIA BETWEEN ECF & ICF
 Plasma and interstitial fluid volume; Determined by:  Isosmotic - Same osmotic pressure or same osmotic
 Hydrostatic pressure potential with plasma, not about concentration. Same
 Colloid osmotic pressure ability to attract water
 Extracellular and intracellular fluid volume; Determined by:  Isotonic - Same solute concentration
 Osmotic effect of solutes
 Intracellular fluid is isotonic with the extracellular fluid HYPOTONIC ISOTONIC HYPERTONIC
0.3 NaCl NSS/ 0.9 NaCl D5NSS,D5LR
OSMOLES:
0.45 NaCL PLR D10W,D50-50
 Total number of osmotically active particles in a D5 in 0.45 NaCl
solution
 D5W - Isoosmotic. take into consideration that Glucose
 1 osm = 1 mol = 6.02x1023 particles
enters the cell, therefore, when it does, it behaves as a
 1 mOsm = 0.001 osm hypotonic solution. So Isoosmotic talaga sya, hindi isotonic.
OSMOLALITY:
 High Osmotic Potential = Hypertonic Solution
 Osm/kg of H2O
 Low Osmotic Potential = Hypotonic Solution
 More accurate
ADDING SOLUTIONS IN ECF
OSMOLARITY:
 Hypotonic= Tonicity decreases  Water enters the cell
 Osm/L of H2O
 Affected by temp  ICF increases and ECF Increases too
 Hypertonic= Tonicity Increases Cell shrinks  ICF
OSMOTIC PRESSURE: decreases and ECF Increases
 The pressure required to oppose osmosis  Isotonic=Tonicity is the same  ECF Increases, ICF the
 Indirectly measures H2O and solute concentrations same
 parang ability to make the water stay ganun
 Inc osmotic Pressure  Inc solute conc GLUCOSE AND AMINO ACIDS:
 Inc osmotically active molecules  Inc Osmotic  Ineffective osmoles because they enter the cell
pressure  Adjusted to isotonicity; admin slowly
 Size doesn’t matter. Number of ion does!!!
 In case of shock  Give Isotonic solutions  Inc ECF
compartment  Inc circulating blood volume
VAN’T HOFF’S LAW  Pag D5W  Imemetabolize lang ng cell yung glucose 
Ineffective for shock
HYPONATREMIA HYPERNATREMIA
 Dec NaCl due to  Inc NaCl
hypoosmotic dehyreation  Dec H2O
caused by
 Electrolyte, H2O loss  Hyperosmotic dehydration
 Used in measuring osmotic pressure  Diarrhea, vomiting caused by:
 But, take into account the Osmotic coefficient or the  Diuretics  DI
correction factor, remember Na is a cation Cl is an anion,  Addison’s Disease  Excessive sweating
they are attracted to each other (aww sweet), therefore,
not all may dissociate. (di daw lahat maghihiwalay, kaya  Inc H2O due to hypoosmotic  Hyperosmotic
asa ka pa) overhydration caused by overhydration caused by:
 For NaCl, there is a 0.93 Correction factor  only 93%  Conn’s syndrome
dissociated (93% chance daw para maghiwalay! Mygad!)  SIADH (Dilutional  Hyperaldosteronism
Hyponatremia)

EDEMA
 Swelling caused by fluid retention

A. INTRACELLULAR EDEMA
 Dec metabolism
 Dec nutrition Dec ion pump activity
- Na-K pump is inhibited  Na stays inside 
water follows
 Inc osmosis- Increase ability to attract water

Page 26 of 89 By: Ayesha Bea Federizo Batch 2017


GRAND PHYSIOLOGY B FINALS REVIEWER

 Inflammation- Increase membrane permeability


II. INCREASED LYMPH FLOW 10-50x
 CVA/Stroke  Dec blood supply going to brain   Increased Filtered fluid Inc interstitial hydrostatic
NaK pump is inhibited  Na stays inside  water pressure (edi mapupush yung fluid papalabas ng
follows  Brain swells  Inc ICP interstitium) + Increase lymph flow ( inc yung
drainage ng fluids papuntang plasma) No edema
B. EXTRACELLULAR EDEMA
 Inc fluid in the interstitial space due to III. WASHDOWN OF INTERSTITIAL FLUID CHON
 Plasma leakage- tagas, punta un fluid sa  Lymphatics are permeable to CHON  sasama yung
interstitium interstititium CHON sa lymph flow  babalik ng
 Failure of the lymphatic drainage- di nya plasma = no edema
mababalik un excess fluid pabalik ng systemic
circ Low compliance of the interstitium 3mmHg
Increased lymph flow 10-50x 7mmHg
Washdown of interstitial fluid CHON 7mmHg
Total 17mmHg
For Edema to occur Capillary Hydrostatic Pressure should
exceed 17mmHg

TRANSCELLULAR COMPARTMENT

 Specialized compartments in the body


STARLING FORCES:
 Capillary Hydrostatic Pressure - Fluid na napupush
paputang Interstitium
 Capillary Oncotic Pressure - Andyan kasi un plasma
proteins diba, kaya pag meron kang hypoalbuminemia,
walang magi impart ng water pulling effect pabalik ng
plasma
 Hindi kasama si interstitial oncotic pressure kasi normally
walang protein sa interstitium

SAFETY FACTORS THAT PREVENT EDEMA


I. LOW COMPLIANCE OF THE INTERSTITIUM:  Fluids  to prevent friction during movement
A. Negative Pressure  Fluid exchange bet capillaries and potential spaces
 -3mmHg, so may vacuum effect or sucking effect  Eg synovial, pericardial, peritoneal, pleural fluid and etc
 12L of fluid, pag nag exceed pa dyan, mag sspill
 Drained by lymphatics
out lang pabalik ng plasma kasi nga may vacuum
effect
 C = V/P ACID BASE BALANCE
 Pag sinabi kasing Increase compliance, kahit
madami kang ilagay na fluid, yung pressure mo
konti lang yung change para mas maccomodate
nya un fluid. Pero, sa Interstitium mo, pag
dinagdagan mo pa ng fluid yan, mag increase
agad ng mataas yung Pressure mo. Pag tumaas
yung pressure mo ilalabas nya din lang agad
yung excess fluid. Kaya maganda pa gang
compliance mo is low.
B. Interstitial Gel HYDROGEN ION:
 Para syang barrier so prevented ang flow ng  Determines body pH ( Normal pH= 7.35-7.45)
water; plus nagiimpart sya ng elasticity and
compact brush pile.  Urine pH is affected by diet or intake, it can be alkaline in
C. Proteoglycan and Collagen Filaments pH but is usually acidic.
 Spacers; parang tulay, pero modulated ang flow
ng water, more on metabolites at nutrients yung CONTROLLING H
pinapadaan nya.  Buffers - Substances that can reversibly bind to H; pang
maintain ng pH parang ganun!
 Pag ang pressure mo nag 0 or higher  No more vacuum
effect magi increase ang compliance mo, so maski Chemical Respiratory Renal
madaming malagay na water, maliit lang yung change sa Buffer system System
pressure, so hindi maibabalik un water sa plasma  Onset Instantaneous 3-15mins Hours-days
Edema Handles Acids and Volatile Non Volatile
 Brush piles pulled apart, dec ang gel formation increase bases acids (CO2; acids and
spacers  Edema H2CO3) ECF bases

Page 27 of 89 By: Ayesha Bea Federizo Batch 2017


GRAND PHYSIOLOGY B FINALS REVIEWER

Power 1 1-2x greater Most  To reabsorb bicarbonate, H+ is needed. To generate a new


important bicarbonate, H+ will combine to non-bicarbonate buffers.

A. Chemical Buffers P.S. Ayoko ng buffers. Ayoko ng kahit anong bahid ng


 Intracellular CHON biochem. Physio na nga may biochem pa, dibaaa! Pero wag
 Hgb- remember the hgb dissociation curve. Pag nayan! Last shifting naman din naman tinanong ni doc kung
Inc H/Dec pH release oxygen anong blab la bla ang mangyayari sa blab la bla diba? CHOOSE
 Pag dec ang H/Inc pH mas rigid ang Hgb so mas YOUR BATTLES ika nga.
maghohold sya ng oxygen
 Extracellular Buffers QUANTIFYING ACID-BASE EXCRETION
 HCO3 buffer system- MOST important ECF buffer;  Bicarbonate Excretion
 Urine flow rate X concentration of bicarbonate in the
 Henderson-Hasselbach equation- computes for blood urine
pH given a conc of HCO3 and CO2  Measures the removal of bicarbonate from the blood
 PO4 buffer system – not that important ECF or the addition of H+ in the blood
buffer but, it plays a major role in Buffering renal  Ammonium Excretion
tubular fluid (DCT, CT Intercalated cell) and ICF  Urine flow rate X concentration of ammonia in the
(yes ICF din sabi ni Guyton); Bicarbonate buffer urine
system is reabsorbed while the phosphate buffer  Measures the amount of new bicarbonate added into
is not reabsorbed so it bears the burden of the blood
controlling pH within the tubules.  Titratable Acids
 Non-bicarbonate and non-ammonium buffer excreted
 Isohydric principle- Any condition that changes the like the phosphate, citrate, urate buffers
balance of one system also changes the balance of  Measured via titration via NaOH. (amount of H+
the others, meaning pa gang H mo nag iba ng present)
concentration, lahat ng system mo mag aadjust

B. Respiratory Buffers
 Control CO2 and carbonic acid
 Hypoventillate- more CO2 stays, respi acidosis
REGULATORY MECHANISM
 Hyperventillate- more CO2 goes out, respi alkalosis
 PRO ACID: HCO4 (Othocarbonic acid) , H, NH4, Titratable
acid
C. Renal Buffers
 PRO ALKA: HCO3
 Excretes acidic or basic urine
 Acid-base balance
ALKALOSIS ACIDOSIS
 Secretion of H ions
HCO3>H / Excess HCO3 H>HCO3 / Excess H
 Reabsorption of HCO3
WHAT TO DO? WHAT TO DO?
 Generation of new HCO3
 Excrete HCO3– Sobra pala  Excrete H – Sobra pala e!
Secretion of H Ions: e! Edi ilabas mo!  Edi ilabas mo!  Acidic
 Via Na-H counter transport Alkaline Urine Urine
 In DCT, CT Intercalated cell  Dec H Secretion- Need mo  Reabsorb HCO3- Dali para
 To reabsorb HCO3, H is secreted yan e! bumalik sa systemic circ!
 NH4 & Titratable acid is  Generate new HCO3 –
 If walang H, Na HCO3 goes out in urine
not excreted – Wag mong Bicarbonate paaa!
 DCT, CT Intercalated cell: Aldosterone Effect
tanggalin, need mo nga  Excrete Ammonia (NH4) –
 Hyperaldosteronism alkalosis; kasi H is
yan eee! Ilabas ang alak! Ay
spilled out
 No Generation of New ammonia pala!
 Stimuli for secretion of H ions :
HCO3 – sumosobra ka na  Mag Hyperventillate-
 Inc PCO2-respi acidosis hah! Ilabas ang feelings, uh I
 Dec extracellular pH
 Hypoventillate-kimkimin mean acid (CO2)!
 Excess aldosterone ang sama ng loob (CO2)
 Inc loss of H, NH4, titratable acids
 Inc HCO3 ABG INTERPRETATION
BABALA: GANITO KO SYA NAALALA HAH, PWEDENG MALI
Reabsorption of HCO3: AKO, PWEDE DING TAMA.
 85% PCT; 10% Thick Asc Limb; 4.9% DCT&CD
Normal Value Alka Acid
Generation of New HCO3:
Blood pH 7.35-7.45 >7.45 <7.35
 Hydrogen Ion Secretion (e makakagawa ka daw
e!) PCO2 (RESPI) 35-45 mmHg <35 >45
 Phosphate buffer HCO3 (METAB) 22-26 >26 <22
 Ammonia, Urate, citrate buffer
 Glutamine metabolism Step 1: Blood pH  Check if Acidic o Alkaline

Page 28 of 89 By: Ayesha Bea Federizo Batch 2017


GRAND PHYSIOLOGY B FINALS REVIEWER

Step 2: Pick a partner bet PCO2 or HCO3; dapat pareho sila


hah! Kung Sino ang partner or kung sino mas malala, sya ang
MAIN problem, yung mga kabit, pang compensate lang.
Step 3: Idamay ang kabit!

Eg#1

Step 1: Blood pH is Acidic


Step 2: HCO3 is Acidic; PCO2 is Acidic, Sino mas malala? HCO3!
So Metabolic Acidosis  Tx: Treat the cause or the primary problem
Step 3: Anyare kay PCO2? Ganito, since nagaway si Blood pH at  For Alkalosis Ammonium chloride, lysine
HCO3, affected si PCO2, e CO2 is acid diba. monohydrochloride (more common)
Ans= Metabolic Acidosis with Respi component  For Acidosis  Safest are Sodium lactate and Sodium
gluconate, pero pwede din Sodium Bicarbonate
Eg#2

MICTURITION
 Conscious and unconscious action
 Micturition reflex  autonomic spinal cord reflex, can be
inhibited or facilitated by centers in the cerebral cortex or
brain stem
 Pelvic nerves via Sacral Plexus (S2 S3 as Center) 
 Yung sa compensatory ek ek, nung nursing days kasi
Principal nerve supply of bladder Both sensory nerve fiber
namin, ganito
and motor nerve fiber;
Compensation pH HCO3 PCO2
 Sensory fibers- detect stretch in bladder wall; Stretch
Fully NORMAL Abn Abn signals in the posterior urethra initiates reflexes that
Compensated causes bladder emptying
Partially Abn Abn Abn  Motor fibers- Parasympathetic fibers that innervates
Compensated the detrusor muscle
Uncompensated Abn Abn Normal  Skeletal muscle fibers transmitted through Pudendal
Uncompensated Abn Normal Abn nerves to the external bladder sphincter – Somatic nerve
fibers that can voluntary control the ext sphincter
How I Understand It:  Hypogastric nerves (L2) – For fullness sensation and pain
 Pag fully compensated ata, yung parang example sa taas,  Urinary bladder (+) Stretch receptors  when stimulated
yung may Chronic Respi acidosis; Chronic na kasi e, so si and stretched  Sends impulse to Afferent nerve: Pelvic
blood pH mo, nagnormalize na, din a sya nag adjust nerves S2 S3 ( Sensory) Motor fibers S2 S3 
 Pag Partial naman yung sa Metabolic Acidosis with Respi contraction of detrusor muscle and relaxation of internal
component; component meaning, dumagdag lang, urethral sphincter signals pass through pudendal nerves
nagcomplicate lang. to the external sphincter to inhibit it; Impulse will be sent
 Pag Uncompensated naman, sa Acute cases, kunwari, to pons and cerebral cortex to see if voiding is convenient,
Acute Metabolic Acidosis. Acute kasi diba, bago lang, wala if it is  (+) voluntary contraction of abdominal muscle 
pang complication plus nagaadjust pa yung blood pH. Inc Pressure in bladder  extra urine enters bladder neck
 Inuulit ko. Pwedeng mali ako. Pwede ding tama. Wag nyo and posterior urethra  Stimulates sretch receptors
ko awayin  micturition reflex is excited and external urethral sphincter
is inhibited  urination
ANION GAP
 Cation # = Anion #  Samplex: If you transect L3  Diba si L2 mo ang
responsible for sympathetic innervations mo, for fullness
 (+) Metabolic Acidosis  HCO3 Decreases, Na & Cl stays
and pain sensation, o edi di na magfflow yung impulse
the same  Anion gap increases; In cases when anion gap
is normal, check for other unmeasured anions (PO4, SO4 dyan,  Puno na yung bladder mo di mo pa alam 
etc) INCONTINENCE (involuntary urination)

-oh, wag ka na magrerenal failure ulit ha 


Kaya mo yan! Ikaw pa 

Page 29 of 89 By: Ayesha Bea Federizo Batch 2017


GRAND PHYSIOLOGY B FINALS REVIEWER

Sex Tests
SEX DETERMINATION AND DIFFERENTIATION 1. Sex Chromatin Test
 Test that detects the presence of Barr bodies
 First discovered by Barr and Bertram the presence of sex
chromosome body AKA Barr Body in female somatic
cells
 Individual has 1 X in excess of 1, so 2 X meron sya  (+)
Barr Body  Positive Sex Chromatin  Genetic Female
 3 Specimens used
Buccal Blood Vaginal

Scrape from the inner


cheek
Barr Body Appears as Barr Body Appears
darkly-staining like a drumstick in
bodies attached to human female
Oogenesis and Spermatogenesis nuclear membrane Leukocyte, in its
nucleus
Oogenesis Spermatogenesis (+) Barr Body in 20 (+) Barr Body in 1-5
Autosomal 44 44 out of 100 cells  out of 300
Chromosomes Positive Female Neutrophils
Sex XX XY Positive=Female
Chromosomes (+) Barr Body in 0-4
After Several Mitotic divisions and 2 Meiotic divisions... cells out of 100 
Negative= Male
Autosomal 22 22 Possible to have false Frequency of HIGHEST POSITIVITY
Chromosomes positive results bec Distribution of PMN bec of the direct
Sex X X or Y there are many Drumstick: effects of estrogen in
Chromosomes chromatin materials > 1-5/300 the epithelium
in nucleus > 2-3/1000
>3. 6/500
Establishment of Genetic Sex at Fertilization
Summary of Chromosomal Abnormalities in Man
Genetic Sex/Gender/Sex
 Established at fertilization
 YOU CAN ONLY HAVE ONE OF TWO SEXES, NOTHING
IN BETWEEN (Yes, wala daw mga Bi )
Genetic Female = XX
 Fertilization= Sperm cell carrying X chromosome fertilizes
ovum (carries X, ALWAYS)  22 pairs of autosomal
chromosome and XX sex chromosome
 Non-fused labia with small clitoris
 Urogenital Sinus is open
Genetic Male = XY
 Fertilization= Sperm cell carrying Y chromosome fertilizes  Remember mag aappear lang ang Barr bodies pag 2 ang X sex
ovum (carries X, ALWAYS)  22 pairs of autosomal chromosome nya eg. Normal Genetic female, Klinefelter’s ,
chromosome and XY sex chromosome Female Mongoloid and Superfemale
 Fused scrotal sac with penis or phallus
Super Female
 Urogenital sinus is closed
 44 autosomal chromosome and 3 Sex chromosomes:
 External Genetalia XXX= 47 in total
 Basis for determining sex at birth Klinefelter’s
 Problem arises when Genitalia is Ambiguous (hard to  44 autosomal chromosomes and 3 Sex chromosomes:
determine) XXY=47 in total
 It’s a male, since there is Y but then again, Sex
Chromatin is positive or (+) Barr bodies bec of 2 X sex
chromosome
Turner’s syndrome
 44 autosomal chromosome and 1 Sex chromosome: X0=
Phallus is too big to be considered as clitoris 45 in total
Labioscrotal swelling looks like labia majora  Negative Sex Chromatin test since 1 lang X
There is incomplete fusion of urogenital sinus
 In cases of Ambiguity Defer signing of Birth cert  Mongoloids/ (+) Down Syndrome/ Trisomy 21
Determine genetic sex, do Sex Test  NO PROBLEM IN SEX CHROMOSOME, problem is in
Autosomal chromosomes

Page 30 of 89 By: Ayesha Bea Federizo Batch 2017


GRAND PHYSIOLOGY B FINALS REVIEWER

 CalvinKlein Kleinfelters Develops in male  Karyotypes:


Tina Turner Turner’s syndrome Develops in female

 Remember: when a sperm or ovum matures, meaning they have


nd
undergone 2 meiotic division only 1 sex chromosome shall
remain, either X or Y in male and only X in female.

Normally
 Sperm cell carries 1 X sex chromosome and it fertilizes
an ovum with of course, as always, carrying 1 X sex SEX DIFFERENTIATION
chromosome  Normal Genetic Female 1. Differentiate Primary Gonads
 Sperm cell carries 1 Y sex chromosome and it fertilizes 2. Differentiate Genital ducts that will differentiate into
an ovum with of course, as always, carrying 1 X sex internal genitalia
chromosome  Normal Genetic Male 3. Differentiate External genitalia structures

Nondisjunction Primary Gonads


 Failure of paired chromosomes or sister chromatids to  Primary organ of reproduction
separate after 2nd meiotic division; di naghiwalay!   Differentiation is Genetically determined
results to 2 sex chromosomes in 1 mature cell and none  Males: Testes ; Female: Ovaries
on the other  Common anlage in M&F  Bilateral Genital Ridge:
 Male: 1 sperm cell has 2 Sex chromosome, other th
 indifferent from fertilization up to 7 week AOG
sperm cell has 0  2 parts: Inner medulla & outer cortex
 Female: 1 ovum has 2 Sex chromosome, other Testes dev’t
ovum has 0  (+) Y  (+) HY antigen, a membrane protein= Testes
Eg: (How to create a) determining product  medulla develops, cortex
1. Superfemale regresses
 Sperm cell with X sex chromosome fertilizes an ovum with XX
 7th-8th week || Tesdulla (testes medulla)
sex chromosome =XXX
2. Turner’s Syndrome Ovary dev’t
 Sperm cell with X sex chromosome fertilizes an ovum with 0  (-) Y (-) HY antigen  cortex develops, medulla
sex chromosome =X0 regresses ; XX sex chromosome are required
 Spermcell with 0 sex chromosome + ovum with X chromosome
(as always) =X0  8th-10th week || Corvary (Ovary Cortex)
3. Klinefelter’s Syndrome
 Sperm cell with 1 Y sex chromosome fertilizes an ovum with XX  Turner’s XO= No Y , only 1 X  neither testes or ovaries
sex chromosome =XXY develops Gonads are long pale streak of connective
 Sperm cell with XY sex chromosome+ ovum with X sex
chromosome=XXY tissue
4. Sperm cell with 0 sex chromosome + Ovum with XX sex  Hormonal treatment of mother during pregnancy has no
chromosome  Normal Genetic Female effect on gonadal differentiation
5. Sperm cell with XY chromosome + Ovum with XX
chromosome=XXXY considered as male bec of Y
6. Sperm cell with XY sex chromosome + Ovum with 0 sex Genital Ducts
chromosome  Normal Genetic Male  Indifferent during early stage of embryonic life
 Sperm cell carrying Y sex chromosome fertilizes ovum with 0 sex  Male: Wolffian duct ; Female: Mullerian duct
chromosome or Sperm cell and ovum with no sex chromosome 
Not viable Male differentiation
2. Hair Root Test  Mullerian duct regresses, wolffian duct develops, giving
 Pluck 3-4 strands of normal scalp hair  tease hairroots rise to:
then stain with fluorescent dye (Quinachrine diHCl)
Internal Genitalia:
microscope: see that Y body? That is the part of the Y Epididymis Seminal Vesicles
sex chromosome  look for the F or Fluorescent body Vas Deferens Ejaculatory duct
within it, if (+)  Genetic male Female differentiation
3. Karyotyping or Chromosomal Analysis  Wolffian ducts disappears, mullerian duct develops
 Most accurate and Most expensive giving rise to:
 Tissue cultures are made and autosomal & sex Internal Genitalia: Fallopian tubes
chromosomes are paired; usually done by scientists who Uterus
specializes in genetics

Page 31 of 89 By: Ayesha Bea Federizo Batch 2017


GRAND PHYSIOLOGY B FINALS REVIEWER

Presence and Absence of functional testes Components of Sexual Identity


Present Absent Component Female Male
Males: XY  (+) Y sex Females or in males with Chromosomal Sex/ XX XY
chromosome  (+) HY antigen nonfunctional testes mullerian Genetic Sex
testes develops if testes duct develops, wolffian duct Nuclear Sex/Test Positive Negative
functions normally wolffian regresses (+) Barr Bodies
duct develops into male internal Gonadal Sex Ovary Testes
Hormonal Sex Mainly Estrogen Mainly Androgen
genitalia
Internal Genitalia Mullerian Duct Wolffian Duct
 mere presence of testes does not guarantee that it will
External Genitalia Lack of Androgen DOS  Penis & Scrotum
develop to be the wolffian ducts, TESTES SHOULD BE Labia Majora & Minora,
FUNCTIONING NORMALLY. Clitoris, Vesibule
Development of Genital ducts
MALE Sex Chromosomes Conditions in Ambiguous Ext genitalia
 Function of embryonic testes:  (+)Ambiguous External genitalia, Possible chromosomal make up:
Spermatogenesis o Normal Female:44XX=46 in total
Secretes: o Normal Male: 44XY=46 in total
o Mullerian Regression factor (MRF)- inhibits
o Abnormal sex Chromosome
mullerian duct dev't  no female internal genitalia
(Unilateral action) ; for dev't of vas deferens and
related structures
o Duct Organizing Substance (DOS) & Testosterone
- stimulate dev't of wolffian duct ; for dev't of vas
deferens and related structures
 No testes or non-functional testes there will be no
secretory products, therefore:
 nothing will inhibit the dev't of mullerian ducts 
genital ducts have the tendency to feminize
 wolffian ducts will not be developed  no male
internal genitalia
FEMALE
 XX  NO Y  NO HY antigenno testes no MRF, DOS & TRUE HERMAPHRODITE
testosterone genital ducts feminizes  Causes: Nondisjunction and Mutations
 Gonads: (+) Ovarian and Testicular tissues: Ovotestis
 Ovaries are not needed for the dev't of female internal  (+) Male facial hair, 2/3 of patients have normal breast dev’t,
genitalia i.e fallopian tube and uterus abundant pubic hair
So if there is no Gonad or Gonad is only rudimentary ,  (+) Uterus may or may not menstruate; not possible to get
irregardless of chromosomal sex only uterus and fallopian preggy on their own
tube develops  70% Chromatin Positive will menstruate
 Turner's syndrome X0 No testes, No ovaries uterus Negative  will not menstruate
and fallopian tubes develops nonetheless bec without testes  External Genitalia- either male or female; often ambiguous
again, internal genitalia tends to feminize.  Reared as males more often (3 out of 4) bec of big phallus
Male Pseudohermaphrodite Female Pseudohermaphrodite
Development of External Genitalia  Sex Chromosomes: XY  Sex Chromosome:XX
 Gonads: Testes ONLY  Gonads: Ovaries ONLY
 Cause: Non-functional Testes  Int Genitalia: uterus and
or No receptors in target fallopian tubes
cells  Tendency of Int&Ext  Ambiguity only in Ext
Genitalia to feminize  genitalia: Big phallus, labia
Ambiguous Int&Ext Genitalia majora looks like scrotum,
 No facial and body hair, incomplete Urogenital sinus
sparse pubic hair, small closure
phallus, ample breast dev’t,  Short stature, with body and
body is of female shape facial hair, male like pubic
MALE
hair distribution, small
 If testes is functional  secretes also dihydrotestosterone along
breast, prominent muscles
with testosterone for masculinization of male external genitalia
 Without testosterone and dihydrotestosterone external  Cause: CAH and exogenous
genitalia tends to feminize too intake of androgens by
st
 Testosterone- also responsible for male secondary sexual pregnant Mom during 1
characteristics i.e facial, body and pubic hair growth, low pitch trimester
voice, more developed muscles (anabolic hormone function) CAH/ Congenital Adrenal Hyperplasia
FEMALE  Hyperplastic adrenal glands Inc androgen secretion
 XX  NO Y  No MRF, DOS, testosterone and Masculinization of external genitalia. Determined during Newborn
dihydrotestosterone  feminine external genitalia (also internal) screening test.
 Estrogen - main secretory product of ovary  for dev't of Turner’s Syndrome Characteristics:
secondary female characteristics i.e high pitch voice, more fat  Short stature, neck webbing, small breast, sparse pubic hair, small
deposition in breast buttocks and thighs phallus; No gonads, but with Infantile uterus, fallopian tube and
 SAD TRUTH: You can change the appearance of your female Ext Genitalia, bec there is no ovaries therefore no
external genitalia through surgery but you won't be able to estrogen so no maturation.
change your genetic make up  No estrogen and androgen  No negative feedback  Increased
FSH&LH

Page 32 of 89 By: Ayesha Bea Federizo Batch 2017


GRAND PHYSIOLOGY B FINALS REVIEWER

Male Reproductive System  Formed from Testosterone by Sertoli cells when stimulated
by FSH
 Essential for spermiogenesis
3 Major Subdivisions
Growth Hormone (GH) and most of other body hormones
1. Spermatogenesis  For controlling background metabolic functions of testes
2. Performance of male sexual act  Promotes early division of spermatogonia
3. Hormonal Regulation of Reproductive functions  If absent (pituitary dwarfs)  Spermatogenesis is deficient or
Spermatogenesis absent Infertility
 Occurs due to stimulation of Ant Pituitary
Gonadotropic homones beginning at Puberty (13 y/o) Maturation and Storage of sperm
 Occurs in all Seminiferous tubule  Maturation Happens in EPIDIDYMIS after 18-24 hours
 they develop capability of motility
 Sperm removed from S.tubule and early portions of epididymis
are NONMOTILE and CAN’T FERTILIZE ovum
 Several inhibitory proteins in epididymal fluid prevents final
motility until after ejaculation
 Storage can be in epididymis but mainly in VAS
DEFERENS, its fertility is maintained for at least 1
month, they are deeply suppressed and therefore
become nonmotile there.
 Excessive sexual activity and ejaculation storage is shortened
 Testes produces about 120M sperm/day

Maturation/Actual Maturation of Sperm


 Happens after ejaculation sperms become motile and
capable of fertilizing ovum
 This is different from the “maturation” happening in
epididymis, dun kasi di pa kaya mag fertilize ng ovum
 Before onset of puberty, sperm is absent. Up until puberty, Physiology of mature sperm
Spermatogonia only.  Normal Motile sperm capable of flagellated
movement @ 1-4 mm/min velocity
Steps of Spermatogenesis:  Activity of sperm is greatly enhanced in a neutral or
slightly alkaline pH (eg. Ejaculated semen)
 Inc temp Inc Metabolism Inc sperm activity but
dec sperm life span
 Strong acid medium (eg. Female genitalia) sperm
dies
 Lifespan in female genital tract when ejaculated is 1-2 days only

 Not the actual sperm that can fertilize egg


 Genetic sex depends on the sperm that fertilizes the egg
 Mitochondria synthesizes ATP for sperm motility
Hormones that stimulates Spermatogenesis
Luteinizing Hormone (LH) SEMEN
 Secreted by APG  ejaculated by male during sex
 Stimulates Leydig cell to secrete testosterone  composed of fluid from (and sperm )Vas deferens, Prostate gland
Testosterone and Seminal Vesicles and few from mucous glands i.e
 Secreted by Leydig cells located in interstitium of testis bulbourethral gland
st
 Essential for 1 stage of forming sperm (growth and division  pH of 7.5
of testicular germinal cells) Seminal Vesicles
Follicle Stimulating Hormone (FSH)  60%  BULK of the semen
 Secreted by APG  Mucoid semen (same with mucous glands)
 Stimulates Sertoli cell  Composed of products Last to be ejaculated
 Without FSH, Process of spermiogenesis does not occur i.e  Serves to wash sperm through ejaculatory duct and urethra
spermatidssperm.  Contains Cholesterol, Fructose, Citric acid, Fibrinogens and
Estrogen Prostaglandins

Page 33 of 89 By: Ayesha Bea Federizo Batch 2017


GRAND PHYSIOLOGY B FINALS REVIEWER

 Fibrinogens and also causes falling off of any sperm that had already begun
 for semen to be a coagulum binding
 Prostaglandins
 Makes female cervical mucus more receptive to Abnormal Spermatogenesis and Male Fertility
sperm movement
 For reverse peristaltic contraction in oviducts  Destruction of Seminiferous tubule
and uterus to move sperm toward ovaries o Bilateral orchitis resulting from mumps or STD’s 
Prostate Gland may result to Sterility
 30% o Inborn degenerated tubular epithelia may result
 Milky semen, slightly alkaline  enhances sperm motility and
fertility to Sterility
 Contains Calcium, Citrate Ion (for alkalinity), Phosphate Ion (for o Excessive temp of testes seminiferous tubular
alkalinity), Clotting enzyme, profribrinolysis cells degenerate spermatogenesis is prevented
 (+) clotting enzyme  catalyzes reaction forming Coagulum from
may cause temporary sterility
fibrinogens to hold sperm in uterine cervix dissolves because of
fibrinolysin liquefies sperm  sperm becomes highly motile  Scrotum acts as a controlled cooling mechanism (maintains 2 C
again below internal temp)
Vas deferens o Warm-down
 10% o Cold-up
 Secretes Acidic fluid  Cryptorchidism
 Sperm motility optimum at pH 6-6.5 o Undescended testes at or near the time of birth
 Seminal vesicle Fibrinogen  Coagulum o Testes are derived from genital ridges in abdomen 3
Prostate gland (+) clotting enzyme  for Fibrinogen to weeks or 1 month before birth, testes normally descends
form coagulum through inguinal canal into scrotum mediated by
Testosterone under HCG stimulation of testes.
Capacitation of Spermatozoa o Testes remains on abdomen abdomen has higher
 Enables sperm to penetrate ovum temperature than scrotum tubular epithelium
 Requires 1-10 hours degenerates may cause sterility
Changes that Occurs in capacitation  After 1 year and still no descent may cause tumor growth 
 Uterine and fallopian tubes wash away inhibitory factors do surgery
that suppressed sperm activity in male genital ducts
 Sperm Count, Morphology and Motility on fertility
 After Ejaculation sperm swim upward toward uterus 
Count
gradual loss of cholesterol in sperms membrane 
 Semen per ejaculation = 3.5ml
acrosome becomes weaker  enzymes from acrosome  Average sperm is 120M per ml of semen
will be released Zona Pellucida penetration  In “normal” males, it varies from 35M to 200M
 Cholesterol  Sperm per ejaculation = 400M
o Contained in seminal vesicle  Infertile if sperm is 20M and below
o Toughens acrosome and prevents enzyme release Morphology
 Abnormal: 2 heads, abnormal tails etc motility is affected
 Membrane becomes more permeable to Ca Ions   may cause sterility
flagellum becomes more powerful for whiplash motion Motility
 For reasons not known they are entirely or relatively not
Acrosome Reaction motile likely to be infertile
 Movement of sperm should be 1 direction only
 Release of enzyme and digestion of membrane of ovum
 Acrosome have abundant:
MALE SEXUAL ACT
o Hyaluronidase  depolymerises hyaluronic acid
 Results from inherent reflex mechanisms integrated in
polymers in intercellular cement that hold ovarian
the sacral and lumbar spinal cord initiated by either
granulose cells together
psychic stimulation from the brain or actual sexual
o Proteolytic enzymes  digest proteins in
stimulation from the sex organs or both.
structional elements of tissue cells that adheres to
Glans Penis
ovum
 most important source of sensory nerve signals for
FERTILIZATION initiating the male sexual act
Ovum is expelled sperm dissolutes the ovum’s granulose cell  very sensitive, have lots of nerve endings causes one to
layers acrosomes release enzyme to penetrate zona experience sexual sensation
pellucida anterior membrane of sperm binds with receptor
proteins in zona pellucida  entire acrosome dissolves all  Stimulation of the anal epithelium, the scrotum, and
acrosomal enzymes are further released  passageway is perinealstructures in general can send signals into the cord
formed for sperm head to go through zona pellucidacell that add to the sexual sensation, which also includes urethra,
membrane of sperm head and oocyte fuses= single cell with bladder,prostate, seminal vesicles, testes, and vas deferens.
equal number of chromosomes from motherfather.
 STD  improperly treated organism still infects the male
st genitalia causing tingling sensation  increases libido
 Only 1 sperm enters the oocyte bec when the 1 sperm
penetrates zona pellucida  calcium ions diffuse inward through
oocyte membrane multiple cortical granules will be released
by exocytosis thereby preventing binding of additional sperm

Page 34 of 89 By: Ayesha Bea Federizo Batch 2017


GRAND PHYSIOLOGY B FINALS REVIEWER

 High testosterone
Psychic element o  Negative feedback  Hypothalamus and APG inhibits
 Sexy thoughts, wet dreams during some stages of sexual life further release of GnRH and FSH LH respectively
(teenager) o FSH stimulate sertoli cells to secrete Inhibin  inhibit
hypothalamus and APG
STAGES OF MALE SEXUAL ACT
1. Penile Erection
 By Parasympathetic Nerves (sacral nerves pelvic nerves penis Effect of Testosterone on Development of Adult Primary
 first effect of male sexual stimulation, degree of erection is and Secondary Sexual Characteristics
proportional to the degree of stimulation, whether psychic or  Penis, scrotum, and testes enlarge about eightfold before
physical the age of 20 years
2. Lubrication  More prolific hair distribution
 By the Parasympathetic  Baldness Testosterone decreases the growth of hair on
 cause the urethral glands and the bulbourethral glands to secrete the top of the head
mucus
 Voice hypertrophy of the laryngeal mucosa and
 most of the lubrication is provided by the female sexual organs
 Without lubrication painfulmale sexual act is seldom
enlargement of the larynx  masculine voice
successful  Thickening of skin and development of acne
3. Emission & Ejaculation  Protein Formation and Muscle Development
 By the Sympathetic nerves  Increases Bone Matrix and Causes Calcium Retention 
 culmination of the male sexual act  Male Orgasm growth spurts
 Emission forerunner of ejaculation (up to formation of semen)  Inc BMR  Inc RBC production
 Increases the reabsorption of Na in the distal tubules of
CONTROL OF MALE SEXUAL FUNCTIONS BY HORMONES the kidneys
 Controlled by Anterior Pituitary Gonadotropins
 GnRH from Hypothalamus APG releases LH & FSH MALE CLIMACTERIC
o LH= stimulate Leydig cells Testosterone release  Decrease in male sexual function due to dec testosterone
o FSH= stimulate Sertoli cells Spermatogenesis production
 Symptoms:
TESTOSTERONE o Hot Flashes
o Suffocation
 Mainly Secreted by Leydig cells (numerous in the newborn
o Psychic disorders Male Menopause
male infant for the first few months of life and in the adult
male after puberty)
Abnormalities of Male Sexual Function
 Not present during childhood years
 Old Age (above 50y/o)  decrease in testosterone
production Prostate Gland
 For development of Male secondary characteristics from  Normally, small throughout childhood and begins to grow
puberty to maturity at puberty under the stimulus of testosterone. Stationary
 The Primary testicular hormone (along with size at age of 20-50
dihydrotestosterone, and androstenedione) Benign Prostatic Fibroadenoma Cancer of the Prostate
 Dihydrotestosterone  more active hormone  Older men  2-3% of male deaths
 Small amount secreted by Adrenals, that’s why women  Causes urinary obstruction  Testosterone increases CA
 Hypertrophy caused by cell growth
with adrenal tumor manifests with male secondary sex overgrowth of prostate  Treatment:
characteristics tissue itself, not by  Remove Testes  no
 For Masculinisation testosterone testosterone production
 CA cell growth is
o Fetal life: HCG from placenta stimulates testes  inhibited.
For Testicular descent  Estrogen decreases
o Puberty-Remainder of life: Testosterone under size of Prostate
stimulus of LH
HYPOGONADISM VS HYPERGONADISM
HYPOGONADISM HYPERGONADISM
 Causes:  Interstitial Leydig cell
 Nonfunctional testes tumors 100x increase in
 Castration testosterone production
 genetic inability of the  S/Sx:
hypothalamus to secrete  rapid growth of the
normal amounts of GnRH musculature and bones
 S/Sx: but also cause early
 Childlike voice uniting of the
 No Hairloss; Sparse epiphyses Small
hair distribution stature
 Infantile penis  Excessive dev’t of male
 Less muscular body sexual organs and male
 Decrease Libido secondary characteristics

Page 35 of 89 By: Ayesha Bea Federizo Batch 2017


GRAND PHYSIOLOGY B FINALS REVIEWER

Female Physiology GnRH


PART I  Produced in Arcuate nucleus in the medial basal
ANATOMY Hypothalamus
External Genitalia/Vulva  Stimulates synthesis and release of LH & FSH if secreted in
 Anything seen from Mons Pubis down to Perineal Body a pulsatile manner (every 1-2 hours)
 Vagina is a part of external Genitalia  If released in continuous manner No release of
Internal Genitalia FSH LH  Amenorrhea
 3 Major Structures: FSH & LH
o Uterus, Ovaries and Fallopian tubes/Oviducts  Produced by Basophilic cells of APG
 Same alpha units with TSH and HCG
GENETICS Hyperthyroidism
 Genetic Sex established upon FERTILIZATION,  Elevated T3 T4  APG decreases release of TSH, since they
determined by the Sperm (either X or Y sex chromosome) have similar alpha units and came from the same cell
BUT BUT BUT, it is still indifferent, meaning you will not be (basophilic cells)  FSH & LH release is also inhibited 
able to identify if it’s Male/Female just yet. amenorrhea
 6 Weeks of Gestation the sex can be distinguished as  Pregnancy test check for Beta HCG also as this may
female or male, first evidence is the Primordial germ cell cause a false positive result in PT
FSH
OVARIAN DEVELOPMENT  Acts on Granulosa cell facilitating their proliferation and
1. 3-6mos AOG Primordial germ cells undergo mitosis production of estrogen and FSH&LH receptors
producing now 7Million oogonia which contains the  Stimulate follicle growth
ovum called PRIMARY OOCYTE LH
st
2. After 6 mos AOG  Oogonium enters 1 Meiotic division  After ovulation, LH acts on the granulosa cells for the
BUT STOPS AT DIPLOTENE STAGE OF PROPHASE production of progesterone and,
3. Upon Birth DIPLOTENE STAGE OF PROPHASE of Meiosis  Theca cells for the production of androgens  Later
1 ; 2 Million Primordial follicles converted to estradiol through the 2 cell 2 gonadotropin
4. Puberty  First Meiotic Division will resume and is system.
completed upon OVULATION becoming now a Secondary  FSH  granulosa cells  Estrogen
Oocyte ( 22X) ; 400,000 Primary follicles LH  granulosa cells  Progesterone
5. Second Meiotic Division is completed upon Fertilization, Festrogen || Logesterone
That is, after Meiosis 1 ovum will go directly into  Synergistic for FSH  Follicular Maturation
Metaphase, bypassing now the prophase stage producing  Without LH  No ovulation  No fertilization 
now a haploid chromosome INFERTILITY
 WHOLE REPRODUCTIVE SPAN OF FEMALE (13-46 y/o) 
only 400 oocytes will be ovulated, the rest of the 7Million
undergoes Atresia MENSTRUAL CYCLE
MATURE GRAAFIAN FOLLICLE  Ave Duration: 28 days (+/- 7 days)
 Among thousands of follicle only 1 will mature to ovulate, o Polymenorrhea= <21 days
after which what remains is the Corpus luteum o Oligomenorrhea= >35 days
 After meiosis I (upon ovulation) a Secondary Oocyte is o Menorrhagia = abnormally heavy or long
formed menstrual flow
 Group of Granulosa cells surrounding the oocyte is the o Metrorrhagia= irregular bleeding in between
Cumulus Oophorus, this Granulosa cells produce menstrual period
ESTROGEN (and Progesterone) o Dysmenorrhoea= painful menstruation
 Theca cells outside produces ANDROGENS  2 cycles: (occurs at the same time)
o Ovarian Ovum is fertilized
FEMALE HORMONAL SYSTEM o Endometrial  Endometrium is prepared for
possible implantation of fertilized ovum
Regulation
 Depends on FSH LH  Pag wala  Atrophic Ovary
o GnRH from Hypothalamus:
 Occurs in ovary and endometrium from day 21-28 of cycle
Stimulatory and inhibitory
 Menstruation happens every 28 days
feedback effects of ovarian
 Children (source of estrogen is the adrenal cortex) 
steroids: estradiol &
APG is very sensitive to Estrogen, so iinhibit agad
progesterone via Long
nya si FSH LH  No menstruation || Puberty 
feedback loop
estrogen sensitivity of APG is lost  no more FSH LH
o GnRH inhibition via GnRH itself
inhibition  Menstruation
via ultrashort feedback
mechanism and via neural PUBERTY
input pathway  9-12 y/o Inc FSH & LH
o Gonadotropins from APG:  4 Stages:
inhibitory effect via Short o Thelarche  Breast Budding
Feedback loop o Adrenarche  pubic and axillary hair growth
o Accelerated growth spurt
o Menarche @ 11-15 y/o

Page 36 of 89 By: Ayesha Bea Federizo Batch 2017


GRAND PHYSIOLOGY B FINALS REVIEWER

2. LUTEAL PHASE – FIXED AT 14 DAYS


 Remaining 14 days
Corpus Luteum
 Formed after ovulation by LH; it’s granulose cells produce
more Progesterone than that of estrogen || Theca cells 
produces androgen (androstenedione) & Testosterone
 After Ovulation  LH converts granulose and theca cells
into  Lutein Cells  produces more progesterone, So
in the cycle after 14 days (during Luteal phase) Level of
progesterone is much higher than estrogen.
 LH decreases corpus luteum regresses  becomes
corpus albicans  Estrogen and Progesterone are now
only moderately secreted on 26th day or 12 days after
ovulation  Onset of Menstruation is signaled
Inhibin
 From ovary, inhibits FSH secretion
REVIEW
 Midluteal phase  High level of Progesterone and Estrogen 
OVARIAN CYCLE Negative Inhibition  Dec FSH LH
TWO PHASES (28 days):  Mid Follicular phase  Dec FSH, Inc Estrogen
1. FOLLICULAR PHASE – NOT PREDICTABLE  Corpus Luteum  produced by LH
st  Cause of LH surge  Estradiol peak
 1 14 days
 FSH  Growth || LH  Maturation
Birth= 2 Million primordial follicles
Ovum is with 1 layer of granulose cells for P.S
nourishment and oocyte maturation inhibiting factor Day 1 is the first day of your Menstruation. So yung
to keep it on diplotene stage of prophase @ meiosis “cycles” mo nagsstart upon menstruation.
1 ENDOMETRIAL CYCLE
Puberty= FSH LH Increases TWO PHASES (28 days):
Meiosis I is resumed and completed upon ovulation PROLIFERATIVE/ESTROGEN PHASE
giving rise to secondary oocyte || 400,000 primary  >Corresponds with Follicular phase
follicles  Begins at Day 1:
 From Primary to pre Antral  FSH is not needed / Non Estrogen  for progressive growth of endometrial glands
Gonadotropin dependent
(which secrete thin mucus into the cervical canal)
 Rapid proliferation of granulose cells and theca cell
formation  follicles become Pre Antral
 thickening of endometrium, continues up to day 14
Pregnancy (ovulation) even upon entering secretory phase
 Placenta releases high Estrogen and Progesterone  Day 5:
 APG is not releasing FSH and LH bec of Negative feedback Due to the thickening of endometrium  Covering up of
Mechanism blood vessels that are exposed  Menstruation ceases
Day 1 of Menstrual cycle SECRETORY PHASE
 Estrogen and Progesterone is low ; FSH is TOO HIGH   Day 14-28
recruitment for accelerated growth of 6-12 pre antral  Estrogen still promotes endometrial proliferation
follicle  only 1 dominates  1 follicle becomes Antral  Progesterone  Prepares endometrium for possible
follicle  rich in estrogen implantation by causing swelling and secretory dev’t in
 Vesicular follicle- a follicle that exhibits excessive growth endometrium and increasing its blood supply
due to FSH  too much estrogen  FSH receptors also No fertilization or implantation
increases  Increase FSH responsiveness  produce more  On 26th day( 2 days before menstruation)  corpus
ESTROGEN AND ESTROGEN ONLY  follicle becomes luteum regresses becomes corpus albican
larger ( Positive Feedback Mechanism) Progesterone Estrogen decreases no more support to
 FSH and Estrogen  increased in theca cells receptor endometrium causing vasospasm of blood vessels due to
allows LH to act for FINAL MATURATION of follicle.
prostaglandins  endometrium is sloughed off
 Vesicular follicle becomes the Dominant follicle the other
11 follicle undergoes atresia because the Increase in blood vessels are exposed again  Menstruation on day
estrogen secreted by the dominant follicle inhibits APG to 1
release FSH REVIEW
 ESTROGEN In Proliferative phase  Endometrial Proliferation
 Dominant Follicle  Ovulation proceeds  PROGESTERONE  for Secretory development and support 
for possible implantation of fertilized ovum
LH  Progesterone decline  signals onset of menstruation
 For Final follicular growth and ovulation
 LH Surge LH at its peak for 2 days before ovulation MENSTRUAL PERIOD
 10-12 hours after LH surge  Ovulation happens  40 ml of blood loss (whole duration)
 Causes of LH surge  Estradiol peak, When estrogen is  Contains fibrinolysin  so it doesn’t clot, if (+) clot  Heavy
secreted on highest level APG is stimulated even more to menstruation bec fibrinolysn is not enough, too much blood.
release LH at highest peak.  Ceases 4-7 days due to re epithilialization
 Too little estrogen is secreted  No Estradiol peak 
No LH surge  No ovulation  Infertility

Page 37 of 89 By: Ayesha Bea Federizo Batch 2017


GRAND PHYSIOLOGY B FINALS REVIEWER

OVARIAN STEROIDS magiging infantile sila, I think breast growth 


Cholesterol wrong choice of word lang. for breast feeding
 Parent compound of all sex hormones  Turner’s syndrome
 Rate limiting step= Cholesterol  Pregnenolone  no ovaries no  Promotes secretory
 2 pathway of Pregnenolone estrogen  changes in
 DHEA pathway  Androgen converted to infantile int & ext endometrium
Estrogen genitalia
 Progesterone pathway  Progesterone  Precocious
th
 During Follicular Phase DHEA (5 ) pathway is puberty   Dec freq and
favoured oversecretion of intensity of uterine
 During Luteal Phase when Corpus luteum is produced estrogen  contractions 
th
due to LH  Progesterone(4 ) pathway is favoured children with prevents
mature ext genitalia miscarriage
ESTROGEN  Dev’t and maturation
 Sources: of female secondary  for nutrition of
o Ovary characteristics travelling fertilized
o Adrenal cortex  Androgen  estrogen  Maintains normal ovum
o If pregnant major source  PLACENTA structure of skin and  Cervical mucus:
 Types: blood vessels Scanty, viscous
o Beta-estradiol – Major form from ovary  Accelerates growth and acidic “white
o Estrone –majority from peripheral coversion of and early closure of mens”
androgen; some from ovaries epiphysis of long  Signals entering
o Estriole – oxidative product of estradiol and bones ( nauuna in next cycle of
estrone tumangkad ang babae) menstruation
PROGESTERONE  Growth of ovarian  (+) itch and foul
 Most important among progestins follicle smell 
 Sources:  Cervical mucus Infection
o 2nd half of ovarian cycle ( Luteal Phase) production  clear,
o Placenta if preggy of course watery and slightly  Increases basal
o Adrenal Cortex alkaline (after mens) body temperature
 Greater Osteoblastic
activity  for bone
HORMONE PRODUCTION IN GRAAFIAN FOLLICLE
deposition
 Menopause no
THECA CELLS Androgens: Androstenedione estrogen  dec
Testosterone
osteoblastic act 
GRANULOSA CELLS Estrone osteoporosis prone
Estradiole  Protein synthesis in
During Follicular phase: liver
Estrogen  Blood coagulation,
CORPUS LUTEUM (Lutein Mainly Progesterone water retention
cells) Estrogen  Preggy  High
estrogen  Edema
TWO-CELL TWO-GONADOTROPIN HYPOTHESIS
Two cells Granulosa and Theca cells
Two Gonadotropin FSH LH PART II
THE FASTEST SPERM
Principle:  Millions of sperm is introduced into vagina upon
Cholesterol to Androgen conversion occurs only in THECA ejaculation  BUT ONLY 1 SPERM CAN FERTILIZE IT
cells because the LH dependent enzyme is only present in  Once sperm fertilizes ovum  Zona pellucida solidifies 
THECA cells. preventing other sperm from entering
TRANSPORT AND IMPLANTATION
Androgens (Androstenedione & Testosterone)  Brought  Ovum travels along fallopian tube for 3-4 days
inside granulosa cells  converted to estrogen via FSH Blastocyst
dependent enzyme aromatase. (Only in Granulose cells)  Mature zygote
 2 types of cell:
o Inner cell fetus
ESTROGEN & PROGESTERONE o Outer cell  trophoblast  placenta
 Metabolized in liver; mainly secreted via Urine  Cilia will propel the zygote toward uterus
ESTROGEN PROGESTERONE  DAY 7  trophoblast secrete proteolytic enzyme
 Dev’t of Female  Glandular and endometrium is digested and liquefied
reproductive organs. secretory dev’t of IMPLANTATION (with bleeding)
P.s reproductive organs  Implantation in endometrium of uterus  need to
ang alam ko magdedevelop pa
maintain corpus luteum for sustained progesterone
din ang female repro even
without estrogen pero secretion  preventing sloughing off of endometrium
 Lobuloalveolar

Page 38 of 89 By: Ayesha Bea Federizo Batch 2017


GRAND PHYSIOLOGY B FINALS REVIEWER

(will cause abortion if corpus luteum regresses <7 weeks HUMAN CHORIONIC SOMMATOMAMMOTROPIN (hCS)
AOG)  secreted by placenta
PLACENTA  an anti insulin diabetogenic hormone
1.) Diffusion of oxygen from the mother to the baby  increases level of maternal glucose to be given to fetus
 Fetus Does not have functional respiratory sytem  if not tolerated by mothergestational diabetes
depends on mother for Oxygen  For maternal lipolysis  Inc free Fatty acids  energy
Factors that enable the fetus to deliver enough oxygen to source for maternal metabolism
the tissue:  May play a role in fetal vasculature formation
 Fetal hgb carries 20-50% more oxygen than an adult hgb
 Hgb concentration of the fetal blood is 50% greater than OTHER HORMONES OF PREGNANCY
the mother
 Bohr effect : CORTICISTEROIDS
o Curve is shifted to the right = Oxygen Inc  Inc glucocorticoids inc mobilization of amino acids
o Maternal and fetal blood meets  baby donates for fetal growth
CO2 to mom  fetal CO2 level is dec  shifts  Inc aldosterone causes edema to mother
curve to left  oxygen affinity decreases  baby
gets oxygen. Double Bohr effect THYROID GLAND
4 parameters that will shift the  Inc cellular and vascularity of gland inc in size
curve to the right:  Inc in thyroxine production for fetal brain
 increase H+ or decrease pH  increase 2-3DPG development
 increase CO2  increase temperature
PARATHYROID GLAND
2.) Diffusion of carbon dioxide from the baby to the  Enlarged Maternal calcium mobilization Ca is
mother given to fetus fetal bone and teeth development
o Baby has higher CO2 levels, tendency is you  give Ca supplements to mom to prevent early
transfer CO2 to mother osteoporosis
3.) Diffusion of foodstuff from the mother to the baby
o mainly glucose MATERNAL ADAPTATION TO PREGNANCY
4.) Excretion of fetal waste product to the mother
WEIGHT GAIN Normally around 24lbs:
 Placenta is Connected to baby via Umbilical cord: 7lbs from fetus
o 3 vessels of Umbilical cord 6lbs from intravascular vol
 2 umbilicar artery – carries deoxygenated 4lbs from amniotic fluid, placenta and
blood fetal membranes
 1 umbilical vein – carries oxygenated blood 3lbs from fats
 Fetal blood does not mix with maternal blood, if it does
METABOLISM Inc maternal thyroid hormone
 may lead to Erythroblastosis fetalis
production Inc BMR by 15%
HORMONES OF PREGNANCY NUTRITION balanced diet
Iron supplements (1000 mg), calcium,
HCG
vitamins etc
 FIRST hormone secreted during pregnancy
Pregnant add 300 kcal to diet
 Secreted by the trophoblast cells of zygote
Lactating add 500 kcal to diet
 Maintains corpus luteum  (+) progesterone and
BLOOD VOLUME Estrogen and aldosterone fluid
estrogen  pregnancy maintenance
 Peaks at 8th week  declines after 8th week  Plateau retention Inc BV
starts at 18-20 weeks CARDIAC OUTPUT Increased blood volume Inc CO
 After 8 weeks  HCG declines  corpus luteum RENAL Inc blood volume Inc GFR
regresses (13-17th week)  Doesn’t matter because RESPIRATION Progesterone inc tidal volume
th
PLACENTA takes over during the 7 week  (+) Inc RR
progesterone and estrogen  pregnancy maintenance
PARTURITION/LABOR
ESTROGEN  process of giving birth
 Pregnant  for enlargement of uterus, palpated just  2 causes:
below xiphoid process A. Progressive hormonal changes
 for enlargement of stromal and ductal structures of the Estrogen Progesterone ratio Oxytocin on uterus
breast AOG <7 mo: More Synthesized in hypothalamus,
 for enlargement of female ext (and int) genitalia Progesterone stored and released by
 for relaxation of pelvic joints and ligaments AOG >7mo: More estrogen Posterior pituitary gland for
 Uterine Contractions titigas yung tyan strong uterine contraction
(also for milk letdown)
PROGESTERONE Fetal hormones
 Causes decidual cells to develop Fetal Pituitary gland Inc oxytocin uterine contraction
 decreases contractility of pregnant uterus Fetal adrenal gland Inc cortisol
 nourishes morula and blastocyst as it travels in fallopian Fetal Membranes Inc Prostaglandinuterine contraction
tube for 3-4 days
 for preparation for lactation

Page 39 of 89 By: Ayesha Bea Federizo Batch 2017


GRAND PHYSIOLOGY B FINALS REVIEWER

B. Progressive mechanical changes Mitosis starts when? (developing female 3-6mo AOG
Stretch of uterine musculature eagers contraction baby)
Cervical stretch and irritation prostaglandin release When is genetic sex established? Upon fertilization
uterine contraction Stage of oocyte during menstruation Metaphase 2 of
Meiosis 2
 upon reaching full term and post dated na advice
When is Meiosis 2 completed? Upon fertilization
mom to have sex sperm contains prostaglandin and
When is Meiosis 1 completed? Upon Ovulation
penis will irritate cervix which causes further
Stage of oocyte upon birth? Prophase:
prostaglandin release uterine contraction
Diplotene stage
STAGES OF LABOR What confers or determines sex of baby? The sperm
To be stimulatory, how do you release GnRH? Pulsatile, every 1-2
1st Onset of REGULAR uterine hours
contraction up to Full cervical Hormone of Ovulation? LH  LH surge
dilation (10cm) st
1 sign of puberty? Thelarche
2nd Full cervical dilation up to Last sign of puberty? Menarche
delivery of baby Ovulation happens when? 14 days before (or
3rd Delivery of baby up to after) onset of
**Very important, if not done delivery of placenta menstruation ||
10-12 hours after
massive bleeding
LH surge
 Uterine Involution 4-6 weeks after delivery, it goes What causes LH surge? Estradiol peak
back to non pregnant state Culprit of Infertility? LH
LACTATION What hormone signals onset of Mainly a decline in
 Estrogen stromal and ductal structures of the breast menstruation? progesterone (LH
and stimulus for prolactin release minor only)
 Progesterone lobuloalveolar growth What phase does LH surge happen? (late) follicular
breastfeeding When does Estradiol peak happen? (late) follicular
Effect of Estrogen in APG and Hypothalamus: Stimulatory
Very High
PROLACTIN
Effect of Estrogen in APG and Hypothalamus: Inhibitory
 released in APG stimulated by estrogen
Moderate-low
for milk production, starts at 5th week of Pregnancy
Main hormone of corpus luteum Progesterone
until delivery of baby
Day 1 of cycle, FSH level is HIGH ( low
Estrogen and
ESTROGEN & PROGESTERONE Progesterone)
 inhibitory to effect of prolactin in the BREAST so if not Transport of ovum along fallopian tube? 3-4 days
pregnant you will not lactate Who fertilizes ovum?
st
1 sperm to arrive
 Pregnant after delivery dec estrogen and Hormone maintaing pregnancy Progesterone
progesterone dec suppression of prolactin effects in Who can make the uterus contract? Estrogen
breast lactation after several Progesterone or Estrogen?
days Nipple stimulation causes release of: Oxytocin and
 after delivery dec estrogen no more stimulus for Prolactin
prolactin release in APG dec prolactin WHAT TO Hormone responsible for non ovulation when PIF/Dopamine
DOOO? stimulate nipples: breastfeeding?
o Release Oxytocin (milk letdown or milk Hormone for milk production Prolactin
expression and uterine contraction to For milk letdown? Oxytocin
prevent post partum bleeding) What hormone produces corpus luteum LH
o Release Prolactin for milk production Corpus luteum secretes Progesterone >
Estrogen
What phase varies? Luteal or Follicular? Follicular
 Family planning factor: Prolactin release PIF or
Produced by corpus luteum, inhibits APG Inhibin
dopamine is also released GnRH is inhibited no FSH
causing a dec in FSH & LH during luteal phase
and LH no menstruation and ovulation pregnancy
# of oocytes ovulated during entire repro 400
prevented for max of 6mos period
# of oocytes At birth? 2M
FAQ’s/MLAQ’s Puberty? 400,000
3-6 mos AOG? 7M
th th
Blastocyst implants in endometrium 5 -7 day post What causes proliferation of granulose cells? FSH
ovulation Stage of follicular growth wherein you don’t Primary stage
th
HCG peaks at? 8 week AOG need gonadotropins
Layer of uterus involved in menstruation? Endometrium
For contraction? Myometrium
Most common site of Ectopic pregnancy? Ampulla of
fallopian tube
Usual Site of Fertilization Ampulla of
fallopian tube
Who guides ovum towards Fallopian tube? Fimbriae
What are the stages of Mitosis? PMAT, Interphase
is not included

Page 40 of 89 By: Ayesha Bea Federizo Batch 2017


GRAND PHYSIOLOGY B FINALS REVIEWER

 Anti stress hormones: Cortisol, Epinephrine,


General Principles in Endocrinology Norepinephrine
Classification of Endocrine Glands
Purely Endocrine Partly Endocrine
Endocrine System Nervous System -only secretes hormones -secretes hormones and serve
 Regulate body functions and processes other functions
 Enable signals from one cell to be transmitted to Anterior Pituitary Gland Nerve Endings
another cell to respond (Adenohypophysis) Trophic  Ach, EP
Hormones:
 Releases  Releases  GH, PrL, TSH, ACTH,
Hormones Neurotransmitters FSH, LH
 Slower but more  Fast but localized  Master Gland
diffused Posterior Pituitary Gland Kidneys
(Neurohypophysis)  Erythropoietin, Vit. D
 Stores and releases
Types of Chemical Messenger System (Mediators) ADH & Oxytocin
Thyroid Gland GIT
 T3, T4, Calcitonin  Gastrin, CCK, Secretin
1. Neurotransmitter
Parathyroid Gland Placenta
 Synthesized in axon terminal, released due to action  PTH  HCG
potential, NTA released via exocytosis into synaptic Islets of Langerhans Heart
cleft facilitated by calcium Ions. Eg: Ach, Nep  Insulin, Glucagon  ANP
Adrenal Cortex
 MC, GC, Androgen Skin
2. Endocrine Hormones  Melanin
 Synthesized by Endocrine glands released into body
fluids i.e. circulating blood because target cell is Adrenal Medulla Liver
 EP, NEP  IGF, VIt. D
located away from secretory cell
Testes Pineal Gland
 Testosterone  Melatonin
 Both a NTA and Hormone: Norepinephrine. Ovaries Hypothalamus
Synthesized in post ganglionic nerve ending and  Estrogen, Progesterone  Releasing and
Inhibiting hormones
Synthesized by the adrenal medulla.

3. Neuroendocrine Hormones
 Synthesized by neuroendocrine cells, released in
circulating blood Eg. Oxytocin & ADH

4. Paracrine
 Target cell is located just near the secretory cell Eg.
Insulin & Glucagon
 Synthesized by Endocrine gland released in
interstitial fluid
Hormones Secreted by the Hypothalamus
5. Autocrine GHRH (Somatotropin Releasing Factor)  (+) GH
 Also released in Interstitial fluid, acts on the same GHIH (Somatostatin)  (-) GH
cell that synthesized them Eg. Tumor Growth factor PIH (Dopamine)  (-) PrL
PRH  (+) PrL
6. Cytokines TRH  (+) TSH, PrL
CRH  (+) ACTH, MSH
 Peptides synthesized and released by the cells of
GnRH  (+) FSH, LH
immune system
 Releasing/Inhibiting Hormone- Hypothalamus
 Trophic Hormones- Ant Pituitary
Functions of the Endocrine system
 Chemical Homeostasis General Characteristic of Hormones
 Fat, CHON, CHO metabolism  Secreted by secretory cells called Endocrine Glands
 Water and electrolyte metabolism  Thrown directly to body fluids, mostly in circulating
 Reproduction blood
 Estrogen, Progesterone, Testosterone  Regulate existing body function or process
 Growth  Long Latent period and duration of response
 GH, TH, Insulin, Progesterone estrogen testosterone  Low plasma concentration
 Behavior  Transport BLOOD
 Free= ACTIVE

Page 41 of 89 By: Ayesha Bea Federizo Batch 2017


GRAND PHYSIOLOGY B FINALS REVIEWER

Protein-bound
 Mechanism of Action of a Hormone
 INACTIVE
 Prolongs plasma half life 1. Hormone Receptor Interaction
 Facilitate transport of hormone to a. Receptor
target cell  Chemical subunits
 Reservoir  Most are proteins- can be degraded
 Binding proteins are produced by liver, and estrogen or resynthesized
can stimulate the production of such  Stereospecific
 Main site of Inactivation is in the LIVER, Cleared by
kidneys  Up Regulation
o If there is an increase in 1 hormone, this can
Classification According to Chemical Composition increase the number of receptors for itself or
POLYPEPTIDES another hormone
o Eg. Increased Estrogen Increased
production of Oxytocin receptors in
Myometrium
 Down Regulation
o (+) Excess hormones  , this can decrease
 Can be stored, secretion is not continuous, only upon
the number of receptors for itself or another
stimulation
 Secretion is via exocytosis facilitated by Ca ions hormone
 Soluble, transported in unbound form, shorter plasma half o Increased Progesterone Decreased
life Estrogen Receptor
 Cannot cross cell membrane, cannot be given orally
 ALL Hypothamalic hormones: GHRH, GHIH, PRH, TRH,
CRH, GnRH, well, EXCEPT PIF/Dopamine Location of Rceptors
 Anterior and Posterior Pituitary hormones: ADH, CELL MEMBRANE
Oxytocin, GH etc
Protein
 TSH FSH LH  Glycoproteins
Peptide
 PTH, Calcitonin
Catecholamines
 Insulin Glucagon
cAMP Other than
BIOGENIC AMINES cAMP/Phospholipase C/
 Synthesized from the Amino Acid Tyrosine whatever
 Thyroid Hormones: T3, T4  CRH, GHIH  GnRH, GHRH, TRH
 Bound to thyroglobulin  ACTH, FSH, LH, TSH  Oxytocin
 Stored, secreted upon stimulation, not  HCG, PTH, Calcitonin  ADH (V1 receptor in the
continuous.  ADH (V2 receptors in the vascular smooth muscle)
 Released via exocytosis DCT)  Catecholamines (α
 99% bounded; 1% free  Catecholamines (β receptor)
 Can cross cell membrane receptors)  A II (vascular smooth
 Long halflife  Angiotensin II (epithelial muscle)
cells in renal tubules)  Insulin
 Ep, Nep, PIF (dopamine)  Glucagon
 80% Ep, 20% Nep, Very small PIF (adrenal
medulla)
 PIF produced in hypothalamus and adrenal Cytoplasm Nucleus
medulla Steroid Hormones: Thyroid Hormones
 Either bound or loosely bound to albumin  Estrogen, Progesterone,  T3 , T4
 Cannot cross cell membrane Testosterone
Steroid  Cortisol, Aldosterone,
 Synthesized from Cholesterol Androgens
 Lipid soluble, insoluble to circulating blood  Vit D
 Bound to plasma proteins, long half life
 Not stored, whatever is synthesized is also released Regulation of Endocrine Secretion
 Can cross cell membrane
 Estrogen, Progesterone, Testosterone
 Cortisol, Aldosterone, Androgens
1. Indirect Nervous Control
 Vit D

Classification According to the Site of Target Cell


Local Hormones General Hormones
 Target cell is located  Target cell is located far
near the secretory cell from secretory cell
 Released in the blood
 H-H target gland axis
 GI Hormones  Thyroid Hormones
 Glucagon,Insulin  Growth Hormones  Ant Pituitary Gland, thyroid, adrenal cortex, ovaries,
(released in Interstitium)  Trophic Hormones: Eg. testes
 NTA- Ach, Nep TSH

Page 42 of 89 By: Ayesha Bea Federizo Batch 2017


GRAND PHYSIOLOGY B FINALS REVIEWER

B. Positive Feedback Mechanism


 Excess free hormone from a target gland further
stimulates the hypothalamus and anterior pituitary
gland

 Only true for Estrogen

 3 Levels of Endocrine Disorders


Primary Target Gland i.e., Thyroid etc
Secondary Anterior Pituitary Gland

 ACTH dependent Zona Fasciculata and Reticularis Tertiary Hypothalamus


 Aldosterone from Zona Glomerulosa is not ACTH
dependent. It is regulated by RAAS and K.

2. Direct Nervous Control


 ADH Oxytocin
 Nep, Ep

C. Modified Negative Feedback Mechanism


 AKA Physiologic Response Driven Feedback
 Independent of the hypothalamus and anterior
pituitary gland
 Regulated secondary to a blood metabolite
 PTH  Calcium Levels

REGULATION OF HORMONE SECRETION

A. Negative Feedback Mechanism


 aka Endocrine Driven Axis Feedback
 Excess free hormone from a target gland will inhibit  Insulin, Glucagon  Blood Glucose
the hypothalamus and anterior pituitary gland.  Aldosterone  mainly Hyperkalemia
 Hypothalamus-Long loop; Anterior Pituitary gland-
Short Loop D. Permissive action of a Hormone
Eg.

 Hormone A does not act directly on the target cell.


Instead, hormone A increases the production of
hormone B and hormone B will act on the target cell.

ANTERIOR & POSTERIOR PITUITARY GLAND

 Bet Cortisol and Androgen, It is mainly excess in Cortisol


 In females, Progesterone NFM mainly affects LH
 In Females, Estrogen NFM mainly affects FSH (beginning
of monthly cycle)
 In Males, Testosterone NFM mainly affects LH ; If
Increased Inhibin  FSH is affected

Page 43 of 89 By: Ayesha Bea Federizo Batch 2017


GRAND PHYSIOLOGY B FINALS REVIEWER

I. ANTERIOR PITUITARY GLAND


GH REGULATION
Types of Adenohypophyseal Cells
Staining Characteristics
Granular Agranular Chromophobes
 Acidophils (40%)
– GH, Prl –ACTH
 Basophils (10%)
– FSH, LH,TSH, ACTH

Secretory Activity
Somatotropes GH/STH 50%
Lactotropes Prl, GH 10-25%
(Mammosomatotropes)
Corticotropes (POMC) ACTH, β-LPH 15-20%
Gonadotropes FSH, LH 10-15%
Thyrotropes TSH <10%  Controlled via Negative Feedback Mechanism: Inc IGF
1 Ant PG is Inhibited  NO GH; and the
A. GROWTH HORMONE Hypothalamus is also stimulated  INC GHIH/
Somatostatin  Ant PG is Inhibited  NO GH (Dual
 GH Stimulates Liver to Secrete IGF 1/ Somatomedin 
Inhibition);
Maximizes effect in promoting growth
 Excess plasma GH  Hypothalamus is Inhibited  NO
GROWTH HORMONE IGF-1
GHRH  Ant PG is Inhibited  NO GH
 Polypeptide; Specie  Major Intermediary of
specific the physiologic action Growth Hormone Release
 Uses cytokine receptors of GH  Presents diurnal rhythm
(JAK2 and STATs)  Regulates cellular  Peak secretions before midnight and in the early morning
o STAT = Signal proliferation,  Secretion is pulsatile
Transducer and differentiation, and
Activator of metabolism Factors Promoting Growth Factors Inhibiting Growth
Transcription  Has autocrine, Hormone Release Hormone Release
 Plasma level higher in paracrine, and
Hypoglycemia ;Fasting Hyperglycemia
infants and children than endocrine effects
Exercise Lack of exercise
adults
Increase in plasma level of amino Increase in plasma free fatty acids
 Released in a Pulsatile
acids – Protein meal –
manner ARGININE (most effective in
 Stimulates the liver to stimulating GH
produce IGF-1 NREM Sleep - (SWS)  Deep REM Sleep – (FWS)  Light Sleep
 Sodium retention Sleep
 aka Somatotropin (STH)  aka Somatomedin Norepinephrine and α-adrenergic α-adrenergic blockers
agonists
 Half life 6-20mins  Half-life is about 12 Β-adrenergic antagonists β-adrenergic agonists
hours
Apomorphine and L-Dopa Serotonin Agonist
 Hyperglycemic Hormone,  Insulin-like Activity
Hormones of puberty (Androgens Chronic intake of steroids
Inc Insulin resistance,
and Estrogens)
Antagonizes insulin effect
on muscle and adipose Stressful stimuli - Fever Somatostatin
 Increases lipolysis  Anti-Lipolytic Activity Ghrelin – coordinates food intake Increase level of Somatomedin
with growth and Growth Hormone
 Directly affects the liver,  Secreted by many
muscles, and adipose tissues of the body
tissue but the predominant Important Hormones in Human growth:
 Stimulates growth of source is the liver  T3 & T4  High during Childhood
human cells, most  Mitogenic and have  Androgen, Estrogen  Puberty only
especially bones and marked effects on  STH/GH
cartilages Linear Growth bone and cartilage
 Insulin
 Promotes protein anabolism, prevents protein catabolism
(+) Nitrogen and phosphorus balance
 Epiphyseal growth Growth Abnormalities
Excessive GH 
 IGF-I is the major form in adults  Early Onset (Before Puberty)  GIGANTISM
IGF-II is the major form in the fetus  Late Onset (After Puberty)  ACROMEGALY
GH and IGF  Maximized effect if epiphyseal plate is open  During Puberty ACROMEGALIC-GIGANTISM

I. Gigantism
 Excessive GH before puberty when epiphyseal plate is
still open

Page 44 of 89 By: Ayesha Bea Federizo Batch 2017


GRAND PHYSIOLOGY B FINALS REVIEWER

II. Acromegaly B. Melanocyte Stimulating Hormone


 Excessive GH after puberty when epiphyseal plate is
already closed  Stimulates the melanocytes to produce melanin  Even
 Complications: distribution
o Diabetes- most common cause of death  Adrenocorticotropic Hormone (ACTH)
o Visual deffect Tunnel Vision o Hormone with melanocyte stimulating effect
o Altered Protein anabolism o When present in excessive amount in the
o Hypertrophy of sweat & sebaceous glands blood, skin pigmentation occurs, but it not
o Galactorrhea evenly distributed
o Cardiomegaly, HPN o Affects not only the skin, also the mucosa
o Sexual dysfunction  Degree of pigmentation depends on the number of
o Peripheral Neuropathy melanocytes present in your skin, they serves as
o Prominent supraorbital ridge protection for UV rays
o Arthrosis o Lighter Skin: Dec melanocytes  Prone to
o Enlarged distal body parts skin CA
 Prognathism, Frontal Bossing, Big jaw, o Darker Skin: Inc melanocytes
normal number of teeth, teeth separated,
Large nose, spade shaped hands and feet  Recently, it was found out that MSH does not exist,
ACTH is actually the one stimulating the melanocytes
Other Growth Abnormalities to produce melanin, giving it a melanocyte
Tall Stature Endocrine Disorder Tall Stature Non Endocrine stimulating effect
Disorder
 Sexual Precocity  Cerebral Gigantism Abnormal Skin Pigmentation/Lack of Pigmentation
 Thyroxicosis  height (Soto’s Syndrome)  Albinism – congenital absence of melanin
dec when left untreated  Marfan’s Syndrome  Piebaldism – congenital defect characterized by patches
 Homocystinuria
of skin that lacks melanin
 Beckwith-Wiedemann
Syndrome
 XYY Syndrome  Degree of severity of injury in the Pituitary Gland
 Klinefelter’s Syndrome depends on the activity of the Anterior Pituitary
Gland, particularly to the amount of secretions

Dwarfism (Endocrine Disorders) Dwarfism (Non-Endocrine Pituitary Failure (Hypopituitarism)


Disorders) Decrease in the secretion:
Laron Dwarfism Kaspar-Hauser Syndrome Gn (FSH/LH Mild cases
 GH insensitivity due to a  pyschosocial dwarfism; TSH Mild to moderate
defect in GH receptors seen in neglected and ACTH Moderate to severe
and a marked decrease in chronically abused
GH (STH Severe cases
IGF-1 children
Malnutrition *GTAG
African Pygmies Syndromes of short stature
 IGF-1 fails to increase at (Turner’s) II. POSTERIOR PITUITARY GLAND (NEUROHYPOPHYSIS)
the time of puberty; Autosomal Chromosomal
partial defect in GH Disorders (Down’s)  Neurohypophyseal Hormones
receptors  Produced as prehormones
Glucocorticoid excess Chronic Cardiac/Pulmonary  Cosecreted with a peptide – Neurophysin
Disorders  Neurophysin I = for ADH
GI/Hepatic/Renal Disorders
 Neurophysin II = for Oxytocin
Cretinism Achondroplasia
 Thyroid Hormone  Autosomal dominant
Deficiency condition wherein there is
A. Anti-Diuretic Hormone (ADH
alteration in fibroblas  Aka Arginine Vasopressin (AVP)
growth factor receptor  receptors:
o V1a and V1b = Vasoconstrictive Effects (PI
Ca++)
o V2 = Anti-Diuretic Effects (G-Protein
cAMP)
 Half-life = 15-20 minutes
 For water balance; Facultative reabsorption of water in
late DCT and CD

 Location of Receptors
Plasma Osmolality R Organum Vasculosum of the
Lamina Terminalis
Volume sensitive R Subfornical Organs
Thirst Center R Superolateral Part of the
Hypothalamus  median
preoptic nucleus

Page 45 of 89 By: Ayesha Bea Federizo Batch 2017


GRAND PHYSIOLOGY B FINALS REVIEWER

 During labor or parturition  for uterine contraction


ADH Regulation  Estrogen  reduces the membrane potential of
Excitatory Stimuli for ADH Inhibitory Stimuli for ADH uterine smooth muscles, lowering the threshold for
Release stimulation (increase the number of Oxytocin receptor
Increase Effective PCOP (OPPP) Decrease effective PCOP mRNA in the uterus)
 1% increase in (OPPP)  Uterine effects – inhibited by progesterone
plasma osmolarity
Dehydration Overhydration Effects of Oxytocin
 Nausea, Vomiting, Major Effects: Minor Effects:
Diarrhea
Hypovolemia Hypervolemia
Milk ejection in lactation Control of Estrous Cycle
 5-10% decrease in BV
Ovarian Steroidogenesis
Warm/Hot Environment Cool/Cold Environment Testicular Steroidogenesis
Nicotine,, Clofibrate, Alcohol Intake Male Ejaculation
Carbamazepine , Angiotensin Induction of parturition Body osmolarity
Pain, Emotion , Stress ,  (Effective Natriuretic Agent)
Standing , Exercise Lipogenesis

ADH Abnormalities
Thyroid Gland
 Butterfly-shaped, located anterior part of the neck on
either side of the trachea
 Divided into right and left lobes joined by the isthmus at
appx at the level of the cricoids cartilage
 Non-palpable, weighs 15-25g (adult)
 Moves up during swallowing due to the thyroglossal duct

 Recurrent Laryngeal nerve – located behind TG, commonly


compressed when TG enlarges. Vital in Thyroidectomy, if cut 
loss of voice

 SIADH Can be seen in patients with cerebral and  Funtional unit  Thyroid follicles or acini contains
pulmonary diseases
Thyroglobulin, a large glycoprotein molecule and colloid
which stores newly synthesized thyroid hormones
 Well vascularised (5ml/g/min) via superior and inferior
thyroid arteries.
 Receives sympathetic innervations  only regulates
blood flow

 Hyperthyroidism  blood supply inc (+) Bruit upon


auscultation

BIOSYNTHESIS AND RELEASE OF THYROID


HORMONES
 Hypersecretion of ADH
 Long surgical procedures  secrete excessive ADH 
low plasma osm and dilutional hyponatremia  water
intoxication so monitor I&O (input & output)

B.
Oxytocin
 Synthesized mostly by the paraventricular nucleus
(hypothalamus)
 Supra-ADH; Para-Ox
 Uses G-protein coupled serpetine receptors which can
increase intracellular calcium levels

Excitatory Stimuli:
 Major: Nipple Stimulation  Milk letdown
 Prolactin  Milk production  BIOSYNTHESIS: ECF CYTOPLASM LUMEN
 Stimulation of reproductive body parts 1. Iodide uptake/trapping
 During Coitus  acts as vacuum to facilitate sperm to  Thyroid Gland can take up and also release Iodides in the
move up, thus fertilization occurs, free ride baby circulating blood
 Psychogenic stimuli – baby cries 

Page 46 of 89 By: Ayesha Bea Federizo Batch 2017


GRAND PHYSIOLOGY B FINALS REVIEWER

 Thyroid Gland needs 150 ug Iodide; Balance Diet 500 ug Iodide; Albumin Little of T3 & T4
80% excreted via kidneys, 20% taken up by TG  Highest plasma conc;
 Active Transport: Sodium-Iodide symporter lowest affinity
 T3 is more potent and active than T4 i.e. It is mainly in excess of
Iodide (ECF)
T3 causes Negative Feedback Mechanism

REGULATION OF THYROID HORMONE SECRETION


 Salivary glands, gastric mucosa, mammary glands, placenta,
choroid plexus, ciliary body  capable of transporting of iodides
but do not synthesize TH  Hypothalamic-Hypophyseal Thyroid Gland Axis
 Competitive Inhibitors = Perchlorate, Nitrate, Thiocyanate & o Major mechanism
Pertechnitate (anti-thyroid drugs)

 Passive Transport: Cl-I

o (+) TRH  (+) TSH  (+) T3, T4


2. Oxidation  TRH Uses Phospholipase C as 2nd messenger
nd
TSH  Uses cAMP as 2 messenger

3. Organification  Wolff-Chaikoff effect


o Autoregulatory mechanism of TG depending on
Iodide availability
o Deficiency/Excess in Iodides  Dec Thyroid
4. Coupling/Condensation hormone synthesis
o Eg. (+) Excess iodides  Na-I symporter and
Organification is inhibited  Dec Thyroid
 Thyroid Hormones hormone synthesis  Hyperthyroidism is
T4 DIT T3 MIT prevented
 Wolf-Chaikoff escape: happens when wolf-chaikoff effect
 STORAGE: Colloid, bound to throglobulin persists for several days  loss of inhibition Thyroid
 Can be able to supply body for 2-3 months hormone synthesis

 RELEASE: LUMEN  CYTOPLASM  ECF TSH + Membrane receptor


1. Endocytosis of Colloid (Pinocytosis) @ apical border
 Cytoplasm
2. Proteolysis of thyroglobulin = Free MIT, DIT, T3, T4
(Cytoplasm)  TSH stimulates Iodide uptake, oxidation, organification,
3. Deiodination of MIT & DIT coupling, endocytosis, T3 T4 release
 MOST IMMEDIATE ACTION: Inc Proteolysis of
thyroglobulin
 Iodine product here can either be released or recycled
4. T3 T4 excretion T3 & T4
 Synthesized from Tyrosine;
 Biogenic Amine, receptors are in the nucleus
 Crosses the cell membrane via OAT-P, a carrier protein
o OAT-P 1  Thyroxine/T4
o MCT 8  T3

 Acts on all type of cells except Spleen, lens of the eyes


 HORMONE TRANSPORT
and Gonads
TBG ( Thyroid Binding T4> T3
PHYSIOLOGIC ACTION OF THYROID HORMONES
Globulin)
 Lowest Plasma Conc; Growth Children: Physical and Mental Growth
Highest affinity  Growth Hormone Skeletal growth = Soft
tissue growth
Thyroxine Binding Prealbumin T4
Thyroid Hormone  Skeletal growth > Soft
(TBPA) /Transerythin
tissue growth

Page 47 of 89 By: Ayesha Bea Federizo Batch 2017


GRAND PHYSIOLOGY B FINALS REVIEWER

 Physical Retardation – can be corrected as Carbohydrate Increased glucose absorption; increased cellular
long as proper diagnosis and treatment is Metabolism utilization of glucose
applied and epiphyseal plates are still open  Hyperthyroidism (+) epinephrine induced
Congenital Pituitary Dwarfism glycogenolysis in liver  Inc CBG
Hypothyroidism Fat Metabolism Increased lipolysis > increased lipogenesis
> Cretinism  Ugly (-) GH  Both bones  Hypothyroidism  Hypercholesterolemia
dwarf, a non and Soft tissue Protein Normal dose anabolic
proportional dwarfism growth is loss  Metabolism Hyperthyroidism Hypothyroidism
 Bones fails to proportional dwarf Inc Protein Dec Protein
grow, but soft tissue  beautiful dwarf Catabolism  Dec Anabolism Dec
growth persists  (-) Mental muscle strength muscle strength
short stature and Retardation Vitamins Thyroid hormone Conversion of carotene to
limbs, pot belly, small vitamin A
skull, macroglossa
 Hypothyroidism  Carotenemia
(+) Mental
Adrenal Medulla Increased secretion of EP and NEP  Sympathetic
Retardation
adrenergic effects
 Mental Retardation- treatable as long as it
is diagnosed and treated before 6 months of THYROID DISORDERS
devt, beyond that, condition becomes  Goiter – Enlargement of the thyroid gland
permanent
2 Types:
CNS Increased cerebration, increased synaptic
transmission, increased nerve myelinization o Diffuse Goiter- generalized enlargement
Hyperthyroidism Hypothyroidism o Multi-nodular Goiter- lumpy enlargement
Hyperkinesia, Hypokinesia,
Insomnia, anxious, Sluggish movements GOITER Hyperthyroidism Toxic Goiter
fine tremors and speech, Euthyroidism Non-Toxic Goiter
drowsiness, poor
Hypothyroidism Non-Toxic Goiter
memory, poor
mental ability, low IQ
CVS Increased HR, SV,  Increased CO  Inc Systolic WHO: CLASSIFICATION OF GOITER
Pressure Stage Goiter Palpable Visible
Increased Heat production Vasodilation  Dec 0    
TPR  Dec Diastolic pressure  
1a  
Hyperthyroidism Hypothyroidism Even if neck is fully
Palpitaions, Bradycardia, extended
tachycardia, Systolic Hypotension 1b    
HPN, wide pulse Includes Only if neck is fully
pressure glands with extended
GIT Increased motility, secretion, absorption nodules
Hyperthyroidism Hypothyroidism 2    
Diarrhea Constipation No need Even in normal
position and when
Blood Inc cellular metabolism  Inc O2 consumption 
near the examiner
hypoxia  (+) Erythropoiesis
3   
Respi Increased IC activity decreased PO2, increased
Very large No need Even at
pCO2 (+) respiratory center increased RR
gland considerable
Endo Increased metabolism and clearance of various distance
hormones

Bones Increased bone formation and resorption
Hyperthyroidism Hypothyroidism HYPOTHYROIDISM
Resorption>Formation Formation and  Dry coarse hair  Easy fatigability
resorption dec  Goiter  Irregular
 Bradycardia menstrual cycle
Muscle Normal or physiologic dose of thyroid hormones
 Arthritis  Weight gain
 increase protein synthesis  increasing muscle
 Cold intolerance  Constipation
strength
 Dry skin, brittle  Myxedema-
Hyperthyroidism Hypothyroidism nails Generalized
Inc Protein Dec Protein  Puffy face
Catabolism  Dec Anabolism Dec  Dull expression
muscle strength muscle strength
Skin CAUSES OF GOITER
Hyperthyroidism Hypothyroidism
Heat Intolerance  Cold Intolerance 
Diaphoresis Dry skin Primary – Problem in Thyroid  Iodine deficiency (endemic
gland goiter)
Body Weight
 Goitrogen Intake (cabbage,
Hyperthyroidism Hypothyroidism turnips)
Increased appetite Dec appetite and dec  Late stage Thyroiditis /
and also Inc in in metabolism  Inc Hashimoto’s Thyroiditis
metabolism  Body weight  Thyroid Cancer
Decreased body  Post Total thyroidectomy
weight  Radiation therapy,

Page 48 of 89 By: Ayesha Bea Federizo Batch 2017


GRAND PHYSIOLOGY B FINALS REVIEWER

excessive intake of Anti Causes of Hyperthyroidism


thyroid drugs Primary  Grave’s Disease (exopthalmic goiter) 
Secondary - Problem in  TSH deficiency Autoimmune Plasma cells produce TSI
Anterior Pituitary Gland  Brain tumor (thyroid stimulating Immunoglobulin) 
Tertiary  Problem in Hypothalamus Goiter, Inc Thyroid Hormone secretion like
so all is Decreased (T3,T4, TSH
TSH, TRH
 Thyroiditis / Hashimoto’s Thyroiditis – Autoimmune disease  Solitary Toxic Adenoma- Benign Hormone
causing the plasma cells to produce antibodies that destroy Na-I secreting tumor of thyroid gland
symporter and inhibit peroxidise activity Secondary/  Pituitary Adenoma  Inc TSH, T3& T4, dec
 In a previously hyperthyroid patient, when treatment is excessive Extrathyroidal TRH, Goiter
 Fastidious or Iatrogenic Intake of Exogenous
 Hypothyroidism
T3 & T4 – (NOT RELATED TO PITUITARY
MYXEDEMA CRETINISM GLAND) Can cause rare cases of ectopic
 Occurs after birth  Occurs At birth or before thyroid tissue
 Most common in the face  Caused by Iodine Tertiary All are Increased
especially around the eyes deficiency on the part of  TSI, unlike TSH, is not subject to negative feedback mechanism
 Low husky voice the mother; rare cases despite of an increase in T3 and T4 levels, so there is continuous
 Carotenemia unknowingly pregnant
production of TSI
 Low IQ mom takes antithyroid
 Myxedema madness drugs crosses placenta
 Hypercholesterolemia depresses fetal thyroid Hyerthyroidism
due to: development T3 T4 TSH TRH Goiter
o low BMR= -40; and  Nonproportional dwarf / Primary
st
Inc (1 ) Dec Dec YES
decreased cholesterol ugly dwarf Secondary Inc
st
Inc (1 ) Dec YES
degradation  Mental Retardation Tertiary Inc Inc
st
Inc (1 ) YES
 Pot Belly
 Large Tongue /
Macroglossia Differential Diagnosis of Hyperthyroidism
 Deaf & Mute
 Short Limbs

Hypothyroidism
T3 T4 TSH TRH Goiter
st
Primary Dec (1 ) Inc Inc YES
st
Secondary Dec Dec (1 ) Inc UNLIKELY
st
Tertiary Dec Dec Dec (1 ) NO

DIFFERENTIAL DIAGNOSIS OF HYPOTHYROIDISM


 Normally, Thyroid gland should’ve taken up
TRH CHALLENGE TEST- IV admin of TRH 6hours=15%
Before TRH After TRH 24hours=25%
Hormone T4 TSH T4 TSH
Thyroidal Low High Same Inc TREATMENT
Pituitary Low Low Same Same
 Patient is Young, gland is small, and mild disease 
Hypothalamic Low Low Inc Inc
Anti-thyroidal drugsBlocks Peroxidase Catalyzed step
HYPERTHYROIDISM/THYROTOXICOSIS in TH synthesis (ano nga ulit yung 3 yun?) Eg:
 Hyperfunctioning Thyroid gland  Inc Thyroid hormone o Propylthiouracil(PTU)- inhibit peripheral
secretion. conversion of T4 to T3
Clinical Manifestations o Methimazole
Diaphoresis Heat Intolerance  Anions Compete with Iodide and blocks first step in
Tachycardia** Insomnia biosynthesis of TH
Goiter Infertility
 Radioactive Iodine  If drugs are not effective and
Irritability Muscle weakness
Anxious Weight loss gland continues to grow, this will destroy all
Diarrhea Fine tremors of fingers** hyperfunctioning cells; not for pregnant and those with
Inc SBP; Dec DBP Increased Pulse Pressure heart problems
Inc BMR High Output Failure
Pre-tibial myxedema (localized) Exopthalmos  If malignant Total Thyroidectomy
Hairloss If Benign Subtotal Thyroidectomy  leave 5g
**= Conclusive of
Hyperthyroidism
 Surgery even when thyroid is hyperfunctioning  Stress
 Hypothyroidism Generalized Myxedema  common in face
Stimulate further release of TH  Thyroid crisis or Thyroid
Hyperthyroidism Localized  Pretibial storm  Pre op, give Anti thyroid drugs and Iodides and
Beta blocker (minimize sympathetic effect especially on
the heart

Page 49 of 89 By: Ayesha Bea Federizo Batch 2017


GRAND PHYSIOLOGY B FINALS REVIEWER

Snow Whites’ Favorite  >17 mg/dL  Ca


(pero i don’t think lalabas to sa finals, pero malay mo!) precipitation may
occur
 Tetany  CNS depression
o Earliest Sign   Dec QT interval
Carpopedal spasm / Tachycardia
Trosseaus sign  Lack of Appetite
o Chvostek’s sign  Constipation
 Diarrhea

ABSORPTION AND SECRETION OF CALCIUM

A  After TRH admin in Pituitary Hypothyroidism


 Admin of high doses of exogenousT3
 Effect of TRH admin to Patient with Pituitary
Hypothyroidism
B  Prolonged Iodine deficiency
 Admin of high doses of Iodides
 After TRH admin in Primary Hypothyroidism
 Inc intake of Goitrogens
 Admin of high doses of Thiocyanate and Nitrate
 Hashimoto’s Thyroiditis (late stage)
 Effect of TRH admin to Patient with TPO deficiency  With the help of Vit D, Ca absorption in the intestine
C TROLOLOL occurs
D  Inc TSI production
 Plummer’s Disease Renal Excretion
 Hashimoto’s Thyroiditis (early stage)  Controlled by PTH
E  Pituitary Adenoma  10% or 100mg/day excreted thru urine
 Tertiary Hyperthyroidism
o 90% reabsorbed in the PCT, LOH, Early DCT
o 10% late DCT, early CD

Parathyroid Gland PHOSPHATE


 Normal value:
CALCIUM o Adults: 3-4 mg/dL
 Normal values: o Children: 4-5 mg/dL
o 9.4 mg/dL or 2.4mmol calcium/L Bones 85%
o 8.9-10.1 mg/dL Intracellular 14-15%
 Contraction of muscles Extracellular <1%
 Blood clotting (Factor IV)  2 forms:
 Transmission of nerve impulses  HPO4= 1.05mmol/L
Extracellular fluid Intracellular fluid  H2PO4= 0.26mmol/L
0.1% of total body calcium 1% of total body calcium  Acidic Plasma ↑H2PO4, ↓ HPO4
Most important, if altered, Cells and organelles Alkaline Plasma ↑ HPO4, ↓H2PO4
immediate s/sx manifests
 Bones – where the rest of the total body Calcium are Renal PO4 Overflow Mechanism
located  1mmol/L = critical value
Calcium in the Plasma  <1mmol/L
Combined with Combined with Ionized o PO4 will be reabsorbed and will not go into the
plasma CHON anionic substances urine.
(citrate, PO4) o All PO4 in the glomerular filtrate will be absorbed
41% (1mmol/L) 9% (0.2mmol/L) 50% (1.2mmol/L or  >1mmol/L 
2.4mEq/L o excretion is directly proportional to additional
non-diffusible; Most important form PO4
cannot pass through  Functional calcium BONE 
the capillary Composed of 2 major components:
membrane
Organic matrix Bone salts
Hypocalvemia Vs Hypercalemia  90-95% collagen fibers  Compressional strength
 Powerful tensile
Hypocalcemia Hypercalemia
strength
 ↓ Ca  ↑ Na entry to the  ↑ Ca  Ca blocks Na entry  5-10% ground substance  2 major bone salts in the
cell  Hyper-excitable cell to the cell  No action  Chondroitin sulphate body
potential  Ca & PO4
 Hyaluronic acid 
 Below 50%: tetanic muscle  12 mg/dL effects Hydroxyapatite form
contraction, seizures begin to appear (crystalline salts)
 6mg/dL: Tetany  15 mg/dL  marked  Others:
 4mg/dL: Lethal effects  Mg, Na, K, Carbonate

Page 50 of 89 By: Ayesha Bea Federizo Batch 2017


GRAND PHYSIOLOGY B FINALS REVIEWER

do not form crystals  Osteoblasts for bone deposition; starts to act if (+)
 Strontium, uranium, osteoclastic activity; Secretes Osteoprotegerin
plutonium, lead, gold  (OPG) competitive antagonist to OPGL  no bone
foreign substances that lysis
can form crystal salts and
may cause osteogenic
sarcoma
DEPOSITION AND ABSORPTION OF BONE
 Occurs even with normal calcium levels to maintain
PRECIPITATION AND ABSORPTION OF CALCIUM &
equilibrium
PHOSPHATE
 For Rearrangement of shape of the bone
 For Proper distribution of support of mechanical forces
 Ca is found in almost all parts of the body but it is only in
 For Renewal of “old degenerated organic matrix”
the BONES that it precipitates.
 Bone strength proportion to bone stress
 Hydroxyapatite in ECF does not precipitate
o ↑bone usage  ↑osteoblastic activity 
 Pyrophosphate – present in all other tissues; inhibits the ↑thickening of bone
precipitation of Ca and PO4 o ↓bone usage  atrophy
o Diminished Pyrophosphate atherosclerosis o ↓bone stress ↓bone deposition  ↓bone
strength
MECHANISM OF BONE CALCIFICATION
VITAMIN D
 Increase absorption of Ca and PO4 in the gut, bones
(deposition) and kidneys (reabsorption)
 Decreases renal calcium and PO4 excretion
 For Bone deposition
 Forms complex with Retinoid-X receptors

 In some cases, the osteoblast doesn’t form


hydroxyapatite  become osteocytes or quiescent
osteoids  source of osteocytes during emergency

 Without hydroxyapatite  Bone remains amorphous 


will be the source of Ca during rapid dec in Ca levels
BONE RESORPTION
 Breakdown and assimilation of components of bone
 Happens in cases of chronic hypocalcemia wherein all
amorphous bones are used up  to inc Ca
 Osteoclasts For Bone resorption

 Low Ca  PTH stimulate kidneys  kidneys will activate


25-Hydroxycholecalciferol to become 1,25-
Dihydroxycholecalciferol  binds to Calbindin  binds
to calcium for intestinal absorption
 ↑Calcium levels  conversion of Vit D via PTH to active
form is prevented

PARATHYROID GLAND
 Situated near thyroid Gland
 In cases of thyroidectomy:
o If 1 or 2 parathyroid glands are removed  OKAY
o If 3-4 parathyroid glands are removed 
hypoparathyroidism

Page 51 of 89 By: Ayesha Bea Federizo Batch 2017


GRAND PHYSIOLOGY B FINALS REVIEWER

PARATHYROID HORMONE (PTH) Dec PTH Normal PTH Normal PTH


 Controls extracellular Ca and PO4 Dec Ca Dec Ca Normal Ca
 Regulates intestinal absorption, renal excretion of PO4, Dec PO4 Inc PO4 Normal PO4
reabsorption of Ca, exchange between ECF and bones
 Excess PTH Hypercalcemia RICKETS  Affects children
Decrease PTH Hypocalcemia  Highly dec PO4; slight dec
Ca
 Due to lack of Vit D
EFFECTS OF PTH ON CA AND PO4  Causes Tetany and Weak
 Increase calcium and phosphate resorption from bone bones
and Inc reabsorption of Ca in Late DCT, CT, Early CD and
Asc LOH Inc Serum Ca levels OSTEOMALACIA  Fat Malabsorption  No
 ↑PTH ↑osteoclastic activity  Bone resorption Inc Vit D absorption Low
Serum Ca Serum Ca and PO4  Inc
 ↑PTH ↑ renal excretion of PO4 in PCT  Dec Serum Osteoclastic activity (bone
PO4 resorption)  Osteoblast
Others: compensates: forms new
bone and new cells for
 ↑ reabsorption of Mg and H ions bone growth  BUT,
 ↓ Na, K and amino acid ions remember, Ca and PO4
* Bones levels are decreased  No
1. RAPID PHASE 2. SLOW PHASE hydroxyapatite  Weak
bones
PTH causes rapid removal of salts Activated osteoblasts and
from amorphous bone crystals osteocytes send signals to
osteoclasts Osteoporosis  Sedentary lifestyle  No
bone stress  Inc
Acts more on the amorphous Osteoprotegerin Ligand (OPGL) is
osteoclastic activity 
bones secreted  stimulates osteoclast
bone atrophy
to mature  bone resorption
Congetinal Hypophosphatemia  Vitamin D resistant rickets
 Failure to reabsorb PO4 by
Control of PTH by Calcium the renal
 Dec Ca  Inc PTH  prolonged  Parathyroid gland tubules
hypertrophies Eg. Rickets and pregnancy  May present as
 Inc Ca (Diet high in Ca, prolonged sun exposure etc)  hypoparathyroidism
Dec Parathyroid gland activity  atrophy  May lead to osteomalacia

CALCITONIN  Calcium, For the


PREGNANCY & LACTATION developing baby and as a
 Peptide hormone from thyroid gland
component of breast milk
 Opposite effects of PTH  Dec Ca and PO4  Management: Inc Ca
 ↓ absorptive activities of osteoclasts and↓ formation of intake  prevents
osteoclasts secondary
 Effect is weak in humans hyperparathyroidism

HYPERPARATHYROIDISM
PRIMARY SECONDARY
HYPERPARATHYROIDISM HYPERPARATHYROIDISM
Hypersecretion of PTH  Extreme Chronic Hypocalcemia 
osteoclastic activity Osteoblast Hypersecretion of PTH
compensates Inc alkaline
 VIt D and Calcitonin works, in contrast with the other. PTH, phosphatase
on the other hand Increases INTESTINAL absorption of both Causes: Causes
Ca and PO4 but, in the bones it only resorps Ca and in the  Adenoma/carcinoma  Chronic renal disease
kidneys, it only reabsorbs Ca. So, overall effect of PTH  Inc  Von Reckling Hausen’s  Rickets / Osteomalacia
Serum Ca, Dec Serum PO4 disease  Pregnancy and lactation
 PTG Hypertrophy
CLINICAL CONSIDERATIONS: S/Sx S/Sx
 High Serum Ca , Low  Low Serum Ca , High
HYPOPARATHYROIDISM
Serum PO4 Serum PO4
TRUE PSEUDOHYPOPARAT PSEUDO-  Brittle bones
HYPOPARATHYRO HYROIDISM PSEUDOHYPOPARA  Osteitis fibrosa cystic
IDISM THYROIDISM  Calciuria, Phosphaturia 
Causes  Resistance to  Genetic Renal lesion 
 Hypoplasia/ PTH; Biologically defects Nephrolithiasis &
Congenital inert PTH o Short stature Nephrocalcinosis
absence of  Circulating PTH o Round face  PARATHYROID POISONING
PTG inhibitors o Short metacarpals
 Accidental  Auto-antibodies Extreme elevation of PTH rapid Inc of Ca & PO4 
o Ectopic bone
removal of to PTH receptor -knuckle knuckle hydroxyapatite develops outside bone  Metastatic
PTG  Kidneys do not dimple dimple sign calcification in alveoli, Kidney tubules, stomach, arteries
respond to PTH

Page 52 of 89 By: Ayesha Bea Federizo Batch 2017


GRAND PHYSIOLOGY B FINALS REVIEWER

ADRENAL PHYSIOLOGY EXCRETION


 Degraded in Liver and Kidneys
ADRENAL GLANDS  Mainly in liver by conjugation with glucoronic acid and
4gms; lies at superior pole of both kidneys sulphate
2 Parts:  25% feces || 75% urine
Adrenal Medulla
 Secretes Catecholamines  Epi & NorEpi GLUCOCORTICOIDS
 Functionally related to Sympathetic NS 1. Cortisol (95%)
Adrenal Cortex  Aka Hydrocortisone
 Secretes Corticosteroids:  Primarily affects glucose metabolism
Mineralocorticoid– Aldosterone
 MOST IMPORTANT GLUCOCORTICOID
Glucocorticoid- Cortisol
Sex Androgens 2. Corticosterone
 3 layers: 3. Deoxycorticosterone
Zona Glomerulosa – Aldosterone 4. Aldosterone  mainly mineralocorticoid
Zona Fasciculata – Mainly Cortisol
Zona Reticularis – Mainly Androgens REGULATION
 Whatever happens to Glomerulosa, happens only in  STRESS  Major Stimulus for secretion of CRF
Glomerulosa, Fasciculata and Reticularis is not
(Corticotropin Releasing Factor) in Hypothalamus 
affected.
APG is stimulated  releases ACTH  adrenal cortex is
stimulated  Cortisol is released
ADRENAL STEROIDOGENESIS
 Negative feedback Mechanism: Inc Cortisol  APG
 Parent Compound of all human steroids  and Hypothalamus is Inhibited
CHOLESTEROL
 Major contributor for cholesterol  Plasma LDL EFFECT OF ACTH TO ADRENAL CORTEX
 Rate Limiting Step: Cholesterol  Pregnenolone by the  Main stimuli of Adenylyl cyclase  ACTH
enzyme cholesterol desmolase or cytochrome P450  cAMP acts as second messenger (receptors are in Cell
sidechain cleavage enzyme membrane) and it stimulates cholesterol desmolase for
the conversion of cholesterol to pregnenolone
ADRENAL STEROID HORMONES
CIRCADIAN RHYTHM  Supraoptic nucleus
NATURAL STEROIDS  Guides release of CRF, ACTH and Cortisol
CORTISOL o Highest in early AM
 Highest Plasma concentration o Lowest in late evening
 Major glucocorticoid activity (but same  Interpret cortisol levels base on sleep cycle, if you
Mineralocorticoid activity)
change sleep pattern  cycle changes too.
 Effect is best achieved when dose is high
ALDOSTERONE
 Highest mineralocorticoid activity GLUCOCORTICOID FUNCTION
 Lowest Plasma concentration 1. Gluconeogenesis (synthesis of Amino acid to
CORTICOSTERONE glucose)
 Little mineralocorticoid and glucocorticoid activity  Main effect
 Increase in enzymes of gluconeogenesis
SYNTHETIC STEROIDS  Mobilizes amino acids from extrahepatic tissue
 Dexamethasone  Highest glucocorticoid activity; no
Mineralocorticoid activity 2. Dereases Glucose utilization
 Prednisolone little glucocorticoid activity; little  Dec oxidation of NADH to NAD+; (-) NAD+  No
Mineralocorticoid activity
glycolysis
 Methyl Prednisone little glucocorticoid activity; no
Mineralocorticoid activity
3. Diabetogenic
 Gluconeogenesis + dec Glucose uptake = Inc Blood
sugar
 Excess cortisol  Insulin is also inhibited  Adrenal
Diabetes
Binding Proteins To prevent rapid fluctuations and as
reservoir CORTISOL ON PROTEIN METABOLISM
 Reduces Cellular protein in all body cells EXCEPT
LIVER
 Inc Protein Catabolism
 Dec Protein Anabolism

Page 53 of 89 By: Ayesha Bea Federizo Batch 2017


GRAND PHYSIOLOGY B FINALS REVIEWER

 Dec Amino acid transport to Extrahepatic  Bone Resorption  Osteoporosis


tissues  Excess cortisol  Mineralocoid activity  water
 To Liver and Plasma Proteins retention  water toxicity
 Inc Amino Acid transport to liver  Deficient Cortisol  ADH effect is potentiated  water
 Inc Mobilization of Amino Acid from retention water toxicity
extrahepatic tissues
MINERALOCORTICOID
Aldosterone
 Secreted by Zona Glomerulosa
 HIGHEST Mineralocorticoid activity
 Cortisol has the same Mineralcorticoid and
glucocorticoid activity
CORTISOL ON FAT METABOLISM
 For Na Reabsorption and K secretion
 For Fatty acid Mobilization from adipose tissues
 Cells cannot utilize glucose  Glycerophosphate REGULATION OF ALDOSTERONE SECRETION
is not produced Inc Free FA in plasma  Inc 1. MAINLY STIMULATED BY: HYPERKALEMIA
glucose for energy 2. Angiotensin 2 (next of importance)
 Oxidation of Fatty Acids in cells 3. Very Slightly inhibited by Hypernatremia
 Stimulates appetite, more generation of fats than  ACTH is necessary for aldosterone secretion but has
mobilization  Central obesity, Moon Facie, Buffalo
little or no effect in its regulation
hump, Thin extremities
 (+) STRESSOR = (+) CORTISOL RELEASE
FUNCTIONS OF ALDOSTERONE
ANTI INFLAMMATORY EFFECTS OF CORTISOL
 Inc Renal tubular reabsorption of Na and secretion of
 Sometimes, Inflammation is more damaging than the
K in principal cells of DCT and CD
trauma itself
 Inc Tubular cells secretion of H in Intercalated cells
 Infection/Trauma  Causes Tissue Inflammation  may
 Principal K (Principal cells  K )
cause tissue and even organ destruction  Give
Inter-H (Intercalated cells  H)
Corticosteroid  Suppress Immune system  prevents
 Excess aldosterone  Inc ECF (5-15%) and arterial
and suppresses Inflammation  Rapid healing
pressure (20-25 mmHg), Slight only because of
Aldosterone Escape
FIVE MAIN STAGES OF INFLAMMATION
 Stage 1-4 is blocked by Cortisol  ECF and Arterial Pressure Inc  atrium senses
increase in Venous return  ANP is released 
causes Pressure diuresis or Pressure Natriuresis
(Aldosterone Escape)

 Only Slightly increases Na levels bec if you can


remember, where Na goes water follows, there is
concomitant water reabsorption, so Na is diluted
immediately
ANTI INFLAMMATORY ACTION OF CORTISOL  Excess Aldosterone  Inc K secretion  Hypokalemia &
Muscle weakness
 Depolarization – all K will exit  causes very negative RMP
(hyperpolarization) Harder to reach critical firing level 
manifest as Muscle weakness
 Excess Aldosterone  Inc H secretion  Inc Blood pH
 Metabolic Alkalosis
 Phospholipase A2 – Parent compound of Leukotrienes and
prostaglandin productions.
 Too Little Aldosterone  K is not secreted 
Hyperkalemia and Cardiac toxicity
 Cortisol  Inhibits Phospholipase A2  No Leukotrienes
and PG’s  No inflammation
 Depolarization – K will not go out  RMP negativity is
decreased (hypopolarization)  Easier to reach critical
firing level  cell is more excitable  Cardiac
OTHER EFFECTS OF CORTISOL toxicity
 Blocks Inflammatory response to allergic rxns  Too little Aldosterone  H is not secreted  Dec pH 
 Dec # of Eosinophil and lymphocyte Metabolic Acidosis
 Stimulates Erythropoiesis Inc RBC  Aldosterone causes Na retention during period of
 Dec Cortisol  Anemia sweating and/or salivation (sweat and salivary glands)
 Excessive Cortisol Polycythemia  Aldosterone  for Intestinal reabsorption of Na
 Enhances production of Surfactant by stimulating Type 2 (colon) prevents Diarrhea
pneumocytes necessary for lung maturation
 When administering steroid injections to mother,
WOF inc blood sugar level, give insulin if Increased
CBG

Page 54 of 89 By: Ayesha Bea Federizo Batch 2017


GRAND PHYSIOLOGY B FINALS REVIEWER

CELLULAR MECHANISM OF ALDOSTERONE ACTION Susceptibility to stress  Hyponatremia


 Aldosterone is a steroid  Receptors are in Cytoplasm and Infection  Dec K secretion 
Hyperkalemia
o ACTH receptor is in Cell membrane utilizing  H retention  Metabolic
cAMP acidosis
 POMC (proopiomelanocortin)  Parent compound of Primary Hypoadrenalism / Addison’s disease  Dec Cortisol
ACTH , MSH, B-Lipotropin and Endorphin Secretion  APG compensates  Inc POMC  Inc ACTH  Increased
 If ACTH is released so are its siblings (haha) Melanin Pigmentation  “bronze skin appearance”

ABNORMALITIES OF ADRENOCORTICAL SECRETION


TREATMENT FOR Hypoadrenalism/Addison’s disease:
 Most Important  Give aldosterone to prevent
HYPERADRENALISM AKA CUSHING’S SYNDROME hypovolemic shock
 Hyper secretion of cortisol  Hypercortisolism  Cortisol administration
 CAUSES
Primary : (Inc Cortisol, Dec ACTH)
PRIMARY ALDOSTERONISM AKA CONN’S SYNDROME
 Adenoma of Adrenal Cortex
 No secondary cause bec APG doesn’t care about Zona
Secondary (Inc ACTH , inc Cortisol dec CRF)
 APG Adenoma  most common cause glomerulosa 
Tertiary (Inc CRF, Inc ACTH, Inc Cortisol)  Primary cause  Tumor in Zona Glomerulosa
 Abnormal hypothalamic function  Effects of too much Aldosterone
Others: (di ko kasi macategorize kung primary or what)  Hypokalemia  weakness and paralysis
 Ectopic ACTH secretion from tumour (lung  Slight Hypernatremia
tumor)  Slight Inc in ECF
 Prolonged steroid therapy exposure  Constantly Elevated BP
 S/Sx:  Diagnosis
 Central Obesity  Inc Aldosterone, Dec Renin:
 Moon Facie Tumor in Zona Glomerulosa  Inc Aldosterone
 Buffalo hump  Negative feedback mechanism  Renin
 Inc Androgen  Hirsutism and Acne, oily face should decrease
 Cortisol has mineralocorticoid activity  Water  Treatment:
retention  Hypertension  Surgical Resection of tumor
 Inc Gluconeogenesis dec Glucose cell  Medical:
utilization  Inc Blood sugar  Epleronone
 Dec extrahepatic tissue proteins  Muscle  Spinorolactone
weakness, striae, Susceptibilty to infection  Aldosterone Receptor blocker
 Inc Bone resorption  Osteoporosis
 Treatment: FAQ’s/MLAQ’s
Surgery  If pituitary adenoma  pituitary resection ACTH receptor Cell membrane
Transphenoidal hypophysectomy Steroid receptors (cortisol, estrogen, Cytoplasm
Adrenal Adenoma  resection of Adrenal cortex aldosterone etc)
Layer that secretes Aldosterone Zona Glomerulosa
Rate limiting step of Corticosteroid Cholesterol to
production Pregnenolone
Highest Natural Glucocorticoid Activity Cortisol
Highest Synthetic Glucocorticoid Dexamethasone
Activity
Highest Glucocorticoid Activity Dexamethasone
Highes Plasma Glucocorticoid conc Cortisol
HYPOADRENALISM AKA ADDISON’S DISEASE Cause of obesity in Hypercortisolism Inc Appetite
 More dangerous than Cushings Most potent stimulus in Aldosterone Hyperkalemia
 Dec secretion of Adrenocortical hormones  secretion
Glucocorticoids and Mineralocorticoids Amino acid of Epi and NorEpi Tyrosine
What adrenal condition causes Addison’s / Primary
Hyperpigmentation Hypoadrenalism
What causes this Hyperpigmentation Inc ACTH
Target organ of aldosterone Kidney DCT and CD
Cortisol is highest at Early morning
Most common cause of Cushing’s Pituitary Adenoma

Glucocorticoid deficiency S/Sx: Mineralocorticoid


S/Sx
deficiency Pancreas
 Composed of: Head, Body and Tail
 Hypoglycemia  Inability to reabsorb Na  2 Major types of tissue
 Dec amino acid and fat and water  dec CO and o Acini cells- secretes directly @ duodenum
mobilization Dec BP  hypovolemic
o Islets of Langerhans
metabolic functions  shock  death
Weakness and Inc  Reduced Na reabsorption

Page 55 of 89 By: Ayesha Bea Federizo Batch 2017


GRAND PHYSIOLOGY B FINALS REVIEWER

 Secretes hormones directly into glucose uptake muscles, adipose and


bloodstream, it has capillary network other tissues
GLUT 5 *Fructose transport Jejunum, Sperm
 Cell types: GLUT 6 LALALA LALALA
Cell Secretion Function Overall effect GLUT 7 *Glucose 6 Phosphate Liver and other
type % transporter in ER tissues
Alpha 20 Glucagon Stimulate Inc Hepatic glucose
Proglucagon Beta and production,
Delta cells Opposes Glycolysis = Inc INSULIN ON CHO METAB
Blood Glucose  Muscle glucose uptake and metabolism
Beta 75% Insulin Inhibits Inhibits peripheral o Remember:
C peptide Alpha cells glucose uptake, oppose
 during much of the day, muscles
Proinsulin glycogenolysis and
gluconeogenesis = Dec depends on fatty acids for energy
Blood Glucose source
Delta 3-5% Somatostati Inhibits Inhibits Intestinal glucose  Normal resting muscle is only slightly
n Alpha and absorption = Dec Blood permeable to glucose except when
Beta cells glucose stimulated by insulin
F <2% Pancreatic Inhibits Direct Pancreatic effect  In bet meals, amount of glucose is too
polypeptide Beta and small to promote significant glucose
Delta cells entry into the cell
o Muscles use large amount of glucose during:
INSULIN  Moderate or heavy exercise
 Secretion: Proinsulin is cleaved  C peptide dissociates  Few hours after meal: Inc Blood
 leaving behind 2 Amino Acid Chains: Alpha and Beta glucose  Inc Insulin secretion
subunits Connected via Disulfide linkages o If muscle is not exercising after meal:
 Insulin Receptors:  Abundance of glucose transported in
muscle cell  Glucose is stored as
2 Alpha Subunit 2 Beta Subunit Muscle glycogen
 External portion  Cytosolic Portion  Insulin increases the rate of transport of glucose into resting
 (+) Hormone binding  (+) Tyrokinase activity muscle cell for at least 15 folds. Without insulin, no matter
domain  When occupied, how high your extracellular glucose is, glucose cannot enter
receptor phoporylates the cell. Insulin facilitates this Uptake.
itself and other
proteins  Peripheral Glucose uptake via Facilitated diffusion
o Insulin receptors causes insertion of glucose
 Mechanism transporters in the membrane, eg:
Insulin Binds to Alpha subunits  Because of the disulfide linkages  Adipose tissues
(bet Alpha and Beta), The beta subunit becomes autophosphorylated  Resting skeletal muscle
activation of local tyrosine kinase  Phosphorylation of multiple  Insulin acts by enhancing glucose metabolism
intracellular enzymes including IRS or Insulin-receptor substrates   Glucose can enter working muscles without aid of
Translocation of multiple intracellular vesicles to the cell membrane  insulin
the vesicles carry glucose transport proteins, which binds with cell  Other tissues not affected by Insulin:
membrane  glucose uptake into cells  Nervous tissues- eg. Brain
 Kidney tubules
 Glucose Transporters  Intestinal mucosa
a. Secondary Active Transport  RBC
-Na-Glucose Co transport  B cells of pancreas
-Energy dependent  Liver – but it accelerates the uptake
SGLT1 SGLT2 EFFECT OF INSULIN TO LIVER
For absorption in Small For Absorption if Renal  Anabolic effects:
Intestine and Renal Tubule Tubule o Glycolysis  Glycogen storage
 Anti-catabolic effects:
b. Facilitated diffusion o Inhibits Glycogenolysis  No glucose release
-No need energy o Inhibits Gluconeogenesis  no glucose
generation
Transporter Function Present in  For liver glucose uptake, storage and use. Mechanism:
GLUT 1 *Basal glucose uptake Placenta, BBB , RBC, o Insulin  Inhibits Phosporylase  prevents
Kidneys, Colon, etc glycogen breakdown  glycogen remains
GLUT 2 *B cell Sensor B cells of islets of stored in liver cells
*Insulin independent langerhans, Liver, o Insulin  Enhances glucokinase  for
*Transport out of Epithelial cells of phosphorylation of glucose to glucose 6
intestinal and renal small intestines, phosphate and for stimulating glycogen
epithelial cells Kidneys synthase  for glycogen synthesis
GLUT 3 *Basal glucose uptake Brain, Placenta,  NET EFFECT: Increase Glycogen upto 5-6% of liver
Kidneys, Etc mass or 100g of stored glycogen in the liver
GLUT 4 *Insulin stimulated Skeletal and cardiac

Page 56 of 89 By: Ayesha Bea Federizo Batch 2017


GRAND PHYSIOLOGY B FINALS REVIEWER

WHEN GLUCOSE IS RELEASED FROM THE LIVER  Promotes storage of CHO, CHON and fats
Decreased blood glucose  Pancreas decreases Insulin release  Inhibits catabolism of proteins ; decreases amino acid
 Reverse effects: Stops Glycogen synthesis in liver, prevents release from cells especially muscle cells
further glucose uptake.  In the liver, depresses gluconeogenesis plasma
amino acids are conserved or stored
Inc Glucagon + Dec Insulin  Phosphorylase is activated   Without Insulin
Glycogen is broken down into Glucose 6 phosphate  Inc o Protein depletion &Increased Plasma Amino
Blood Glucose acids  Increase protein catabolism, decrease
protein anabolism
Dec Insulin Glucose 6 phospatase is stimulated  phosphate o S/sx: Protein wasting, Extreme weakness and
radical splits away from glucose  free glucose in blood Deranged organ functions

INSULIN ON FAT METABOLISM INSULIN AND GH


 Promotes Fat Synthesis and storage  Interacts synergistically for growth when given
 Fat sparer simultaneously  Dramatic Growth
 Factors that lead to Increase Fatty acid synthesis:  Both promotes cellular uptake of different selection of
o Inc glucose transport into liver cells  when amino acids which is required for growth
the 5-6% limit is reached  Glycogen synthesis
is inhibited; Glucose splits into pyruvate  INSULIN AND POTASSIUM
Converted to Acetyl Coa ( precursor to lipid  Insulin Increases NaK ATPase pump in most tissues 
synthesis  FAT FORMATION pumps K into the cell
 Excess Citrate and Isocitrate o For acute life threatening hyperkalemia
 Direct effect of activating Acetyl Coa Carboxylase 
For carboxylation of Acetyl Coa  Forms Malonyl PRINCIPAL ACTIONS OF INSULIN
CoA (1st stage of FA synthesis)  FA synthesis RAPID (secs) INTERMEDIATE (mins) DELAYED (hrs)
 Liver produces Lipoproteins  Inc Triglyceride levels
Inc transport of:  (+)Protein  Inc mRNA for
in blood Lipoprotein lipase is activated   Glucose Anabolism lipogenesis
Triglyceride is broken down into Fatty acids and Alpha  Amino acids  (-) Protein
Glycerol Phosphate  Hormone Sensitive lipase is  K Catabolism
Activated  Triglyceride is formed, the storage form  (+) Glycolysis
of fats in adipose tissues  (-)Gluconeogenesis

Insulin is Lacking : MECHANISM OF INSULIN SECRETION


 Hormone sensitive lipase is inhibited  (+) release of  Primary Insulin Controller: Increased Blood glucose
fatty acids from adipose into circulating blood Concentration  Glucose influx thru GLUT2 receptors of
 Storage of FA is almost blocked Beta cells  once inside, glucose is phosphorylated to G6P
by Glucokinase (rate limiting step)  G6P is oxidized to form
 Promotes Lipolysis  Release of Free FA  this ATP  ATP-sensitive K channels of the cell is inhibited  K
becomes main energy substrate used by all tissues channel closes and depolarizes cell membrane Voltage
EXCEPT THE BRAIN gated Ca channel opens  Ca influx  (+) fusion of docked
 Increases Plasma Cholesterol and Phospholipid insulin containing vesicles with cell membrane  Insulin is
Concentrations  worsens Atherosclerosis especially excreted in ECF via exocytosis
in people with uncontrolled DM
 (+) use of fat as energy substrate  Ketosis and REGULATION OF INSULIN SECRETION
Acidosis  may lead to coma and DEATH INCREASE DECREASE
 Inc Blood glucose  Dec Blood
 Insulin promotes glucose transport through cell  Inc Plasma FA glucose
membrane into the fat cells|| Some glucose is used to  Inc Plasma Amino acids  Fasting
form large Quantities of Glycerol Phosphate  B adrenergic Act  Somatostatin
o Glycerol Phosphate  Supplies Glycerol  Glucagon, Ach  A adrenergic Act
Combines with FA  forms TRIGLYCERIDES,  GI hormones: Secretin,  Norepinephrine
stored in Adipose tissues Gastrin CCK, GIP  Epinephrine
 GH, cortisol (alpha
With Insulin Without Insulin  Parasympathetic receptors)
Hormone Present Absent stimulation
Senstitive lipase  Insulin resistance, obesity
Phospholipids Decrease Increased  Sulfonylurea drugs:
Ketone Bodies Negative Positive Glyburide, Tolbutamide

INSULIN ON PROTEIN METABOLISM & GROWTH  Amino Acids strongly potentiates glucose stimulus for
 For transport of amino acids into the cells insulin secretion
 Increases translation of mRNA forming new proteins  GI hormones only moderately increases insulin
 Turns on Ribosomal machinery secretion, Anticipatory release
 Increases rate of transcription of selected DNA  Glucagon, GH, cortisol, Progesterone estrogen may
genetic sequences in the cell nuclei  increases RNA increase risk of DM development when secretion is
quantiyties and protein synthesis large in quantity and prolonged

Page 57 of 89 By: Ayesha Bea Federizo Batch 2017


GRAND PHYSIOLOGY B FINALS REVIEWER

CCK and Gastrin) * FFA,


INCREASED BLOOD GLUCOSE *=same with Insulin Ketones
 When blood glucose increases 2-3x normal:
 Plasma Insulin Increases 10-fold within 3- SOMATOSTATIN
5mins  Secreted by delta cells
 Then decreases about halfway back toward  Remember: delta cells inhibits Alpha & Beta cells
norm n another 5-10 mins  Hypoglycemic
 At about 15mins, insulin rises (even higher  Inhibits Intestinal glucose absorption
nd
than the initial) the 2 time and reaches  Inc secretion when:
plateau in 2-3hours  Inc Blood glucose
 Inc Blood glucose Inc Insulin Secretion  Inc  Increase amino acids and Fatty acids
glucose transport into liver, muscle and other cells   Inc GI hormones concentration
Dec blood glucose concentration to normal  Inhibitory effects
 Acts locally by suppressing within the Islets
ROLE OF INSULIN IN SWITCHING BETWEEN CHO and LIPID of langerhans  dec secretion of insulin and
METABOLISM glucagon dec utilization of absorbed
 Insulin promotes CHO utilization for energy and nutrients by the tissues  prevent rapid food
depresses fat usage exhaustion  makes food available for
 Low blood glucose  Insulin is inhibited  Fat is longer time
used (except the brain)  Decreased pancreatic polypeptide secretion
 High blood glucose  Insulin is stimulated  CHO is  Inhibits secretion of GIT hormones and dec
used instead of fats blood flow to GIT Decreases GI motility 
 GH and cortisol Extend period of time over which food
 secreted during periods of hypoglycaemia  nutrients are assimilated into the blood, dec
both hormones inhibits cellular utilization of CHO absorption and Inc water and
glucose (no glucose uptake)  Slow effect: electrolyte absorption
Fat utilization  APG Inhibits GH and Thyrotropin Secretion
 Epinephrine
 Enhances fat utilization during periods of
stress: exercise, anxiety, circulatory shock etc IMPORTANT
 Stress  Epi release  Inc plasma glucose  Normal Person
conc and a greater Inc in Plasma FA Fasting Blood Glucose = 80-90 mg/100 ml of blood
st
concentration 1 hour conc increases to 120=140mg.100 ml of blood
Within 2 hours  goes bact to controlled levels
GLUCAGON
 Starvation
 Large polypeptide hormone; Molecular weight: 3485 Gluconeogenesis  provides glucose to maintain blood
with 29 chains of amino acids glucose levels
 Secreted by Alpha cells
 Primary target  LIVER DISORDERS OF GLUCOSE METABOLISM
 Skeletal muscle is not a target tissue of this hormone DIABETES MELLITUS
 Hyperglycemic hormone  Syndrome of disordered metabolism  (+)
 Half life: 5-10 mins inappropriate hyperglycemia, secondary to
 Degraded mainly by liver  Defects in insulin production
 Autoimmune beta cell destruction
Mechanism:  Insulin insensitivity or resistance
Glucagon  Activates Adenylyl cyclise  Activates cAMP   Impaired action of insulin on target tissues
Phosphorylase B is converted to phosphorylase  (+)  Due to either an absolute deficiency of insulin
Glycogenolysis  Glycogen is broken down into glucose  Inc secretion or reduction in biologic effectiveness of
Blood Glucose insulin or BOTH
 Diagnostics:
Actions: Fasting Blood Sugar (FBS)
 Inc Glycogenolysis  No caloric intake for atleast 8 hours, no water
 Inc Gluconeogenesis intake
 Inc Ketogenesis OGTT
 Inc Ureagenesis  (+) fasting
 Inc Insulin secretion  3 blood extractions
 Inc Lipolysis a.Get fasting blood levels
 Decrease Lipogenesis st
b.1 hour after drinking 100 or 75g
glucose drink  blood glucose is as
Regulation: expected, high
Increase secretion Decrease c.2
nd
hour after  goes back to
Inc Blood Amino Acid (alanine, arginine) * Inc Blood glucose controlled level
same with Insulin* (+) insulin and
Exhaustive exercise somatostatin
Beta adrenergic stimulation (via cAMP) secretion
Vagal Stimulation Secretin

Page 58 of 89 By: Ayesha Bea Federizo Batch 2017


GRAND PHYSIOLOGY B FINALS REVIEWER

RBS or Random Blood Glucose  Always protect foot and check for any wounds as
 Should not be more than 200mg/dL patients may not feel pain and since there is delay in
wound healing  prone to infection + ischemia 
Hb1ac or Glycosylated Hgb gangrene
 shows the average level of blood glucose over the
previous 3 months
 It shows how well you are controlling your diabetes
HYPOGLYCEMIA
Insulinoma
(+) DM if  Adenoma of islets  Excessive insulin production
 Hb1ac = Greater than or equal to 6.5%
 10-15% malignant  administer 1,000g of glucose
 FBS= Greater than or equal to 126 mg/dL (7 mmol/L)
 OGTT= Greater than or equal to 200 mg/dL (11 mmol/L) every 24 hours to prevent hypoglycaemia
 RBS= Greater than or equal to 200 mg/dL (11 mmol/L) Insulin Shock and Hypoglycemia
 Do repeat testing to confirm  Blood glucose falls to
 50-70 mg/dL  CNS becomes excitable
TYPE 1 VS. TYPE 2 DM Hallucinations, extreme nervousness,
TYPE 1 TYPE 2 trembling, cold sweat, Tachycardia,
AKA Juvenile onset DM, Non Insulin Weakness, Apprehension, Hunger
Insulin dependent dependent DM  20-50 mg/dL  Clonic seizures, loss of
DM consciousness, confusion, drowsiness,
Age Of onset Usually <20 y/o Usually >30 y/o diplopia, headache
Body Mass Low to normal Obese  <20 mg/dL  Seizure stops  Comatose 
Plasma Insulin Low or absent Normal to High death
initially
Treatment:
Plasma Glucagon High, can be High, resistant to
suppressed suppression
 Immediate IV large quantities of glucose
Insulin sensitivity Normal Insensitive or
(D50-50)
reduced  Glucagon
Therapy Insulin, Lipid Diet, exercise, oral  Epinephrine
Lowering drugs hypoglycaemic  Hard candies, Juice
agent, Lipid lowering
drugs
Insulin (latter part of
disease progression)
Ketosis prone Positive Negative
 Ketosis is due to glycogen and protein breakdown
 Other factors that can cause Insulin resistance in type 2 DM
o PCOS/ Polycystic Ovarian syndrome
o Cushing’s syndrome
o Excess GH  Acromegaly

Metabolic Syndrome
 (+) if 3 out of 5 of the ff:
 Insulin resistant
 Fasting Hyperglycemia
 Lipid Abnormalities  Inc Triglycerides, Dec
HDL
 Hypertension
 Inc Abdominal circumference

Gestational Diabetes
 High blood glucose occurring during pregnancy
 s/sx: Polyuria, Polyphagia, Polydipsia
Rapid weight loss and asthenia, always feels
sleepy, dehydrated etc

Complications of DM
 Tissue Injury
 Inc risk for heart attack, retinopathy and
blindness, ischemia of limbs Gangrene
 Peripheral neuropathy and ANS dysfunction
 Impaired CVS reflexes, bladder control,
dec peripheral sensation, and other
symptoms of nerve damage
 Hypertension due to renal injury Antok ka na?
 Atherosclerosis die to abnormal lipid
metabolism

Page 59 of 89 By: Ayesha Bea Federizo Batch 2017


GRAND PHYSIOLOGY B FINALS REVIEWER

VISION 1 Posterior Pole, the area of Retina with most acute vision,
within it is a depression called
 Fovea Centralis, the more specific area of most acute
LAYERS OF THE EYE vision because of 1:1:1 photoreceptor interneuron Ratio,
Fibrous Layer/Coat that is 1 Photoreceptor (rods and cones) synapses with 1
 Outermost layer Bipolar cell which will also synapse with 1 Ganglion cell,
 Made up of: there is direct impulse to brain plus there is no Rods, no
 1/5 anteriorly Cornea blood vessels, Contains Densely packed Cones 
 4/5 Posteriorly Sclera GREATEST VISUAL ACUITY
Vascular or Muscular Layer/Coat
 Middle layer  3-4mm away from Fovea centralis  Optic nerve exits,
 Contains Stored Vit A which goes 15 degrees Medialwards or toward nasal
 Made up of: side of eyeball  forming now your Optic Disc or
 On the posterior side, Choroid, which is extended papilla
anteriorly. (+) Blood vessels, and melanin pigments o Contains axons of ganglion cells
from dark brown, light brown, gray, blue , as is seen in o No Photoreceptors  Action Potential
the Iris. cannot be generated  BLIND SPOT
 Expansion of the Choroid  Ciliary Body  Retina terminates anteriorly as  Ora Serrata
o Ciliary muscles – intrinsic muscle, Origin o No photoreceptors
is on Medial side or inner portion, Inserts
on Lateral side or outer portion  Using Opthalmoscope  Find the orange or yellowish portion
o Ciliary Process – outside part, secretes  Red Reflex  At its most center portion is your Fovea
Aqueous Humor Centralis
Ciliary Process contains Zonules or Ligaments that  An absent or reduced red reflex indicates an opacity of the
helps hold the lens. cornea (infection or scar), lens (cataract), or vitreous
REMEMBER: hemorrhage.
Ciliary muscles Contract  it is pulled toward Medial Side   Suspensory Ligaments and Ciliary muscles determines shape
loosens Suspensory Ligament  Lens Thickens or becomes of the lens
rounder  stronger lens  (+) or increased Light bending
COMPARTMENTS OF THE EYEBALL
Ciliary Muscles Relax  pulled toward Lateral side   Iris
Stretches or tightens Suspensory Ligaments  Lens thins or o Like in the Choroid, it Contains Deposited
flatten, dec curvature  weaker lens  (-) or dec Light
melanin pigments
Bending
o Opaque, Colored portion of the eye
o (+) Radial and Circular muscles (discussed
 Ciliary Body narrows anteriorly  Iris, gives color to
earlier)
the eyes. Contains:
o Regulates Light entering in the pupil
o (+) RADIAL or dilator pupillae muscles
 Color of the Iris is never black, it is actually Dark
- Innervated By the Sympathetic NS, C8 &
brown, Other color includes light brown, gray, blue
T1
(associated with Osteogenesis Imperfecta), Violet to
- (Destruction causes  Horner’s Syndrome)
Transparent if melanin pigments are absent, lastly,
-For Mydriasis or Pupillodilation
Green associated with presence of other pigments
(Pheomelanin)
o (+) Sphincter or Constrictor pupillae
muscles or Circular  Iris is Transparent  Difficulty in concentrating light
- Innervated by the Parasympathetic NS,
Cranial Nerve 3 or Occulomotor Nerve.  Pupil
Preganglionic Neuron  Edinger o Space in between Iris
westphal Nucleus, Postganglionic o Colorless, appears black depending on light
Neuron  Ciliary ganglion refraction
- For Miosis or Pupilloconstriction o Normal Size: 1.5mm ( Most constricted) –
10mm (most dilated)
 Space in between Iris  Pupils o Size is regulated by the circular and radial
 Anterior to Iris Anterior Chamber muscles of the Iris
 Posterior to Iris  Posterior Chamber
 Circular Pupilloconstrict  See
o Both Contains Aqueous Humor synthesized
lesser details
by Ciliary Process
 More posteriorly, at the back of Lens  Vitreous  Radial Pupillodilate  Bird’s eye
Space: view
o (+) Vitreous Body – has semisolid fluids so  Lens
fluid flows slowly o Anterior to vitreous body
o Held in place by Zonular or Suspensory
Nervous or Neural Layer/Coat
Ligaments which are attached to the cilliary
 Innermost layer, The Retina has 10 layers with different
process
cells in each
o Normally Biconvexed so light entering is
 Ganglion cells of Retina converges  forms Optic
converged to fall in the focal point (Retina)
Nerve
 Macula lutea, yellowish pigmented spot near the

Page 60 of 89 By: Ayesha Bea Federizo Batch 2017


GRAND PHYSIOLOGY B FINALS REVIEWER

Anterior cilliary vein empties into sclera area 


Venous Circulation
 Choroid
o Vascular portion IOP/ Intraoccular Pressure
Normal: 10-20 cmH2O
o Supplies oxygen and nutrients to eye Ave: 15cmH2O
structures
 Retina Glaucoma
o Neural Tissue with photoreceptors, Posterior  degenerative disease involving loss of retinal ganglia cells due
2/3 lining of the choroid to but not limited to inc in IOP as a result of poor drainage of
 Posterior Chamber the aqueous humor through the Anterior Chamber
o space between the Iris, zonule, and the lens  Beta blockers may help by decreasing AH production
that contains aqueous humor  Open Angle: Increase secretion of aqueous humor
o Lateral to suspensory ligaments, Posterior to  Closed Angle: Obstruction in the drainage
Iris
 Anterior Chamber MUSCLES OF THE EYE
o space between the cornea and iris that  Extraoccular Muscles
contains aqueous humor Muscle Action, Moves Nerve Innervation
o anterior to Iris and pupil eyeball
 Vitreous Chamber or Body Superior Oblique Downward & CN 4
o space between the Lens and the Retina that Laterally
is filled with Vitreous Humor (clear and Inferior Oblique Upward & Laterally CN 3
gelatinous material) Superior Rectus Superiorly & Medially CN 3
o maintains or gives shape to the eyeball Inferior Rectus Inferiorly & Medially CN 3
Lateral Rectus Laterally CN 6
ACCESSORY STRUCTURES OF THE EYE Medial Rectus Medially CN 3
 Eyelids
o Secretions hydrates the conjunctiva  Intraocular Muscles
o Tear films keeps the eye moist, for nourishment
Radial Muscles or Dilator Pupillae
and protection from injury and infection
 Glands: Circular Muscles or Constrictor Pupillae
Zeis Moll Kraus Meibomian Cilliary Muscles: Thickens lens when Contracted
Modified Modified Exocrine and Modified sebaceous gland. Thins Lens when Relaxed
Sebaceous Apocrine aqueous (+) lipid secretion, prevents
gland sweat secreting water from evaporating 
Gland accessory maintains eye moisture FORMATION OF IMAGE
lacrimal  Image is in retina, Retinal field is the one inside the
glands retina
 Object is in Visual Field, The one that you see is the
SYNTHESIS OF AQUEOUS HUMOR visual field
1. Ultrafiltration of Plasma  There is inversion of Image in retina  Bec of
o Ciliary body has fenestrated capillaries  lamination in Retina, as it goes up in the cerebrum 
filters both solutes and solvents along with You see an upright position of the Image
electrolytes and small proteins  Adequate Stimulus for Photoreceptor is Electromagnetic
2. Transport of Ions & Electrolytes toward the Posterior Light
Chamber  Can also be stimulated by Mechanical and electrical
o This will produce a concentration gradient stimuli if the intensity of the stimulus is greater than the
causing now  stimulus to see light rays. To see a phosphene ring 
3. Osmosis apply heavy pressure
o Fluid is now attracted towards posterior
chamber
 Stimulate Temporal side by pressing  See
light/Phosphene ring in nasal side
Aqueous Humor Vs Plasma  Stimulate Nasal side See light/ Phosphene ring in
Aqueous Humor Plasma Temporal side
(+) Prostaglandin (-) Prostaglandin  Rods are stimulated when adequate pressure is
(+) cAMP (-) cAMP applied
20x more Ascorbic acid and lactic
acid ACCOMODATION / NEAR RESPONSE
 Near Response is actually accommodation
OUTFLOW OF AQUEOUS HUMOR
o Thickening Of the lens
o Converging of the Eyeball
Conventional Pathway Unconventional
Pathway o Pupilloconstriction
Aka Anterior Circulation, Uveo-scleral circulation,
THICKENING OF THE LENS
Trabecular Pathway Posterior Pathway Principle:
% of drainage 80-95% 5-20%  Light Rays should enter and leave perpendicular to the
Drains into Canal of Schlemn Posterior Chamber surface
Route Collateral vessels Cilliary Body  Ascends to  Your eyes has the capability to bend light
suprachoroidal area 
Aqueous vein 

Page 61 of 89 By: Ayesha Bea Federizo Batch 2017


GRAND PHYSIOLOGY B FINALS REVIEWER

 This depends on the activity of your Ciliary muscles, Near point  closest distance at which you can focus on an object
whether it contracts or relaxes. ( alam mo na yan!)  Remember:
 Increased thickness  Better Light bending  Inc  The Human lens is normally Biconvex
Refractive capability  Biconvex lens  will converge light  focuses light in one
focal point
 Biconcave lens will diverge light  light scatters
CONVERGENCE
 Bring the visual axis toward each other as attention is
focused on nearer objects
 this is to focus towards the area of fovea centralis ERRORS OF REFRACTION
PUPILLOCONSTRICTION Emmetropic Eye Normal Eye
Ammetropic Abnormal Eye
 You focus the light in Fovea centralis  Focus on the  2 Factors:
lesser details
o Axial Length
REFRACTION o Refractive power of lens
 Capability to bend and absorb light
 Refractive Media of the Eyeball (CLAV) Myopia Hyperopia
o Cornea Aka Near Sightedness Far Sightedness
 Angulated surface  Bends light Causes Long Eyeball Small or short eyeball
most (2/3) Short Lens, Spherical etc Lens is too weak
o Lens Too Strong Lens
 Bends 1/3 of light Image Loc In front of Retina Behind Retina
o Aqueous Humor Mgt Biconcave lens  Biconvex lens  to
Diverge light first before converge light
 Bends small % of light it converges immediately
o Vitreous Humor Remarks Usually happens to
 Bends small % of light children who watch TV
 Focal Point/Principal focus at very near distance
o where all the refracted light meet behind the
lens
o where all light rays converge after the lens
 Nodal Points
o The 2 centers of the lens, i.e, Anterior and
Posterior centers
o Assures that light rays will enter parallel, at
right angles perpendicular to the surface,
does not bend light
 Focal Distance
o Distance between the LENS (nodal points)
and the FOCAL POINT/PRINCIPAL FOCUS,
considering also the light source
Presbyopia
 Old sightedness
 Lens becomes Inelastic by age 14, becomes evident at
age of 40
 Lens is weak  cannot thicken or thin out immediately
 Correction  Bifocal lenses, progressive lenses both for
near and far sightedness

Astigmatism
 Cornea is uneven or is oblong shaped, no lens problem
 Image falls behind or in front of Retina
 Correction  Cylindrical lenses- forms focal line
Contact lens  will refract light

Test For Visual Acuity


Snellen’s Chart ( Normal=20/20)
 Chart should be at eye level, 20 ft or 6m away
 Make sure room is well lighted and convenient
 Test one eye at a time, cover the other
 Mistakes are allowed, 2 at most
 Use E chart if patient cannot read (ask point of direction)
 How to Interpret:
o Numerator - distance at which the tested eye
can see
@ 20ft= Far point Lens is relaxed, no accommodation is needed;
furthest point an eye can focus on
o Denominator - distance at which a normal
healthy eye can see.

Page 62 of 89 By: Ayesha Bea Federizo Batch 2017


GRAND PHYSIOLOGY B FINALS REVIEWER

Eg. 20/40 o Bipolar cells are stimulated  releases NTA’s


 you correctly read at 20 feet away what can be read by a  Stimulates Amacrine cells  releases
person with normal vision from 40 feet away. inhibitory NTA’s  Inhibits ganglion cells

 To check for
astigmatism
 Normal : No
distortions seen

Minimum Separable Acuity/ Gap resolution

 Detects smallest space


between parts of a target
 Make sure that subject is
parallel and same level
 Are they kissing or what???
 Normal (accdg to Doc Valerio
and Guyton) = 25 seconds of an Arc

Minimum Perceptible Acuity/Spot detection

 detect smallest
possible target
Horizontal Pathway
 Pathway of vision mediated both by Amacrine and
Horizontal cells
 Amacrine and Horizontal cells sends impulse and
Vernier Acuity
influences activity of adjacent bipolar and ganglion cells
 These cells are not only directly connected to
photoreceptors
 Detects smallest lateral
o Axon of horizontal cell is connected to
displacement
dendrite of bipolar cell
o Amacrine cells is connected to the ganglion,
it also communicates with the bipolar cells
which will send impulse directly to the brain
 If lateral side is stimulated  it will not push through
VISION 2 to the visual centers  image is blurred
NEUROPHYSIOLOGY OF VISION Samplex: The horizontal cells inhibits the
Photoreceptors
RETINA The Amacrine cells inhibits adjacent bipolar and
ganglion cells
 Most posterior aspect or the innermost portion of
eyeball
 Light strikes perpendicular to axis  light falls in macula PHOTORECEPTORS: RODS AND CONES
lutea  Most specifically in Fovea Centralis, the area of  Both a Receptor and Sensory Neuron for vision
most acute vision bec of the 1:1:1 ratio discussed earlier  Rods and cones are actually modified dendrites of a
 Parafoveal or extrafoveal area  1 photoreceptor neuron, this dendrites have:
synapses with plenty of bipolar cells which will also o Outer segment – contains photopigments;
synapse with plenty of ganglion cells  image wouldn’t phototransduction
be as clear as compared in the fovea centralis o Inner Segment – contains organelles, i.e
 The photoreceptors, bipolar cells and ganglion cells are mitochondria ((+) ATP) which supplies
bridged by Horizontal and amacrine cells energy to process photopigments, also
contains pumps
 Horizontal and Amacrine cells o Nerve cell body or Perikaryon – contains the
o Inhibitory in nature nucleus
o Commonly releases GABA & glycine  o Synaptic body – axon in the photoreceptor,
inhibitory to retina forms synapses with other assoc neurons.
o May also release Ach and other NTA’s  also Releases NTA’s that will commonly stimulate
inhibitory to retina assoc neuron most specifically the bipolar
o Depolarization of Photoreceptors  cells
Stimulates Horizontal cells releases
inhibitory NTA’s  Bipolar cells are inhibited

Page 63 of 89 By: Ayesha Bea Federizo Batch 2017


GRAND PHYSIOLOGY B FINALS REVIEWER

Rods Cones  Rhodopsin is further converted into all-trans retinal


Shape (Outer Rod shape Cone or Pyramid because barthorhordopsin, lumirhodopsin etc are all
Segment) shape unstable chemicals so they must be converted to all-
Inner Segment Thin Thick trans retinal
 All-trans retinal may be converted into 11 cis-retinal by
Location Parafoveal or Mainly in foveal or
using isomerase
Extrafoveal central region 
 Synthesis of Rhodopsin= 11 Cis-retinal + scotopsin(from
dispersed retinal highly convergent
metarhodopsin II)  Rhodopsin
pathway retinal pathway.
 All-trans retinal or cis retinol is from RETINOL
Can also be seen in
 Trans-Retinol is from Vit A
parafoveal region
 All-trans retinal will activate G proteins specifically
Number 120 million 6 million Transducin
Sensitivity High Low
 When light strikes Rods  Rhodopsin is converted
Threshold Low High into all trans-retinal (same with cones)
Function Night or scotopic Day or Photopic or
vision greater color vision VITAMIN A
Acuity Lesser Higher  Stored in liver ito cells
Photosensitive Rhodopsin Iodopsin  First sign of Hypovitaminosis A or Vit A deficiency is
pigment Nyctalopia or Night Blindness
High sensitivity to Lower sensitivity to  Vit A shots:
light light o Every month  6,000 units
More Less Photopigments o Every 6 months  200,000 units
photopigments
captures more light Leak Channels
High Amplification, Low amplification  Non gated, always activated
single photon  So ions can go in and out
detection  This is true for K leak channel
Low temporal High Temporal
Resolution: slow resolution: fast Na Channel
response + long response + short  Not considered as a leak channel, but a gated channel, it
integration time integration time closes when cGMP levels are low
Most sensitive to Most sensitive to  Always open because of cGMP
scattered light direct axial rays  cGMP mediated
Rod System Cone system
Achromatic, 1 type Chromatic, 3 types of When GDP is acted upon by guanylyl cyclise it becomes 
of rod pigment cones cGMP  Plenty of cGMP Na channels opens
 Photopigments and Vit A (retinol) are stored in pigment
epithelium. When needed goes to apices of Na is abundant extracellularly so tendency is that there is Na
photoreceptor and converted to iodopsin and influx Na goes inside so membrane potential of outer
rhodopsin. segment of rods becomes less negative

BUT BUT BUT,


RODS - RHODOPSIN Remember, there is mitochondrion in inner segment for Na-K-
 Only detects black and white, which are not colors ATPase pump. So Na that enters the outer segment will go to
btw inner segment and again will be pumped outside + there is a
 Active Rhodopsin  Metarhodopsin 1 Active drive for K to go out via K leak channels  Maintained Rods
Rhodopsin  Metarhodopsin 1 ( Metarhodopsin 2 membrane potential
sabi ni Doc Valerio, Sabi din ni Guyton)
Howerever, When light strikes most apical portion of the outer
segment of rods Bleaching of Rods  all-trans retinal is
formed and will activate G-protein which always holds GDP
let goes GDP because of GTP GTP will now also activate
phosphodiesterase but all-trans retinal may also activate your
phosphodiesterase Phosphodiesterase will cleave cGMP to
5’-GMP.
 If you do not have any cGMP, Na channels will not
open and when Na channels are closed, the activity of
Na-K pump and K leak channels are both continuous
and the whole photoreceptor becomes
hyperpolarized. That’s why you would see that when
light strikes your photoreceptors, your photoreceptors
are not actually depolarized – they are hyperpolarized
 Light strikes Rods  Rhodopsin is converted to its
metabolic products: Light will bleach rhodopsin until it All-trans retinal and activated G-proteins can activate
becomes All-trans Retinal your phosphodiesterase because when you have your
active G-protein, your guanylyl cyclase is active and

Page 64 of 89 By: Ayesha Bea Federizo Batch 2017


GRAND PHYSIOLOGY B FINALS REVIEWER

your GTP is cleaved to GDP and cGMP  Each contains diff photopigments and are maximally
G-proteins have an inherent GTPase activity that will sensitive to one of 3 primary colors (RYB)
cleave GTP to GDP for G-proteins to be inactive again.  Cones responds to all colors but is maximally sensitive
GTPase wants to be inactive but G-proteins want to only to 1 color
be active  Sensation of any color is determined by the frequency
of impulse from each of cone system, the frequency of
SO IN SHORT, WHEN LIGHT STRIKES THE PHOTORECEPTORS, light waves that will strike the color cones
THEY ARE HYPERPOLARIZED  you do not release glutamate
and when glutamate is not released, horizontal cells are not Opponent Process Theory
stimulated and it will not be able to inhibit the bipolar cells so  Colored lights activates opposing neural process
it will be able to generate local potential until it summates for Green as to Red
it to stimulate ganglion cells. When you Stimulate green, you don’t Stimulate Red and vice versa
Yellow as to Blue
So pano Pag DARK? When you stimulate blue you don’t stimulate yellow and vice versa
Black as to white
But they are not really colors, so yeah whatever
Dark  Somehow depolarized but does not reach AP, local
potential only unless  they summate  reaches Action
Potential  releases NTA’s: Glutamate (present in bipolar cells Retinex Theory
as well as in horizontal cells)  Horizontal cells are stimulated  Attributes color vision to combined action of neural
activity at several neural levels of visual system which
to release inhibitory neurotransmitters and it goes to the
photoreceptor and inhibits the bipolar cells includes retina and visual cortex
 Cones will just be receptors  will send impulse to
In Ganglion cells, upon NTA release you produce visual cortex  Color is created and processed in Visual
cortex  you see colors
immediate Action Potential
 We have layers in the cerebral cortex – V1, V2, V3, V4, V5,
Bipolar cells local potential but it can summate V1V4a and V1V4B wherein the colors would fall.

Glutamate
COLORS
 Common NTA used
 Both Inhibitory and Stimulatory
Primary Secondary Complementary Color Cones
 Synapses in Center  Stimulatory
Colors Colors Colors
 Synapses in Round  Inhibitory
Red R+Y = Orange Red-Green Red
Yellow R+B= Violet Blue-Yellow Green
CONES: COLOR VISION Blue B+Y= Green Blue
 Wavelength range of visible light = 397nm-723nm, in
this wavelength you will be able to discriminate colors,
and Cones are responsible for this detection
 Photochemicals in cones almost have same chemical
composition with that of rhodopsin
 Difference is in Protein Portion  The opsin or
Photopsin
 Sees Black  No color cones are stimulated
 Sees White  All color cones are stimulated
 Damaged cones  Black and white vision  Interpretation:
 Orange You stimulate more of Red cones than Green
THEORIES OF COLOR VISION  Blue  You only stimulate blue cones
 Yellow Equal red and green cones
Trichromasy Theory  Red Only red cones
 The difference in absorption of light accounts for color  White  All color cones, equally
vision  Black  None
 Tricolor mechanism of color detection and this depends
upon absorption of lightwaves to recognize a certain Attributes of Color
color, and they are your:  Hue – Name of color. Eg: Red
 Intensity – Quality . Bright or dull? Eg: Bright Red
Primary colors (RYB)  Saturation – relative purity of color Eg: Fuchsia Pink
Red Longest wavelength
Yellow Intermediate
GANGLION CELLS
Blue Shortest

Primary Retinal or Color Cones (RGB) W X Y


Red Longest wavelength 570nm 40% 55% , most numerous 5%, least
Green Intermediate 535 nm numerous,
Blue Shortest 445 nm LARGEST
 In short, this theory states that there are cones sensitive to Directional Dendrites don’t For Light changes
certain lightwaves to be able to recognize the color movement: Left right spread widely For
front back small fields of vision,
Young Helmholts Theory Specific
 Postulates theory of 3 kinds of Cones ( Red, Green, Blue)

Page 65 of 89 By: Ayesha Bea Federizo Batch 2017


GRAND PHYSIOLOGY B FINALS REVIEWER

Crude rod or Gen Fine details of visual For rapid changes  Green blindness  problem in green-red and green-
vision under dark image. in visual Image blue combinations
conditions
Receives most Eg: nose, mouth etc Fires without
 Tritanope  blue blindness, almost the gray color is
excitation from accuracy
seen, may have few spots of blue
Rods Black and
white only Matching of Spectral Colors
Eg. Shapes Eg. Shadow,  for patients who can’t read or write
“Malikmata”  patient will find and match the colors.

COLOR BLINDNESS American Optical Instruments


 X linked recessive  uses shapes
o Due to abnormality in 1 or 3 cones Holgren’s Test
o Males are commonly affected Stilling’s Test
o Females are most likely carriers, but they can Rayleigh Match test
be affected too, provided that the dominant
X gene did not neutralize the recessive other LIGHT ADAPTATION
X gene Moment from dark going into bright light
Prefixes Prot Red Stimulates mainly Cones, little of Rods
Deuter Green
Tri Blue  Person is exposed to bright light for hours now 
Suffixes Anomalous/Anomaly Color  large portions of photochemicals in rods and cones is
reduced to retinal and opsins
weakness
 Retinal of both rods and cones is converted to Vit A
Anopia or anope Color
blindness
 Concentrations of photosensitive chemicals of rods
Eg: Protonomalous  Weakness in seeing shades of Red and cones remaining are significantly reduced
Protananopia  Cannot see red so you it is hard to  Sensitivity of eyes to light is reduced
distinguish violet from blue, orange tinted red appears as dim
yellows etc Other changes:
 Pupilloconstriction – to regulate light rays entering
DISORDERS OF THE COLOR VISION OF THE EYE pupil that will strike retina  for better vision
Trichromats  Utilization of central portion of retina ( fovea)
 All cones system are functioning, Normal RGB cones  Bleaching of rhodopsin
 Normal Colour Vision  Greater activation of cones

Dichromats
 2 cones system are functioning, but 1 is missing DARK ADAPTATION
You only stimulate your Rods bec of low light
Monochromats intensity
 Only 1 cone system is functioning, 2 are missing Remember: cones are only stimulated when there is
high intensity of light
Achromatopsia  Person is exposed to darkness for so long
 Total color blindness only, patient can still see Black and  Retinal and opsins in the rods and cones are
white converted back into the light sensitive
pigments
TEST FOR COLOR BLINDNESS  Vit A is converted back to retinal  give more
light sensitive pigments
Ishihara Color Blind Test
 most common test for color blindness  The limit is determined by amount of opsins in the
 uses numbers and lines, plates contg. figures made up rods and cones to combine with retinal
of colored spots on a background of similarly shaped Other changes:
colored spots  Pupillodilation – so all lights may enter
 Utilization of peripheral portion of retina (parafovea)
 Synthesis of Rhodopsin
 Greater rods activation

Visual Field
 Visual area seen by one of your eyes at a given instant
(monocular vision)
 The right eye sees something that the left eye cannot,
and vice versa
 Nasal field of Vision  area seen at nasal side
 Temporal field of vision  area seen at lateral side
 Barriers:
o Nose, medially
o High cheekbones, lower portion

Page 66 of 89 By: Ayesha Bea Federizo Batch 2017


GRAND PHYSIOLOGY B FINALS REVIEWER

o Thick eyebrows and Scaphocephaly (long Size Perspective


narrow head), upper portion
 This is why when you check for visual field, you tell
patient to stare at straight level  Smaller one is
farther, the bigger one
PERIMETRY is closer
 Systematic measurement of differential light sensitivity
in visual field by detection of presence of test targets in
a defined background
 Maps and quantifies visual field especially at extreme
periphery of visual field Distribution of Shadows and Illumination
 Tests all angles and measures peripheral vision
 Known as
Usage: Chiaroscuro
Placed on cheek  Illumination
One eye is covered, other eye looks and intensity is bright
in the mirror
 near
Patient should not move  If it is lighter 
Far

Motion Parallax
DEPTH PERCEPTION
 Assess Distance and movement of a particular object  If object
 3 cues: is near and
o Distance by sizes formed in retina moving  almost
o Distance by stereopsis ( binocular vision) all parts are
o Distance by moving parallax moving
Parallax- if you look at some nearby object and move your
 Object
head a little from side to side, the object looks like it is
moving back and forth. is far  you only
see few parts of
them moving
MONOCULAR CUES – FARFIELD DEPTH PERCEPTION
Familiar Size STEREOSCOPIC CUES- NEARFIELD DEPTH PERCEPTION
 You know the size of an object
 Pupils are normally 6cm or 2 in apart
 In a considerable distance, one may appear thin and
 Fixation point (Merriam-Webster) - the point in the
small but as you move closer it becomes bigger, that is
visual field that is fixated by the two eyes in normal
your familiar size
vision and for each eye is the point that directly
stimulates the fovea of the retina
Occlusion
 When you move
 The one seen wholly is the
the object closer 
nearer object
distance between images
 If only some parts are seen
formed in both retinas
 farther
will become farther
 When you move
it farther  distance
between images formed
in both retinas will
become closer
Linear perspective

 If near  Appears
Parallel
 As it goes farther  It
VISUAL PATHWAY
converges until only 1 line is
seen  All info coming from left visual field of both eye is
conveyed to the right side of brain and vice versa
 The Retina has Temporal and nasal side on both eyes
 Temporal side and nasal side  forms part of optic
nerve
 The Ipsilateral temporal side forms the optic tract, the
nasal side will cross over and joins the optic tract

Page 67 of 89 By: Ayesha Bea Federizo Batch 2017


GRAND PHYSIOLOGY B FINALS REVIEWER

 Damage/Lesion:  Images are formed in Retina


 Temporal side of retina conducts impulse on ipsilateral
primary visual area
 Nasal Side of retina conducts impulse going to
contralateral side of primary visual cortex ( BA # 17)

Cuneus Vs Lingual Gyrus


Cuneus Lingual Gyrus
Location Superior Calcarine Inferior Calcarine
Area Area
Projection in Projected inferiorly Projected
visual field Superiorly
(remember the
principle of
inversion)
Lesion Inferior Superior
Quadrantanopsia Quadrantanopsia
 If you damage the right side of Cuneus  left inferior
quadrantanopsia
 Damage of most of the portion of calcarine area
except the most posterior aspect (occipital lobe) 
Macular Sparing  sees small pinpoint
 Both Cuneus and Lingual is affected  Homonymous
Quadrantanopsia

Pathways
Magnocelluar Parvocellular
Conduction speed Rapid Slow
Receives input Large Type Y Type X Retinal
from retinal ganglion ganglion cells
cells
Transmits Black and white Color and conveys
info accurate point to
point or detailed
infos

DIE VISION. DIEEEE!

From Retina  Optic Nerve  Optic chiasm  optic tract


superior colliculus  lateral geniculate body  optic
radiation or geniculo calcarine tract  primary visual center or
commonly known as the calcarine area.

Page 68 of 89 By: Ayesha Bea Federizo Batch 2017


GRAND PHYSIOLOGY B FINALS REVIEWER

long-chain organic substances that



TASTE AND SMELL PHYSIOLOGY contain nitrogen
 allow us to separate undesirable or even lethal foods from  alkaloids. such as quinine, caffeine,
those that are pleasant to eat and nutritious.
strychnine, and nicotine.
 elicit physiological responses that are involved in digestion
and utilization of foods. o In high intensity, usually causes the person
 strongly tied to primitive emotional and behavioral functions or animal to reject the food. Important for
of our nervous systems. poisonous substance rejection
 The sense of smell also allows animals to recognize the  Saccharine  sweet taste + Bitter aftertaste
proximity of other animals or even individuals among
animals.  Umami
Reference: Guyton and Hall 12th Edition + Handouts o Japanese word meaning delicious
TASTE PHYSIOLOGY o the dominant taste of food
o containing l-glutamate,
TASTE o the precise molecular mechanisms
responsible for umami taste are still unclear.
 mainly a function
of the taste buds in the
 A person can perceive hundreds of different tastes.
mouth,
 Slight changes in the chemical structure, such as addition of
 one's sense of a simple radical, can often change the substance from sweet
smell also contributes to bitter
strongly to taste
perception. THRESHOLD FOR TASTE
 it allows a person
to select food in accord
with desires and often in
accord with the body
tissues' metabolic need for
specific substances.
 Threshold minimum at 20-30 degrees Celcius
 the texture of food, as detected by tactual senses of  Varies with temp of solution
the mouth, and the presence of substances in the  LOWEST THRESHOLD IS BITTER  protective function
food that stimulate pain endings, such as pepper,
greatly alter the taste experience TASTE BLINDNESS
 Occurs especially for different types of thiourea
PRIMARY SENSATIONS OF TASTE compounds.
 Phenylthiocarbamide
o substance used frequently by psychologists
for demonstrating taste blindness
o 15 to 30 percent of all people exhibit taste
blindness
TASTE BUDS
 diameter = 1/30 millimeter
length =1/16 millimeter.
 sour  composed of about 50 modified epithelial cells,
o caused by acids,  hydrogen ion o sustentacular cells
concentration, o taste cells
o more acidic the food, the stronger the sour  Taste pores – has taste hairs as receptor
sensation becomes. surface for taste.
 Salty  continually being replaced by mitotic division of
o elicited by ionized salts, mainly by the surrounding epithelial cells,
sodium ion concentration. o young cells
o sodium cations, are mainly o mature cells lie toward the center of the bud
o anions also contribute to a lesser extent  soon break up and dissolve.
 sweet o The life span of each taste cell is about 10
o not caused by any single class of chemicals. days in lower mammals but is unknown for
o Some of the types of chemicals that cause humans.
this taste include sugars, glycols, alcohols,  The outer tips of the taste cells are arranged around a
aldehydes, ketones, amides, esters, some minute taste pore
amino acids, some small proteins, sulfonic  Microvilli
acids, halogenated acids, and inorganic salts o From the tip of each taste cell
of lead and beryllium. o Aka taste hairs that protrude outward into
 Bitter the taste pore to approach the cavity of the
o Also not caused by any single type of mouth.
chemical agent. o provide the receptor surface for taste.
o almost entirely organic substances.
o Two particular classes of substances

Page 69 of 89 By: Ayesha Bea Federizo Batch 2017


GRAND PHYSIOLOGY B FINALS REVIEWER

 Vesicles Sweet and bitter taste sensations the portions of the


o contain a neurotransmitter substance that is receptor protein molecules that protrude through the apical
released through the cell membrane to membranes activate second-messenger transmitter
excite the nerve fiber endings in response to substances inside the taste cells, and these second messengers
taste stimulation. cause intracellular chemical changes that elicit the taste
 Stimulation of Taste Buds  Substances must be in signals.
solution or dissolved in saliva
Generation of Nerve Impulses by the Taste Bud
Location of Taste Buds  First application of the taste stimulus the rate of
 Circumvalate papillae discharge of the nerve fibers from taste buds rises to
o Forms a V-line posterior of the tongue a peak in a small fraction of a second  then adapts
o Large number of taste buds within the next few seconds back to a lower, steady
 Fungiform papillae level as long as the taste stimulus remains.
o Distributed all over, Moderate numbers of  Strong immediate signal is transmitted by the taste
taste buds nerve, and a weaker continuous signal is transmitted
o over the flat anterior surface of the tongue as long as the taste bud is exposed to the taste
 Foliate papillae stimulus.
o Postero-lateral surface of the tongue TRANSMISSION OF TASTE SIGNALS INTO CNS
o Moderate Numbers  Taste impulses from the anterior 2/3 of the tongue
pass first into the lingual nerve  chorda tympani 
 Additional taste buds are located on the palate, and a
few are found on the tonsillar pillars, on the epiglottis, facial nerve tractus solitarius in the brain stem.
and even in the proximal esophagus.  Taste sensations from the Posterior 1/3 of tongue,
circumvallate papillae on the back of the tongue and
 Adults have 3000 to 10,000 taste buds, and children
from other posterior regions of the mouth and throat
have a few more.
are transmitted through the glossopharyngeal nerve
 Beyond the age of 45 years  many taste buds
degenerate causing taste sensitivity to decrease in  tractus solitarius, but at a slightly more posterior
old age. level.
 Base of the tongue and other parts of the pharyngeal
Specifity region by way of the vagus nerve tractus solitarius f
 taste substance is in low concentration  single taste .
responds mostly to one of the five primary taste All taste fibers synapse in the posterior brain stem in
stimuli the nuclei of the tractus solitaries  send second-
 high concentration  most buds can be excited by order neurons to a small area of the ventral posterior
two or more of the primary taste stimuli, as well as by medial nucleus of the thalamus  third-order
a few other taste stimuli that do not fit into the neurons are transmitted to the lower tip of the
"primary" categories postcentral gyrus in the parietal cerebral cortex  it
curls deep into the sylvian fissure  adjacent
MECHANISM OF STIMULATION OF TASTE BUDS opercular insular area
Receptor Potential
 change in electrical potential in the taste cell  The taste pathways closely parallel the somatosensory
 membrane of the taste cell,is negatively charged on pathways from the tongue.
the inside with respect to the outside
 Application of a taste substance to the hairs causes Taste Reflexes Are Integrated in the Brain Stem
them to be depolarized it is proportional to the o From the tractus solitarius,  superior and
logarithm of concentration of the stimulating
inferior salivatory submandibular,
substance.
sublingual, and parotid glands  help
Mechanism = binding of the taste chemical to a protein control the secretion of saliva during the
receptor molecule that lies on the outer surface of the taste ingestion and digestion of food.
receptor cell near to or protruding through a villus membrane
 opens ion channels allows positively charged sodium ions ADAPTATION AND TASTE PREFERENCE
or hydrogen ions to enter and depolarize the normal negativity  Taste sensations adapt rapidly, often almost
of the cell taste chemical itself is gradually washed away completely within a minute or so of continuous
from the taste villus by the saliva, which removes the stimulus. stimulation.
 Final extreme degree of adaptation that occurs in
Type of receptor protein central nervous system itself, although the mechanism
 Sodium ions elicit salty sensations and site of this are not known.
 Hydrogen ions sour taste sensations  adapt almost entirely at the receptors.

For Salty and Sour  the receptor proteins open specific ion Taste preference
channels in the apical membranes of the taste cells, thereby  simply means that an animal will choose certain types
activating the receptors. of food in preference to others, and the animal
automatically uses this to help control the diet it eats
 often change in accord with the body's need for
certain specific substances.

Page 70 of 89 By: Ayesha Bea Federizo Batch 2017


GRAND PHYSIOLOGY B FINALS REVIEWER

 results from some mechanism located in the central activating olfactory nerves greatly multiplies the
nervous system  previous experience with excitatory effect of even the weakest odorant.
unpleasant or pleasant tastes plays a major role in even the most minute concentration of a specific
determining one's taste preferences or taste aversions odorant initiates a cascading effect that opens
extremely large numbers of sodium channels. This
 Taste receptors often become sensitized in favor of a accounts for the exquisite sensitivity of the olfactory
needed nutrient. neurons to even the slightest amount of odorant.

SMELL PHYSIOLOGY
PHYSICAL FACTORS THAT AFFECT THE DEGREE OF
 least understood among senses.
STIMULATION
 Sense of smell is a subjective phenomenon that
 only volatile substances that can be sniffed into the
cannot be studied with ease in lower animals
nostrils can be smelled.
 poorly developed in human beings
 stimulating substance must be at least slightly water
soluble so that it can pass through the mucus to reach
the olfactory cilia
 it is helpful for the substance to be at least slightly
lipid soluble, presumably because lipid constituents of
the cilium itself are a weak barrier to non-lipid-
soluble odorants.

MEMBRANE POTENTIALS AND ACTION POTENTIALS IN


OLFACTORY MEMBRANE OLFACTORY CELLS
 lies in the superior part of each nostril.
 Medially folds downward along the surface of the  Membrane potential inside unstimulated olfactory
superior septum cells = -55 millivolts. At this potential, most of the
 laterally, it folds over the superior turbinate and even cells generate
over a small portion of the upper surface of the o once every 20 seconds up to two or three
middle turbinate per second of continuous action potential
 In each nostril, has a surface area of about 2.4 square
centimeters.  depolarization of the olfactory cell membrane = -30
millivolts or less
OLFACTORY CELLS o action potentials increases to 20 to 30 per
 receptor cells for the smell sensation second, which is a high rate for the minute
 are bipolar nerve cells derived originally from the olfactory nerve fibers.
central nervous system itself. the rate of olfactory nerve impulses changes
 about 100 million of these cells in the olfactory approximately in proportion to the logarithm of the
epithelium interspersed among sustentacular cells, stimulus strength
 The mucosal end of the olfactory cell forms a knob RAPID ADAPTATION OF OLFACTORY SENSATIONS
from which 4 to 25 olfactory hairs (also called  Olfactory receptors
olfactory cilia), adapt about 50 percent in the
o react to odors in the air and stimulate the first second or so after
olfactory cells stimulation they adapt very
o 0.3 micrometerin diameter little and very slowly
o 200 micrometers in length  Most of the additional
o project into the mucus that coats the inner adaptation occurs within the
surface of the nasal cavity form a dense central nervous system. Same
mat in the mucus with taste.
Bowman's glands
 Secrete mucus onto the surface of the olfactory
membrane. Postulated neuronal mechanism for the adaptation is the
 Spaced among the olfactory cells in the olfactory following:
membrane  Large numbers of centrifugal nerve fibers pass from
the olfactory regions of the brain backward along the
STIMULATION OF OLFACTORY CELLS olfactory tract and terminate on special inhibitory
Olfactory Signal Transduction= Binding of the odorant to a cells in the olfactory bulb, the granule cells.
G-coupled protein receptor  activation of adenylate cyclise  after the onset of an olfactory stimulus, CNS quickly
 converts ATP to cAMP cAMP activates a gated sodium develops strong feedback inhibition to suppress relay
channel that increases sodium influx and depolarizes the cell  of the smell signals through the olfactory bulb.
exciting the olfactory neuron  transmitting action potentials
to the central nervous system  Granule Cells
 Olfactory cilia o special inhibitory cells in the olfactory bulb
o portion of each olfactory cell that responds o Means of sharpening specific ability to
to the olfactory chemical stimuli distinguish odor
 Psychological adaptation > receptor adaptation

Page 71 of 89 By: Ayesha Bea Federizo Batch 2017


GRAND PHYSIOLOGY B FINALS REVIEWER

AFFECTIVE NATURE OF SMELL to the hypothalamus.


 Smell, even more so than taste, has the affective
quality of either pleasantness or unpleasantness Septal nuclei
smell is probably even more important than taste for  are midline nuclei that feed into the hypothalamus
the selection of food. and other primitive portions of the brain's limbic
 In addition, in some lower animals, odors are the system.
primary excitant of sexual drive.
This is the brain area most concerned with basic
THRESHOLD FOR SMELL behaviour, basic olfactory reflex
 Very Low The Less Old Olfactory System
methylmercaptan can be smelled when only one 25  The Lateral Olfactory Area
trillionth of a gram is present in each milliliter of air   composed mainly of the prepyriform and pyriform
mixed with natural gas to give the gas an odor that cortex plus the cortical portion of the amygdaloid
can be detected when even small amounts of gas leak nuclei
from a pipeline.  Learned Control
 Concentrations only 10 to 50 times above the
threshold evoke maximum intensity of smell. Hippocampus
 smell is concerned more with detecting the presence  most important for learning to like or dislike certain
or absence of odors rather than with quantitative foods depending on one's experiences with them.
detection of their intensities
many signal pathways from this area also feed directly
TRANSMISSION OF SMELL SIGNALS INTO CNS into an older part of the cerebral cortex called the
Limbic System part of the brain that originally subserved paleocortex in the anteromedial portion of the
olfaction later evolved into the basal brain structures that temporal lobe
control emotions and other aspects of human behavior only area of the entire cerebral cortex where sensory
signals pass directly to the cortex without passing first
Olfactory bulb through the thalamus
 Olfactory signals are transmiited into automatic but partially learned control of food intake
 lies over the cribriform plate, separating the brain and aversion to toxic and unhealthy foods
cavity from the upper reaches of the nasal cavity. The Newer Pathway
 The cribriform plate has multiple small perforations  passes through the thalamus dorsomedial
through which an equal number of small nerves pass thalamic nucleus  lateroposterior quadrant of the
upward from the olfactory membrane in the nasal orbitofrontale cortex
cavity to enter the olfactory bulb in the cranial cavity
There is close relation between the olfactory cells in the conscious perception and analysis of olfaction
olfactory membrane and the olfactory bulb:
o short axons from the olfactory cells terminating in
multiple globular structures within the olfactory bulb
called glomeruli.
Each bulb has several thousand such glomeruli, each of which is
the terminus for about 25,000 axons from olfactory cells.
Each glomerulus also is the terminus for dendrites from about 25
large mitral cells and about 60 smaller tufted cells, the cell
bodies of which lie in the olfactory bulb superior to the
glomeruli.
These dendrites receive synapses from the olfactory cell neurons,
and the mitral and tufted cells send axons through the olfactory
tract to transmit olfactory signals to higher levels in the CNS
 Different glomeruli respond to different odors. It is possible
that specific glomeruli are the real clue to the analysis of
different odor signals transmitted into the central nervous
system.

The Very Old, Less Old, and Newer Olfactory Pathways


 The olfactory tract enters the brain at the anterior
junction between the mesencephalon and cerebrum;
there, the tract divides into two pathways,
one passing medially into the medial olfactory area of
the brain stem very old olfactory system
the other passing laterally into the lateral olfactory
area input to
(1) a less old olfactory system
(2) a newer system

Very Old Olfactory System


 The Medial Olfactory Area
 area consists of a group of nuclei located in the
midbasal portions of the brain immediately anterior

Page 72 of 89 By: Ayesha Bea Federizo Batch 2017


GRAND PHYSIOLOGY B FINALS REVIEWER

EQUILIBRIUM o Predictive function : Signal beginning or end of


Source: Dr Barbon’s Lecture, Lecture guide, Icelimpin notes head rotation, and predicts incoming
 Ability to maintain posture or balance malequilibrium
 Depends on receptors of inner ear  Vestibular apparatus
 Receptor organ in the Semicircular canal is the Crista Ampullaris,
 Involves activity of: which is Located in the Ampulla
1. Proprieceptors Activation of the Canal
2. Visual receptors - weak  Hair cells is embedded at the base of cupula
3. Exteroreceptors 1. Head rotates  Movement of the canal  displacement
of endolymph inside the canal  Cupula activation
Normal Body Position 2. Movement of the cupula  activation of the hair cells
 Concerned more with head (receptors)
 Upright head, thorax and extremities 3. Receptor or hair cells activity is transferred to afferent
 Attributed to stimulation of extensor muscles of extremities, nerve or the vestibular nerve aka utriculosaccular nerve
back and neck  antagonizes gravity, these are your impulse transmitted to center
postural or antigravity muscles
 Displacement of endolymph  Movement of cupula  hair
 Functional unit of nervous system reflex arc cells activity is change transmitted to vestibular nerve 
VESTIBULAR APPARATUS center
 The receptors of the canal, utricle and saccule & cochlea is
 Receptors of the inner ear
the same = hair cells
 Present in temporal bone, 2 sets  Left and Right
 Mediated by CN 8 or Vestibulocochlear nerve specifically
UTRICLE AND SACCULE
Vestibular portion
 Connected to Semicircular canal, also for maintenance of
1. (+) branches for both vestibular apparatus and cochlear
balance
structures
 Also have endolymph, hair cells and gelatinous mass , No
 Neuronal Acitivity associated with Audition
cupula (walang name yung gelatinous mass nya)
 Made up of:
 Contains the Otoconia / Otoliths / Statoconia / Ear dust
1. Semicircular canal
o Considered as gravity receptors
2. Utricle and Saccule
o Mass of calcium carbonate crystals embedded in
SEMICIRCULAR CANAL gelatinous substance or layer
 Hair cells are also embedded in the gelatinous
substance
 Receptor organ in Utricle and Saccule = MACULA
Macula
 For Static Equilibrium: determination of head or body
position when body is not moving or with respect to
direction of gravitational pull
 For linear movement or acceleration
o Suddent thrust forward of the body  otoconia is
 Arranged at right angles to each other, representing 3 planes thrown backward d/t inertia
in space when the head is bent forward around 30 degrees o Backward  Otoconia is thrown forward d/t inertia
 Composed of 3 paired ducts/canal = total of 6  For detection of Centrifugal forces(a form of inertia)
1. Right and Left Anterior or Superior Vertical tendency of an object following a curved path to fly way
 Located in vertical plane, projects from center of curvature  so we correct this by leaning
forward and 45 degrees outward towards the center
2. Right and Left Posterior Vertical  Macula is never concerned with linear velocity
 Located in vertical plane, projects
backward and 45 degrees outward
Orientation of Utricle and Saccule
3. Right and Left Horizontal  Approximately Horizontal,
o Horizontal: Macula of utricle
located in same plane, so it will meet the horizontal  Upright position  Utricle dependent (upo)
plane when you bend the neck 30 degrees || if neck is
o Vertical: Macula of Saccule
straight  canal is Semi horizontal  Lying position  Saccule dependent (sahig)
 Identical or Parallel or on same plane:
o Righ Anterior and Left Posterior Upright Lying
o Left Anterior and Right Posterior
Maccula of utricle is Horizontal in Maccula of saccule is Horizontal
o Right and Left Horizontal
position in position
 PURPOSE: DETERMINE DIRECTION OF MOVEMENT
Maccula of saccule is vertical Maccula of utricle is vertical
 These canals contain Endolymph Utricle is Stimulated Saccule is Stimulated
 Receptor organ in Semicircular canal  Crista Ampullaris Saccule is Inhibited Utricle is Inhibited
Crista Ampullaris  Motion Sickness  Problem is in Utricle
 WHOLE SENSORY ORGAN OF SEMICIRCULAR CANAL Management: Lie down, so Saccule is the one stimulated 
 Located in the ampulla, the dilated or swollen end of the dizziness disappears or lessens
canal
 Made up of hair cells (located at the base of the cupula, Activation of Utricle and saccule
attached to vestibular nerve) with Stereocilia and a large  Activity of hair cells is dependent on otoconia displacement
Kinocillium covered by a gelatinous mass called Cupula caused by gravity
 Cupula is where the hair tufts from hair cells project  Otoconia is made up of crystal  Heavier and denser than
 Cupula is considered as the whole structure in the gelatinous mass, and is mostly affected by gravity
ampulla that represents the side of the canal
separating the other side Static:
 For Dynamic or Kinetic Equilibrium for body in motion: 1. Otoconia is pulled down by gravity  gelatinous mass is
spinning or rotator movement compressed  haircells embedded on it are displaced

Page 73 of 89 By: Ayesha Bea Federizo Batch 2017


GRAND PHYSIOLOGY B FINALS REVIEWER

2. Displaced hair cells  impulse goes to Center (CNS)  Movement is dependent on hair cells activity
 Additional movement in macula if there is linear motion  This is where the Kinocilium of different hair cells are directed
inorder to determine forward, backward or sideward
movement LINEAR ACCELERATION
 Forward, Backward or sideward movement
 Receptor organ in Semicircular canal = Crista Ampullaris  For  Activation of Utricle is due to effect of inertia, which would
dynamic or kinetic equilibrium cause movement of the hair cells
 Receptor organ in Utricle and Saccule = MACULA  for static
equilibrium, linear movement and centrifugal forces  Statoconia or Otoconia
HAIR CELLS o Gelatinous mass, which is heavier than hair cells
 2 types: Kinocilium and Stereocilia because of the calcium carbonate crystals
Kinocilium Stereocilia o Due to Inertia:
Tall and large diameter Thin and shorter  Movement forward  statoconia or
Movement of both hair cells are important for determination their activity otoconia moves backward, opposite
 Movement Backward  Statoconia or
otoconia moves forward
Activation of Hair Cells  Isipin nyo na lang masyadong mabigat yung
 RMP = -16mv otoconia kaya naiiwan sya sa opposite direction
 Ion channels activated = K and Ca channels
 NTA= Glutamate  Striolar Groups or Division
Depolarized/Activated Hyperpolarized/Inactivated o Forward movement
Movement of stereocilia is toward Movement is away from  Anterior group of hair cells are activated
the Kinocilium kinocilium or toward stereocilia  Posterior group of hair cells are inactivated
Stereocilia is pushed toward Stereocilia is pushed away from
Kinocilium Kinocilium o Lateral movement (eg. Towards left)
 lateral group of the left utricle and medial
Kinocilium moves away from Stereocilia moves away from
group of right utricle is activated
stereocilia kinocilium
o Lateral movement (eg. Towards right)
 Lateral group of right utricle and medial
Impulse Generation of Hair cells in Vestibular Apparatus: Saccule, group of left utricle is activated
Utricle and Semicircular canal
 Resting Activity ofHair cells or tonic activity (not moving)  Utricle is mainly responsible for detection of movement when
o generates 100-200 impulses/sec negotiating a curve (In centrifugal forces), when head is upright.
o this tonic activity is helpful to determine position
of head SEMICIRCULAR CANAL PART 2
 No impulses generating  Patient is asleep and doesn’t  Again, this is where the Crista Ampullaris is so it is for
know head position  Dynamic or Kinetic equilibrium
 (+) sideward, forward, backward movement of head or body o Detection of angular or rotator
 changes in positioning of macula  Increased activity of acceleration
hair cells, stereocilia moves toward kinocilium   Predictive function associated with cerebellar
depolarization of hair cells activity which is for coordination and muscle tone
in preparation for movement
UTRICLE & SACCULE PART 2 o This predictive functions is associated
 This is where the macula is so it is for most particularly with Archicerebellum
 Static equilibrium or vestibulocerebellum mostly of the
 Head position determination posterior part
 Space orientation  Again receptors are present in the Crista ampullaris, its
 Linear movement or acceleration determination activation is due to rotation of the body: the displacement of
 Detection of centrifugal forces endolymph  causes movement of cupula and this
Saccule Activation Utricle Activation displacement will cause hair cells to move
Vertical Acceleration  Up and down Horizontal acceleration  Forward,
backward, movement toward a circle Movement of Endolymph
either clockwise, counterclockwise  Movement is Clockwise rotation  Endolymph moves
(Centrifugal force)
opposite side or counterclockwise initially d/t inertia
Eg. Going up and down in elevator etc Eg. Walking towards a circle, walking
forward etc
 Movement of the body  Endolymph inside also moves, in
Lying Position, also for upright Upright Position opposite direction
position only in determining up or  To determine the spinning movement of the head / body speed
down movements rotation should be equal or greater than 1°/sec2
If < 1°/sec2 you will not determine any movement

Gravitational forces exerted by Otoliths/Otoconia/Statoconia  Kinocilium is oriented towards the center


 Otoliths are your gravitational receptors in Utricle and There is tonic activity when resting  100-200 impulses/sec
saccule
 Head at rest : 100-200 impulses/sec ROTATION
 Tilts toward Kinocilium  Depolarization Head Rotation to the Right
 Tilts toward stereocilia  Hyperpolarization  Semicircular Canal moves toward the right
o because it is fixed in the inner ear
STRIOLA o canal follows movement of the head
 Central, depressed portion
 Endolymph moves toward opposite side
in the otoconia or otoliths
o d/t inertia
 Serves as a reference point
to determine divisions of group of hair  Increased activity in the right Crista  stereocilia moves
cells in macula, which includes: toward kinocilium  Depolarization  Increase Action
Anterior, Posterior, Lateral potential
and medial group  Decreased activity or inbited activity in Left Crista

Page 74 of 89 By: Ayesha Bea Federizo Batch 2017


GRAND PHYSIOLOGY B FINALS REVIEWER

o On the left side: Kinocilium moves toward o Rotation to the left in horizontal plane  Past
stereocilia pointing to the right
 We know the direction of the rotation because there is
variation in the activity of left and right Crista  Tendency to fall
o A false sensation, you fall because you try to
Head Rotation Continues correct the balance
 Like for Ballet dancers  o Tendency to fall and vertiginous sensation are
 If you can talk to rotating person rotating person will not opposite in direction, like in past pointing
be able to determine the direction of the rotation
 Comtinuous rotation  eventually, endolymph will follow  Autonomic sensations
the rotation of the canal  hair cells in the crista will assume o Nausea and vomiting
again a normal resting position so Direction of rotation is o Pallor
only determined at the start of rotation o Sweating
 Right and left crista is equally activated o Vasoconstriction  pale
o Changes in BP, HR, RR
Head Rotation Ends o Miosis during rotation
 When you stop d/t inertia Endolymph continues to flow o Mydriasis after rotation
towards the right  there is continuous sensation of rotation o Dizziness- like vertigo but environment doesn’t
and this will cause Post rotatory sensation spin
 Sensation of the direction of rotation is towards the left this TEST MADE TO DETERMINE INTEGRITY OF CANAL
time, d/t movement of endolymph Remember: Opposite Opposite, same same
o Left Crista is activated *Sa lecture guide kasi Opposite din yung sa Tendency to fall at PP, pero siguro
o Right Crista is inactivated initially lang yun, e yung case naman dito Post rotation na, yun siguro yun? Haha
 After a while, Crista ampullaris or the canals will assume its ewan.
normal resting position

POST ROTATORY SENSATION/REACTIONS


Effects produced by rotation
 Vertigo
o Sensation that environment is rotating, feeling of
unsteadiness and of actual disturbance of
equilibrium  If you rotate in Horizontal plane  stimulate Horizontal canals
o d/t excitation of semicircular canals or their central
connections

o Horizontal plane: Post rotator vertiginous


sensation is in opposite direction of rotation
o Sagittal plane: Post rotator vertiginous sensation
is falling backward or forward
o Frontal plane: Post rotator vertiginous sensation is  If you rotate in Sagittal plane  You stimulate all canals
falling to right or left side

 Nystagmus/Eye movements
o Rhythmical slow eye deviation to one side and a
quick return to normal, forward looking position
o 3 types:
 Horizontal- due to stimulation of
horizontal canals  so when turning in  If you rotate in Frontal or coronal plane  You stimulate all vertical canals
an upright position of the head in
horizontal plane PARTS OF NERVOUS SYSTEM ESSENTIAL FOR EQUILIBRIUM
 Vertical- eyes move up and down; d/t  Spinal cord
stimulation of all canals, rotation on o Neck reflexes
sagittal plane tumbling o Motor neurons at anterior horn regulates activity
 Rotatory- round about movement of the of the muscles
eyes in antero-posterior axis. d/t  Brainstem
rotation of frontal plane and stimulation o Subconscious maintenance of posture
of vertical canals  body attached to o Midbrain  for righting reflex/correcting reflex
wheel (with involvement of photoreceptors)  allows
 At the beginning of rotation  nystagmus would be on same body to have correct position
direction as rotation
o Pons and medulla maintain balance
 After rotation  nystagmus would be on opposite direction as
rotation
o Reticular system of pons keeps us awake
 Whatever the direction of the fast or quick or phase/ component is, o Dec activity of pons  muscles that maintain
that is the direction of nystagmus postures weaken (observerd when asleep)
o Controlled by the Higher center  brainstem  Righting reflex is the only reflex not located
 Used for reading in cerebral cortex
 Maintain visual fication on stationary point  Problems in the eye imbalance
while body rotates
 Slow phase/component  All righting reflex is dependent on Midbrain
o Dictated by the vestibular apparatus Labyrinth except for optical righting reflex (uses
occipital cortex)
 Past Pointing (PP)  Cerebral cortex
o An involuntary act, a subconscious correction for o Conscious maintenance of posture
the impression that there is continuous rotation
immediately after stopping the rotation  Cerebellum- Unconscious maintenance of psoture
o Vestibulocerebellum/Archicerebellum

Page 75 of 89 By: Ayesha Bea Federizo Batch 2017


GRAND PHYSIOLOGY B FINALS REVIEWER

 Flocculonodular lobe, mostly posterior


 Posture, gait and equilibrium
maintenance, when not moving (static
equilibrium)

 Prediction of movements, to prepare


body for rapid movements

o Paleocerebellum/Spinocerebellum
 Greater connection with Spinal cord
 Control of muscle activity
 Maintain equilibrium while in motion
(Dynamic Equilibrium)
 Motor coordination
 Located in anterior lobe, mostly in
medial portion

o Neocerebellum/Cerebrocerebellum
 Posterior lobe, mostly lateral
 Control and coordination of voluntary
and learned movements
 For motor learning
o Athletes have well
coordinated muscle

 Problems in inner ear, cerebellum, spinal cord, brainstem and cerebral


cortex will lead to Malequilibrium
 For Fine adjustment of posture  Brainstem, Cerebral cortex and
Cerebellum

Anti Gravity muscles


 Muscles (mostly extensors) required to maintain posture,
includes leg muscles when standing
 Arm extensors  pro gravity
 Flexors are anti gravity but is not used for posture
maintenance well unless you’re a bird
 Damage on structures  inability to maintain posture

Posture regulating mechanisms


 Maintains upright and balanced position and provides
constant adjustments needed to maintain a stable postural
background for voluntary activity
 In whatever we do we maintain upright position
of the head, except when sleeping

VESTIBULO-OCULAR REFLEX or DOLL’s EYE REFLEX


 (+) movement of head (+) movement of endolymph (+)
Attitudinal Reflex = Inborn
action of vestibular apparatus  Crista sends impulses to
 Tonic Labyrinthine reflex
Vestibular nuclei  activates abducens, trochlear and
No selection, pag extend extension lahat etc
occulomotor nerve Effect: Extraocular muscles
Mostly involved in animals, little in humans
 At start of rotation Abducens activates opposite side of
eyeball || Occulomotor activates ipsilateral side
 Rotation towards right, tendency is you move eyeball to the
left

Corticobulbar tract
 Sets of neuroncontrolling muscles in cephalic area including the neck
 includes all CN except (also 1 and 2) CN 3, 4, 6  Tonic neck reflex

PRINCIPAL POSTURAL REFLEXES


 To maintain equilibrium

OTHER SENSORY RECEPTORS FOR EQUILIBRIUM


 Proprioceptive Receptors
o Golgi tendon Organs
o Pacinian corpuscles
o Muscle spindles
o Joint receptors

Page 76 of 89 By: Ayesha Bea Federizo Batch 2017


GRAND PHYSIOLOGY B FINALS REVIEWER

o Eletrical stimulation on mastoid (painful) 


 Eyes/Vision activates labyrinth in temporal region
o Essential for balance maintenance if there is o OBSOLETE
abnormality in labyrinth or brainstem and
proprieceptors, if none, one will be able to  Posturography
maintain balance even in the absence of vision o Platform moves on all sides  vestibular and
o For us to see the relation in the environment, but retinal nystagmus is observed
this is slow, and will not help in performing rapid o Movement of extraocular muscles ((+) sensors) to
movements determine movement
o So damage in vision pero intact yung ibang o Computer records change of muscle tone
receptors  slow movements
EQUILIBRIUM DISORDERS
 Exteroreceptors  Kinetosis/Motion Sickness
o Detects wind on the body o Most common problem
o Usually utricle problem
 Vestibular nucleus o Management: lie down / tilt or bend head forward
o Has superior, lateral, medial and inferior regions when in a car
o Lateral  receives impulses from utriclefor static
equilibrium  Meniere’s diseas/Endolympatic Hydrops
o Superior and medial  receives impulses from o Excessive production of endolymph
semicircular canal for dynamic equilibrium o Whole vestibular app is the problem including the
o Inferior  receives impulses from all structures in cochlea
Vestibular App  Lesion will lead to static and o Problems include Malequilibrium  Vertigo
dynamic equilibrium problems
o Hearing  tinnitus and Sensory neural hearing
o Lateral and Medial utricle and semicircular  so
for dynamic and static  for trunk and neck loss, most specifically low frequency deafness
muscle  maintain normal posture of head and (problem in Apex)
body (vestibulocollic refelex) o (+) Nystagmus
o All connected to vestibular apparatus and to
different parts of the nervous system
Tinnitus Nystagmus
Ampulla is bulged Even without doing caloric
TEST FOR EQUILIBRIUM
You hear sounds even when test
 Positional testing it is absent ringing Slow component always
o Tests integrity of Proprieceptors sensation moves toward the direction
o Hand (proprieceptors of dorsal cuneatus) d/t Excessive endolymph of the abnormality
that can activate organ of Fast component moves away
 Romberg’s Test corti from abnormality
o Test for proprioceptors and vestibular apparatus
o Stand upright, feet together close eyes  Sa samplex, included ang Past pointing for meniere’s
o Observe for swaying movements disease, so is it safe to assume that they can also experience
other post rotatory sensations such as Tendency to fall and
 Barany’s Test etc? Makes sense naman diba?
o Test for Vestibular apparatus
o Utilize a swivelling or rotating chair to detect
Guys,till now, yung nakahighlight ng ganito di pa din clear,
responses from the labyrinth (vestibular apparatus)
sabe sa recordings sa isa, Neo, sabe dun sa isa Paleo, sabe sa
o Look at post rotator sensation
previous trans archi. Huhu I don’t knowww.
o Usually upright position

 Caloric Stimulation
o Cheapest
o Test for the semicircular canals
o Instill warm or cold water in external ear 
changes in activity of labyrinth
o Effect of convection current
o Change in temp will affect movement of
endolymph with consequent motion of cupula
Normal:
Warm water= 40-41 C
 Nystagmus towards stimulated ear (fast component)
Cold water= 18-19 C
 Nystagmus towards unstimulated ear(fast component)

 Abnormal if no movement is elicited or no nystagmus is


seen
 Remember this when irrigating an infected ear to avoid
development of nystagmus or vertigo use water with
same temp as body
o Patient is only malingering if he states that he is
experiencing vertigo
o Before doing this test, check ear first for any
problems like serumen blockage, perforations etc.
Bec nothing will be stimulated if there is any

 Galvanic stimulation
o Test for semicircular canals

Page 77 of 89 By: Ayesha Bea Federizo Batch 2017


GRAND PHYSIOLOGY B FINALS REVIEWER

MOTOR PHYSIOLOGY 1 o Loc: Precentral Gyrus of Frontal lobe


o Commands skeletal muscles to contract
Sources: Dr. Vila’s Lecture, Ganong’s Lecture guide, Icelimpin and Bernabe Trans
o Somatotopic map or Body Part Representation in
pre central gyrusMOTOR HOMUNCULUS
SOMATIC MOTOR SYTEM
 Under Peripheral Nervous system Motor Homunculus
 Cortical association area found in parietal Lobe   No Specificity, Entire face and hand is mainly represented
Connection with frontal lobe (Motor function)  Effect in  Muscles of the face: Brachiomeric in origin, meaning they
Skeletal muscle (Voluntary motor system) are small muscles that do fine movements
Eg. Palakihan ng butas ng ilong
 Law of Plasticity / Neural Plasticity  The more you use it, the greater
Descending Fiber tracts
the representation in Motor homunculus
 Will cause contraction of skeletal muscle, for VOLUNTARY  Feet at the top of the gyrus and the face at the bottom
MOTOR SYSTEM  The facial area  represented bilaterally; the rest  generally
 From cerebrum specifically, the FRONTAL LOBE, with Inputs unilateral
from Parietal lobe  Cortical motor area controls the musculature on the opposite side of
 They will descend utilizing your Spinal cord and Brainstem the body.
o Corticospinal Tract  The cortical representation of each body part is proportional in size
to the skill with which the part is used in fine, voluntary movement.
o Vestibulospinal
 Areas for speech and hand movements are especially large in the
o Reticulospinal cortex
o Rubrospinal
 Lamarck’s theory
 Supplies the cranial motor nucleus o Theory of use and disuse
 Commander o Frequent use  Hypertrophy || Disuse  Atrophy
 40% Arises from Parietal assoc area
 60% arises from Frontal Lobe  PREMOTOR AREA
 Remember: Involuntary Motor System ANS o Brodmann area #6
o Effectors: Smooth and Cardiac muscles + Glands o Loc: Lies anterior to precentral gyrus on lateral and
 Pyramidal and Extrapyramidal system  This terms are not used medial cortical surface of BA#4
anymore o Sets posture of complex movement get
individual prepared to move
CEREBELLUM AND BASAL GANGLIA  How to write, jump, shoot etc
 Command Modifiers o Also has a somatotopic map Greater
 Where Action Potential are noted first representation of Contralateral eye movements,
Head rotation, hands, ear (ear daw ba?)
o Receives input from sensory regions of parietal
Cerebellum Basal Ganglia cortex and projects to primary motor area, spinal
Feedforward Mechanism Feedback Mechanism cord and brainstem reticular formation
 Exact Anticipation  Are you sure?
o Mostly involved in control of proximal limb
 Planning of movement before  Corrects and
they are initiated evaluates muscles needed to orient the body for movement
 Prevents extra movements or movements as they
Contractions happen  SUPPLEMENTARY MOTOR AREA
 to adjust and smooth movement o Counterpart of BA # 6 & BA # 8, extension in
 Where motor memory is medial side
st
“In 2018, MD naako” “After 1 shifting, Gusto ko pa o Loc: on and above the superiorbank of the
ba to? Gusto ko pa ba to? cingulate sulcus on the medial side of the
Huhuhu
hemisphere.
Lesion  Intentional tremors Lesion  Resting tremors
o It projects to the primary motor cortex and also
 Bell Magendie Law
contains a map of the body
o Posterior Horn  Sensory Function
o Antereior Horn Motor Function o For mental rehearsal of complex movement 
 Most Proximal/Lateral portion of anterior horn organizes and plans motor sequences
represents the most proximal/Lateral portion o Less precise than primary motor area Needs
of body respectively greater amount of stimulus to command primary
motor area to do things
5 NEURAL AREAS o Cause complex bilateral contraction
1. Cerebral Cortex 4. Spinal Cord
2. Basal Ganglia 5. Brainstem  Both Primary motor and Supplementary motor area are involved in
3. Cerebellum performing voluntary movements that are complex and involves
planning
 Primary Motor  Executes movement
 Supplementary Motor  Organizes and plans motor sequences
CEREBRAL CORTEX  Premotor  setting of posture for complex movement
 4 Anatomical Lobes
Parietal Lobe Sensory  POSTERIOR PARIETAL AREA
Occipital Lobe Vision o Projects to premotor area
Temporal Lobe Hearing o Lesions of thes omatic sensory area cause defects
Frontal Lobe Motor in motor performance that are characterized by
 Main Concern in Motor Sytem FRONTAL LOBE inability to execute learned sequences of
 Groups of Gyri=4 1 Vertical, 3 Horizontal movements such as eating with a knife and fork.
 Vertical Gyrus aka Pre central Gyrus o Concerned with aiming the hands toward an
object and manipulating it, and hand–eye
Motor Cortex and Voluntary Movement coordination
o Anterior to Central Sulcus  Lesion  inability to execute learned
sequences of movements. Eg, using knife and
o Occupies appx posterior 1/3 of frontal lobe fork while eating
o Contribute to the descending pathways involved in
 PRIMARY MOTOR AREA motor control.
o Brodmann area #4

Page 78 of 89 By: Ayesha Bea Federizo Batch 2017


GRAND PHYSIOLOGY B FINALS REVIEWER

Other areas: Pars Compacta Pars Reticulata


 Frontal lobeInferior frontal gyrusBROCA’S AREA Location Superior Inferior
o BA#45 Pars Triangularis NTA Domapine GABA
o BA#44 Pars Opercularis (O-four-cularis) Function Motor Vision
o Motor speech area
o also well represented since you speak everyday, Ventro Anterior(VA) and Ventro Lateral(VL) Nucleus
well unless you’re mute or whatever  Functionally related to basal ganglia via Ventral group of
 FRONTAL EYEFIELD AREA lateral area of thalamus
o Brodmann Area #8  Portion of thalamus used by the descending (motor) tracts
o Loc: Ant to BA #6  VA= BA # 6; Substantia Nigra, Globus Pallidus and Reticular
o Eyeball coordination, Vertical or horizontal, Formation
depending on vestibular nuclei (medulla  VL= BA # 4 ;Substantia Nigra, Globus Pallidus and
oblongata) involved Cerebellum
o Lesion @ right side Nystagmus towards left side Structure NTA
STRIATUM (Caudate Nucleus and GABA(inhibitory)
 Primary somatosensory cortex in the postcentral gyrus of Putamen)
the parietal lobeSensory Homunculus GPi&GPe GABA (inhibitory)
Sensory Homunculus VA & VL Nucleus Glutamate (stimulatory)
 Parts of the body are represented in order along the postcentralgyrus Cortex Glutamate(stimulatory)
 Legs on top and the head at the foot of the gyrus
Thalamus Glutamate(stimulatory)
 The size of the cortical receiving area for impulses from a particular
part of the body is proportionate to the use of the part Subthalamic Nucleus Glutamate(stimulatory)
 (+) Specificity Face particularly the Lips; Hands particularly the SubstantiaNigra: Pars Compacta Dopamine (stimulatory &Inhibitory)
thumb SubstantiaNigra: Pars Reticulata GABA (inhibitory)
Law of plasticity :Greater representation in cerebrum  Greater sensitivity *For Vision*

 Ideas Cortex
 Planning  Basal Gangli and Lateral Cerebellum
 Execution  Intermediate cerebellum /Spinocerebellum *Principal Connections of Basal Ganglia*
BASAL GANGLIA Dashed lines = Inhibitory || Solid lines = Excitatory

 A misnomer, should be BASAL NUCLEUS since it is found in DIRECT PATHWAY INDIRECT PATHWAY
Cerebrum (CNS) Function For facilitation of Inhibition of movement  Slow
 Involved in the planning and programming of movement; in movement  Fast motor motor activity
the processes by which an abstract thought is converted into activity Reduce Motor Activity
voluntary action Enhance Motor Activity
 Feedback Mechanim Corrects and evaluates movements as Lesion Indirect pathway Direct pathway predominates 
they happen leads to predominates Hyperkinesia
 Influence the motor cortex via the thalamus (corticospinal Hypokinesia
pathways provide the final common pathway to motor Pathway Cerebral Cortex Cerebral Cortex releases
flow releases Glutamate  Glutamate  Striatum is
neurons)
Command Striatum is stimulated stimulated  Striatum releases
: I will
Contract
 Striatum releases GABA GPe is inhibited GPe
my Biceps GABA GPi is cannot release GABA (2 things
inhibited GPi happen)
cannot release GABA 1. GPe Will not Inhibit
 No one will inhibit GPiGPi can release GABA
Thalamus Thalamus  Thalamus is Inhibited 
Caudate Nucleus releases Glutamate to Stop contraction
 Have interconnections with frontal portion of neocortex  cerebral cortex  2. GPe Will not Inhibit
Plays a role in some cognitive process Biceps Contract Subthalamic Nucleus
Putamen Subthalamic Nucleus
 Part of Lentiform Nucleus releases Glutamate GPi is
Globus Pallidus stimulated GPi releases
 Part of Lentiform Nucleus GABA Thalamus is
 2 Portion Inhibited  Stop
o Medial Globus Pallidus Interna (GPi) contraction
 Projects to nuclei in brainstem to motor Dopamine STIMULATORY INHIBITORY
neurons and Drain in Spinal cord
o Lateral Globus Pallidus Externa (GPe)
 Both portion contains inhibitory GABAergic neurons  they
release GABA

Subthalamic Nucleus (Diencephalon)


 Functionally related to basal ganglia

Substantia Nigra (Midbrain)


 Functionally related to basal ganglia
 2 divisions
 Balance is the key for optimum functioning

Page 79 of 89 By: Ayesha Bea Federizo Batch 2017


GRAND PHYSIOLOGY B FINALS REVIEWER

PARKINSON’S DISEASE  Where cerebrocerebellum projects


 Aka Paralysis Agitans  Emboliform Nucleus & Globose Nucleus
 Damage in Midbrain (BEST ANSWER) / SubstantiaNigra /  Adjacent hemispheric portion of
Nigrostriatal tract / Striatum spinocerebellum
 Most severely affected = Putamen *Ganong  Fastigeal Nucleus
 Result from the degeneration of dopaminergic neurons in  Most medial
the substantianigra pars compacta No Dopamine  Medial portion of spinocerebellum

 Direct Pathway Affectation: DIVISIONS


No DopamineStriatum will not be able ANATOMICAL DIVISION
to release GABA  No Inhibition of Anterior Spinocerebellum/ Paleocerebellum / Intermediate
GPiGPi releases GABA  Thalamus is cerebellum
inhibited  No Glutamate release Posterior Neocerebellum / Corticocerebellum / Cerebrocerebellum
Hypokinetic symptoms Flocculonodular Archicerebellum/ Vestibulocerebellum

 Indirect Pathway Affectation: Vermis


No Dopamine Striatum will not release  connects 2 cerebellar hemisphere
GABA to GPe No Inhibition of GPe  Control of axial and proximal limb muscles
 Regulates antigravity muscles
A. GPe releases GABA  o Muscles of the neck, back, trunk and proximal
Inhibits GPiGPi will not release GABA to muscles of extremity
ThalamusThalamus releases Glutamate
Hyperkinetic Symptoms Paravermal portion/ Intermediate cerebellum
B. GPe releases GABA   Regulates distal portion of extremities
Inhibitory to Subthalamic Nucleus GPi
will not be stimulated, will not release FUNCTIONAL/PHYSIOLOGICAL DIVISION (via longitudinal)
GABA  Thalamus releases Glutamate
Hyperkinetic Symptoms Neocerebellum/ Corticocerebellum / Cerebrocerebellum
 Newest
 No Dopamine  No Stimulatory for Direct Pathway + No  Lateral hemisphere, with Anterior and Posterior portion
Inhibitory to Indirect Pathway  INDIRECT PATHWAY  For motor planning (along with premotor area) learning,
PREDOMINATES somehow motor coordination
 (+)hypokinetic and hyperkinetic features  Works with premotor and supplementary motor area (mental
Hypokinetic Hyperkinetic rehearsal)
Akinesia (difficulty initiating Cogwheel Rigidity
movt) Paleocerebellum / Spinocerebellum
Bradykinesia Resting tremors (pill rolling  Also with anterior and posterior portion
tremors)  For distal portion of extremities
 Smooths and coordinates movement while being executed
prevents extra movements
HUNTINGTON’S CHOREA  Formed by of the most vermis and adjacent medial portions
 Initially, damage in Striatum of hemisphere
 Facilitated activity of STN: Striatum is Degenerated  No  Receives proprioceptive input
GABA from striatum GPe will not be inhibited   Receives a “copy” of motor plan from motor cortex
continuous release of GABA from GPe to GPiGPi is further  Control of distal portion of the limb especially hands and
inhibited GPi will not release GABA to Thalamus fingers (lateral brainstem pathway)
Thalamus can release Glutamate  Hyperkinesia
 Problem in subthalamic nucleus Inc inhibition to Archicerebellum/ Vestibulocerebellum
Subthalamic Nucleus(STN) STN will not release Glutamate  Oldest part
GPi will not be stimulated, no release of GABA from  Flocculonodular lobe (same with anatomical)
GPiGPi will not be able to Inhibit the Thalamus Thalamus  Concerned with Gait or Equillibrium in general Vestibular
releases Glutamate Hyperkinesia Input
Refer to Principal Connections of Basal Ganglia Diagram
 Learning induce changesin vestibulo ocular reflex
 S/Sx: Choreiform movement/ dance like movement, Slurred
 Eye movement regulation (along with Frontal eyefield Area)
and incomprehensible speech, incapacitation, dementia,
Athetosis, Ballismus
 Guys, inemphasize ni doc dito na ang problem talaga e dahil kay Lesion/Damage in cerebellum:
Subthalamic nucleus  Incoordination
 Ataxia
 Dysmetria
CEREBELLUM  Dysdiadochokinesia
 Feedforward mechanism, Motor learning, store knowledge  Intention tremors- leads to Decomposition of movement
about motor activity (motor memory)  Malequilibrium
 Inputs from Spinal cord, vestibular nuclei, Pons  Asthenia – loss of muscle strength
 Output is from deep cerebellar nuclei (specially Dentate  Astasia- loss of muscle endurance
Nucleus)  To ventrolateral Nucleus of Thalamus  Hypotonia, Atonia
 Regulate rate, range and force of contraction of muscle and  Dyskinesia
even the direction of the contraction to prevent any
o Intention tremors, Dysdiadochokinesia, dysmetria BRAINSTEM
 Deep Cerebellar Nuclei Midbrain, Pons and Medulla
o provide the only output for the spinocerebellum  Control of axial and proximal musculature, postural muscles
and the cerebrocerebellum  Regulates activity of most of your antigravity muscles
o Counterpart of basal ganglia in cerebrum muscles of proximal extremities, back, trunk and neck
 Dentate Nucleus  Pathways:
 Most lateral, Biggest, and most active

Page 80 of 89 By: Ayesha Bea Federizo Batch 2017


GRAND PHYSIOLOGY B FINALS REVIEWER

Medial Descending Pathways LESIONS


o Reticulospinal  Remember:
 reticular formation to spinal cord o Facilitation of Lateral Corticospinal and
 facilitates extensors Rubrospinal tract  flexed extremities
 Medial Reticulospinal tract – for ANS o Reticulospinal, Tectospinal and Vestibulospinal
regulation Extended Extremities
o Vestibulospinal
 vestibular nuclei to spinal cord  STROKE  Bilateral Lesion in cerebrum (+) damage in
 facilitates extensors lateral and medial corticospinal tract, Rubrospinal tract is
 lesion loss of balance intact  Flexed upper extremities, Extended Lower
o Tectospinal extremities = DECORTICATE
 tectum to spinal cord
 facilitates extensors
 STROKEBilateral Lesion in Midbrain (+) damage
Lateral Descending Pathways
corticospinal tract, rubrospinal tract yung mga asa Medulla
o Rubrospinal
nalangang intact (sabe ni doc yan, pero sa samplex, sa pons
 (midbrain)Red nucleus to spinal cord
daw)  extended upper and lower extremities
 Facilitates flexors of upper extremities
=DECEREBRATE
SPINAL CORD  Para mag decerebrate or decorticate dapat bilateral ang
 Gray matter and white matter lesion
 Anterior Motor || Posterior  Sensory
 Damage in Medulla rarely lives because this is the location
Anterior horn cells of respiratory and cardiac centers
 where the spinal cord fibers synapses
 Motor neurons can be:  Lesion in Tectospinal and Rubrospinal difficulty in leg
o alpha motor neurons – innervate entire turning, unresponsive pupillary light reflex, will still contrict
extrafussalfibers when asked to read, so positive saaccommodation
o gamma motor neurons – innervates  Damage in level of pons Ipsilateral affectation
intrafussalfibers  Damage in midbrain  Contralateral affectation
o Beta motor neurons – interneurons or association
neurons; most abundant, inhibitory or excitatory
 Arrangement:
PATHOLOGIC REFLEXES
o Flexor muscles, Proximal muscles medial side
o Extensor muscles, distal muscles Lateral side Manifests due to damage in descending tracts  UMNL
 Chaddock’s reflex
o From sheen going downward
CORTICODESCENDING PATHWAYS o Stimulation over lateral malleolus to external edge
 Corticobulbar and Corticospinal both originates from cortex
of foot
o Corticobulbar – descend into brainstem bilaterally
o Corticospinal – most important output pathway from the
o Abnormal  Big toe points upward, extension of
cerebral cortex great toe/dorsiflexion of great toe
 60% from frontal lobe  Babinski reflex
o 30% Primary motor area o Normal in infants
o 30% Premotor area o Strike from lateral side going to medial side of the
 40% from Posterior Parietal lobe sole of the foot, draw an imaginary inverted J
2 Pyramidal tract o Test  plantar flexion = Normal
1. Corticobulbar o Abnormal  dorsiflexion of the big toe on
2. Corticospinal
stimulation of the sole, fanning of smaller toes
A. Lateral Corticospinal
 descends lateral side of spinal cord, decusates at  Oppenheims
level of lower medulla, o Strike from Lateral side, towards big toe *sabi ni
 Considered Pyramidal (80-90%) doc
 Descends with rubrospinal tract o Sabi sa Medical free dictionary (+) if dorsiflexion
 regulates flexors of upper and lower extremities, of the big toe upon stroking downward along the
fine movements, distal limbs medial side of tibia
B. Medial/Anterior/Ventral Corticospinal tract
 Does not decussate at the level of medulla
o Test  plantar flexion = Normal
oblongata, it decussates in spinal cord
 Considered as Extrapyramidal (10-20%)
 Descends with tectospinal, vestibulospinal and
reticulospinal
 Postural changes, adjustments
 For control of axial and proximal musculature,
antigravity musles
 Most of projections come from premotor cortex

 Corticobulbar and Corticospinal


 Primary pathway for the initiation of skilled voluntary
movement. But it does not mean that movement, even skilled
movement is impossible without it, this applies for dogs and
cats, for us humans, we need this of course.

 Lateral Descending Pathway  Lateral Corticospinal and


Rubrospinal tract  For Flexion of upper and lower extremities
 rubrospinal Upper extremity flexor only
 Lateral corticospinal  Upper and lower extremity flexor
 Medial descending Pathway  Reticulospinal (lateral part lang
yung Extensor), Vestibulospinal and Tectospinal  For Extension of
upper and lower extremities

Page 81 of 89 By: Ayesha Bea Federizo Batch 2017


GRAND PHYSIOLOGY B FINALS REVIEWER

P.S Nag Recordings at Lecture Guide sya, dagdagan ko lang ng Ganong notes,
plust tatanggalin ko yung ibang takaw sa space na picture maski ayaw nya.
Haha 

LATERAL AND MEDIAL CORTICOSPINAL TRACT

 Lateral Corticospinal Tract


 “PYRAMIDAL TRACT” The reason why it is called your Pyramidal
Tract is because it decussates at the Pyramidal Decussation in your
Lower Medulla.
 80-90% of Corticospinal Go down to the lateral side

 Medial/Anterior/Ventral Corticospinal Tract


 Included in the “EXTRAPYRAMIDAL TRACT” It does not decussate on
the pyramids. It goes straight down and decussates at the level of the
spinal cord.
 10-20%  Go down to the medial side  Terminate in the anterior
 Decussate in the Anterior White Commissure  Stimulate
Anterior Horn Cells on the CONTRALATERAL SIDE

 Whether it is lateral or medial/anterior corticospinal, they will


stimulate the anterior horn cells on the contralateral side. The
only difference is where they decussate.

 Lateral Spinothalamic Tract Decussates at the level of the spinal cord


 Decussate at the Anterior White Commissure

Page 82 of 89 By: Ayesha Bea Federizo Batch 2017


GRAND PHYSIOLOGY B FINALS REVIEWER

MUSCLE SPINDLE AND GOLGI TENDON


 Skeletal muscle  Inside it you will find MUSCLE SPINDLE
 Tendon  You will find GOLGI TENDON

 When you strike the tendon, you stretched the muscle Muscle
Spindle is stretched
 When muscles contract, you stretch Golgi Tendon
 Both are stretch receptors, but it depends how you stretch it.

MUSCLE SPINDLE GOLGI TENDON


Sensory Neuron/
IA and II IB
Sensory Innervation
 The side that experienced weakness does not have full supply that is why Gamma Motor
Motor Innervation -
it only experienced weakness. Neurons
 For example we are at the level of T1  At this level, you have Stretch Contraction
hemisection Anterior, Lateral and Posterior Funniculus is destroyed  Stimulus (Stimulated when the (Stimulated when the
Anterior Horn Cell is destroyed So it will manifest as LOWER MOTOR muscle is stretched) muscle contracted)
NEURON LESION because you destroyed the Anterior Horn Cell itself Contraction/Tension
Stretching of the built by the muscle
 What about Below the level of T1? anterior horn cells are not Monitors
muscle during muscle
destroyed  There would still be supply to their descending tract (10%) contraction
 UPPER MOTOR NEURON LESION
 If it is a sensory receptor, it is innervated by sensory neurons. Muscle
Spindles are an exemption. They are also innervated by motor
neurons. It is also innervated by Gamma Motor Neurons.

 What is the reaction of your muscle when you stimulated it? CONTRACT
 What happened when it contracted? STRETCH/RELAX
 How I understand it : When you stretch your muscle  Muscle spindle
works  after some time  It will contract  After contractions  Golgi
tendon works  muscle relaxes

 MUSCLE SPINDLE
 Stretch sensitive receptor
 Feedback circuit for muscle length
 Consists of thin intrafusal muscle
fibers attached to the associated
muscle fibers which surrounds the
muscle spindle.

 Inside the muscle spindle


Intrafusal Fiber
 Whole muscle Extrafusal
Fiber

 Intrafusal Muscle Fibers are devoid


of actin-myosin contractile
elements. They form a capsule
containing several nuclei.
─ When nuclei are arranged in a
linear fashion  NUCLEAR CHAIN
─ When nuclei are clustered or
KNEE JERK REFLEX (PATELLAR-TENDON REFLEX)
clumped in the central region 
 When you elicit a knee jerk reflex, what stretch receptors are you
 NUCLEAR BAG
stimulating? MUSCLE SPINDLE
 How do you elicit patellar tendon reflex?
 As mentioned earlier, muscle spindle is quite unique It has
1. Ask the patient to sit on a high chair  Make sure that the legs are
both sensory and motor innervation  Motor Innervation:
hanging
Gamma Motor Neuron  Gamma Motor Neuron  Supply
 If there is no high chair, ask the patient to cross his/her legs
Intrafusal Fibers  Both Nuclear Chain and Bag  Why? In its
2. Using the reflex hammer, tap the patella
end, it has actin and myosin filaments  That is why it is
 Not totally at the patella  Tendon will only pass through it (Tendon
stretched and return to its normal
is near the patella)
 Golgi Tendon  When it is stretched, it does not return  No
 When you strike the reflex hammer  You stimulate the MUSCLE
actin and myosin filaments, what they have is connective tissue
SPINDLE first
 That is why muscle spindle has an efferent innervation and golgi
tendon doesn’t.

Page 83 of 89 By: Ayesha Bea Federizo Batch 2017


GRAND PHYSIOLOGY B FINALS REVIEWER

Structures of muscle spindle and group II neurons neurons


Remember: Innervated by static gamma Innervated by Dynamic gamma
 2 types of intrafusal fibers: motor neuron motor neuron
o Nuclear bag fibers, also with 2 subtypes --- Oppose sudden changes in
 Static nuclear bag muscle length
 Dynamic nuclear bag steady-state length of the speed of movements
o Nuclear chain fibers
muscle and allow for quick corrective
 2 kinds of sensory endings
movements.
o Primary (group Ia)
 Wraps around the center of the dynamic
DYNAMIC RESPONSE
and static nuclear bag fibers and nuclear
 Primary Endings Responsible for DYNAMIC RESPONSE
chain fibers. In short, sa lahat mag wrap sya
o Rate of the change or velocity of the change in the
o Secondary (group II)
length
 Located adjacent to the centers of the static
 Dynamic Reflexes = Rate in the change of muscle length
nuclear bag and nuclear chain fibers; these
fibers do not innervate the dynamic nuclear
STATIC RESPONSE
bag fibers.
 Secondary Endings and also Primary Endings Responsible for
 Dynamic nuclear bag is only innervated by Group 1a or primary
STATIC RESPONSE
sensory endings
o Maintain length of muscle fiber
 Nuclear chain is supplied by both Ia and II
 For maintenance of the length of the muscle which is contracting
2 types of gamma motor neuron
 Which is the one that gives the signal that the muscle is
shortening and lengthening / Isotonic contractions  Primary
Dynamic gamma motor neuron Static gamma motor neurons
Endings (Ia)
 Supply dynamic nuclear  Supply static nuclear bag
 When you maintain a certain length / Isometric contractions ( you
bag fibers fibers and the nuclear chain
don’t change muscle length)  Secondary Endings (II)
 Activation of this fibers
increases the dynamic  Activation of this increases the
sensitivity of the group tonic level of activity in both
Ia endings group Ia and II endings
 decreases the dynamic
sensitivity of group Ia
afferents, and can prevent
silencing of Ia afferents during
muscle stretch

 Two types of sensory fibers are associated with muscle spindle


intrafusal muscle fibers:
PRIMARY ENDINGS (Ia) SECONDARY ENDINGS (II)
Annulo Spiral Flower Spray
Type Ia myelinated sensory fiber Type II myelinated sensory fiber
Innervates both the Dynamic & Innervates only the nuclear
Static nuclear bag and nuclear chain fibers and the Static
chain nuclear bag
Detects amount of muscle Not sensitive to rate of change
stretch but more sensitive to of muscle length but provides
the rate of change of muscle information about the static
length length of the muscle
Velocity sensitive fibers --

 STRETCH REFLEX
─ Type Ia sensory fibers conduct impulses to the spinal cord entering
the dorsal root and synapse directly (monosynaptic) with alpha
motor neurons in the ventral horn that conduct impulses to the
extrafusal muscle fibers in the same muscle where the type Ia fibers
originated.
─ Also known as MYOTATIC REFLEX which results to contraction of the
 Size Principle  The firs one to be recruited is your slow twitch
stretched muscle
type I muscle since the size is small
─ Stretch Reflex has two components: DYNAMIC PHASE AND STATIC
 Slow twitch muscle – for Marathoners
PHASE
 Fast twitch muscle  Sprinters
STATIC PHASE (RESPONSE) DYNAMIC PHASE (RESPONSE)
 GOLGI TENDON
Weak, slow, continuous Strong, sudden stretch of the  Tension sensitive encapsulated receptors which consist of a net like
stretch of the muscle spindle muscle spindle  For carrying collection of knobby nerve endings among the fascicles of a tendon
 For posture and balance load, when doing work arranged in series with the extrafusal muscle fibers
Involves almost equal activity Involves activity mostly of the  Intermingle with the tendon fibers
of the static nuclear bag and dynamic nuclear bag; same  Stimulated when the tension imposed by muscle contraction is
nuclear chain activity of the nuclear chain increased
Involves activation of group Ia Greater activity of the group Ia

Page 84 of 89 By: Ayesha Bea Federizo Batch 2017


GRAND PHYSIOLOGY B FINALS REVIEWER

 Like the muscle spindle, the golgi tendon organ reacts vigorously when
the tendon is undergoing stretch (dynamic response) and then settles
down to a steady state level that is proportional to the degree of
tension (static response)
 Stimulated by pressure stretch and active contraction
 Low threshold
 Functions as transducers in feedback circuit that regulates muscle force
 RENSHAW CELL
TYPES OF MOVEMENT GENERATED BY MOTOR SYSTEM ─ Interneuron  Release inhibitory neurotransmitter  Inhibit
both stimulatory and inhibitory
 VOLUNTARY MOVEMENT
 Voluntary ─ It doesn’t mean that when it is inhibitory, it is really inhibited 
When you inhibit the inhibitor = stimulation
 Movement is purposeful
 Learned ─ this circuitry prevents refl ex stimulation of the extensors when
flexors are active
 REFLEXES
─ Prevents clonus
 Rapid
 Stereotype
 Involuntary  DYNAMIC REFLEX
 Sensory Neuron: Ia Primary Afferent Supplying both static and
 RYTHMIC MOTOR PATTERNS
dynamic Nuclear Bag and Nuclear Chain (Intrafusal Fiber)
 Stereotype
 Repetitive
 Static Reflex  Secondary Afferent  Supply on the Nuclear
 Confer initially as fast repetitive Eventually stops
Chain and static nuclear bag
 Can be seen in pathologic conditions, especially in an upper motor
neuron lesion
 INVERSE MYOTATIC
 Disynaptic
 Upper Motor Neuron Lesion Comatose patient (CVA,
hemorrhage)  Try to dorsiflexthe foot for some time 
Release  Knee-Jerk Reflex  Stretch muscle (MYOTATIC)  Reaction:
 Normally: It would return Contraction  Stimulate INVERSE MYOTATIC
 Reflex-like
 In comatose patient, when you release it It would return, 
but it would have repetitive movements until it stops  Receptor: Golgi Tendon
REFLEX  Sensory Neuron: Ib
 Receptor  Sensory Neuron  Motor Neuron  Effector (Skeletal  Center: Spinal Cord
Muscle)
 RECEPTOR:Muscle Spindle, Golgi Tendon, and Free Nerve Endings (PAIN)
or sometimes touch  We know that there is no motor neuron that innervates golgi
tendon, however, there is a motor neuron in inverse myotatic
reflex. It is a reflex, so it just doesn’t end at the center (it should
have a receptor, sensory neuron, center, motor neuron and an
effector).

 Motor Neuron: Alpha Motor Neuron


 Effector: Skeletal Muscle
 SENSORY / RECEPTOR ORGAN:Muscle Spindle, Golgi Tendon, Free Nerve
Endings  WITHDRAWAL REFLEX
 AFFERENT NEURON: I, II III, IV  Reflex when you are trying to avoid noxious stimulus
 CENTER: Spinal Cord (if it is a spinal cord reflex)  Withdraw  Usually a flexor reflex
 EFFERENT NEURON:Alpha or Gamma Motor Neuron  Polysynaptic  Involves the contralateral and ipsilateral leg
 EFFECTOR: Skeletal Muscle  Receptor: Pain Receptor (Free Nerve Endings)
 Sensory Neuron: Spinal Nerve
 Center: Spinal Cord
TYPES OF REFLEX  Motor Neuron: Alpha Motor Neuron
 MYOTATIC REFLEX Monosynaptic  Effector: Skeletal Muscle
 INVERSE MYOTATICDisynaptic
 WITHDRAWAL REFLEX Polysynaptic

 MYOTATIC REFLEX
 Monosynaptic Example: KNEE JERK REFLEX
 Receptor: Muscle Spindle
 Sensory Neuron: Dorsal Root Ganglion Ia and II
 Center: Spinal Cord
 Motor Neuron: Alpha Motor Neuron
 Effector: Skeletal Muscle
 Muscle spindle is innervated by Gamma motor neurons
 So if ang tanong is direct innervation of muscle spindle Gamma
 RECIPROCAL INHIBITION motor neurons
─ When you stimulate the flexor, you inhibit the extensor  Golgi Tendon has no motor neurons
─ You cannot stimulate both flexor and extensor at the same time  Extrafussal fibers or whole muscle are innervated by Alpha
motor neuron
 If mga reflex, eg: Myotatic, reverse myotatic and withdrawal
reflex  Alpha motor neuron
 Muscle spindle: 1a and II
 Golgi Tendon: 1b

Page 85 of 89 By: Ayesha Bea Federizo Batch 2017


GRAND PHYSIOLOGY B FINALS REVIEWER

THERMOREGULATION TEMPERATURE READINGS


Source: recordings + Limpin notes  Average body temperature
 BODY TEMPERATURE o calculates total amount of heat stored in the body
o difference between heat produced & heat lost by = 1/3 surface T + 2/3 core T
the body
o Normal range = 36.7 – 37.1 C  Core Temperature Vs. Skin/Surface Temperature
o Most common temp taken
 Importance of maintenance of Body Temp:
o Metabolic process Core Temp Skin/Surface Temp
 Inc temp  Inc metabolism Core temp of deep tissues = constant depends on the temperature of the
varying less than 1 F under normal environment  Varies widely
 The body is the one that maintains
condition Almost constant, no wide
metabolic process it cannot be variations Eg: Dec environment temp  body
controlled by temperature of the temp also decreases
environment This is the Internal environment of the
o Enzyme system body : organs with blood flowing Inc environment temp  body temp
 Body depends on the enzymes system thorough it also increases
present in the body
CNS maintained
 Extreme temp  Enzymes are very
fragile and can be broken down easily  Big Variation between Skin temp and Core Temp  In a matter of
 The Body temp is also able tomaintain time Core temperature will compensate
the function of the enzymess system
HEAT PRODUCTION: Factors that Inc Temp
TEMPERATURE VARIATIONS  BMR- MAIN FACTOR
 Various parts of the body or different sites:  Extra metabolic production of heat Muscular exercise
Oral, axillary and Rectal  Specific dynamic action of food
o Oral temperature o with corresponding caloric equivalent
 Influenced a by breathing, drinks o Inc caloric equivalent in fat then protein and
 always from the internal environment of lastly Carbohydrates
the body  Cooler  Sympathetic stimulation
 Considered as superficial o Epinephrine and Norepinephrine elaboration 
o Axillary Temp Inc heat production
 Considered as superficial  Hormonal stimulation
o Rectal Temperature o endocrine cause like thyroxine etc  Inc heat
 Highest, represents core temperature production
 Not very accurate reading because of  Infections, Drugs & anesthetics
bacteria  unusually high temp
o Tympanic membrane HEAT TRANSFER
 Facilitated by physical loss of heat
Temperature from oral to axillary – difference of 0.5 centigrade 1. Radiation (60%)
Temperature from axillary to rectal difference of 0.5 centigrade o Heat loss thru heat waves, no direct contact of
Temperature from oral to rectal – a difference of 1 centigrade body & the wall

 Temperature from outside towards inside of the body 2. Conduction (air 15%,objects 3%)
increases o With direct contact between 2 surfaces
 There is consistency in these Variations of temperature o Eg: when you seat down on a cold chair direct
movement of heat towards the chair since the
 Time of Day body has higher temperature than the chair
o circadian rhythm= 0.5-0.7 C fluctuations
o Early Morning 3. Convection
 Still in bed, not doing anything  Basal o utilizes air movement
Temperature o Eg: When a person is on cool environment  air
 Lowest temperature movement from the body towards the
o Early evening (6-7pm) environment
 done all physical activities throughout
the day  Inc temp and BMR 4. Insensible water loss and Sweating or Evaporation (27%)
 Highest temp o even if seating on the chair only there is
perspiration
 Hormonal influence
o Ovulation period 5. Respiration (2%)
 Morning: Lowest basal Temp  then o observed in animals
temperature spurts o not very useful in human unless aso ka 
 Inc temp= increase possibility to get
pregnant (female is probably ovulating) 6. Via Urine and Feces (1%)
 So Temperature here can serve as a Rhythm  for family
planning  Transfer of heat is from a higher temperature to lower
temperature, it depends on which has a higher gradient or
 Age in Children whic is warmer
o Children usually have higher BMR (Basal o If Environment Temp higher than body  Transfer
Metabolic Rate)  Higher temperature of heat is from environment to the body
o If the environment temp is lower than the body 
 Constitutional Hyperthermia
transfer of heat is from the body to the
o Seen in some normal adults
o Normally high body temperature even without a environment
cause

Page 86 of 89 By: Ayesha Bea Federizo Batch 2017


GRAND PHYSIOLOGY B FINALS REVIEWER

ADAPTABILITY TO TEMP CHANGES o came from the inner surface of the body warm
Poikilothermic Vs. Homeothermic
Poikilothermic Homeothermic  As blood goes along artery has close contact with venous
Eg. Cold blooded animals Eg. Human beings, except plexuses : if the temp is cold
newborn/infants o Venous plexuses will prevent the warm blood
Body temp depends on the temp of
the environment going to the artery to go to the skin
No means of maintaining body
With means of maintaining body o There is transfer of heat from the artery to venous
temperature and no compensation temperature
plexus & carried back to the interior part of the
body
 Newborns or infants are considered as poikilothermic
o Coping up mechanism is immature o There is a conservation of heat from a mixture of
o no adequate means of maintaining body direction from the artery to the vein
temperature + body temperature is maintained
rudimentarily  When BV constrict it prevents the blood going to the skin &
o So when exposed to cold temperature lead to the warm blood goes towards the artery so less blood is
hyperthermia and eventually dies
going to the skin
Adaptive Mechanism
 Specific physiological mechanism 2 Mechanisms:
 Employed by the body towards the vessels A. Counterheat Mechanism
o Cold place  VASOCONTRICTION  heat is o direct transfer of heat from the artery to venous
conserved plexus, to conserve heat
o Hot place VASODILATION  Increases heat loss B. Constriction of Blood Vessel
 Cold temp (+) Shivering Inc contraction of muscle o prevents the arterial blood to go to the skin
(uncoordinated)  Increases BMR  Inc Body heat
o Conserves a lot of heat
 Brown fat  For Non-shivering thermogenesis (NST)
o found in body especially used for newborn
 Sweating  When Temp is warm
o Compensation for hyperthermia o (+) Vasodilation  more warm blood goes to
skin heat loss
Types of Fat
TEMPERATURE REGULATING MECHANISM PART 1
Structural Neutral
 Cold related mechanisms to increase heat production:
Found in different parts of the body Found in waistline = Bilbil
 Shivering
Eg: cells, cell memembrane nerves, all Contains many mitochondria, o Increases BMR 300-400x
that has lipid component sympathetic innervations & Blood o Problem: Inc BMR There is Inc in
vessels oxygen demand, so people with chronic
Brown Fat White Fat heart disease can’t tolerate the decrease
Specialized fat for baby Appears during adult life in oxygen  may lead to Ischemia
o Only seen in adults,
Very rich tissue with a lot of Also contains Lipid droplets  Hunger
mitochondria, nuclei & lipid droplets
with a lot of sympathetic innervations, Also have sympathetic stimulation
o food caloric equivalent is a natural
blood vessels all around response mechanism
Found in inner structure like thorax, o Inc caloric equivalent  Inc heat
Means of the body to maintain heat abdomen & shoulder production
especially in newborn(baby) o Mas malamig mas masarap kumain
 Voluntary activity
no sufficient heat  limited only  In adults, brown fats  Inc catecholamine secretion (Epi & NEpi)
disappears o Remember: Cold is a form of physical
upon sympathetic stimulation (cold) 
catecholamine released most stress Inc release of catecholamine 
specifically norepinephrine  Inc sympathetic stimulation Inc BMR
breakdown of energy substrates  inc  Inc heat production
heat production
ENERGY EXPENDITURE CHANGES WITH ENVIRONMENTAL TEMP
SKIN RADIATOR SYSTEM 1. Zone of thermoneutrality
 Skin together with subcutaneous fat (especially the fats)
o temperature of the body where energy
o Very important for conservation of heat
expenditure is almost zero
o Fat is a very good insulator
o Important for children
 they are not a good medium for heat
transfer Eg: Baby in a very cold operating room + cold IV fluids
 Baby will try to compensate to Inc heat production by any means
 8 fold difference in heat conduction between maximum o This adaptive mechanism can lead to failure on the
dilation & maximum constriction system of the baby because the baby cannot response
o Inc temp  vasodilatation  high heat adequately to that condition
conduction o Baby needs a higher the temperature, so we try to find
temperature where energy expenditure is almost zero.
o Dec temp  vasoconstriction low heat
conduction
2. Sweating in Hot; Shivering in Cold
o Uses energy
ROLE OF SKIN BLOOD FLOW IN TEMPERATURE REGULATION
 Blood going to the arteries
TEMPERATURE REGULATING MECHANISM PART 2

Page 87 of 89 By: Ayesha Bea Federizo Batch 2017


GRAND PHYSIOLOGY B FINALS REVIEWER

 Cold related mechanisms to decrease heat loss:  Efficient receptors/ effector organ
 Cutaneous vasoconstriction o Cold environment detected by cold
o less blood is exposed to the skin receptors  send impulse brain  adapt to
low temperature regulate organs 
 Curling up
adaptive mechanisms takes place  correct
o maintain fetal position  dec body surface
area  dec heat loss the low temperature  Inc temp

 Horripilation (goose bumps/goose flesh) TYPES OF BODY RESPONSE


o Normally: the hair in skin is slant position 1. Behavioral response
o When exposed to cold arrector pili contract o occurs mostly as a response to skin temperature
 hair moves up (upright) skin thickens  changes
dec heat loss
Eg: cold  wear a jacket or rub hands together
COUNTER CURRENT HEAT EXCHANGE
 Direct transfer of warm blood 1. Autonomic response
o Heat Transfer=artery to vein without blood going o occurs as a response to core temperature changes
to skin o Responsible in 80% of cases
 Direct constriction o Includes vasoconstriction, vasodilatation,
o preserves heat of the body sympathetic & thyroid stimulation as well as
increasing catecholamine, sweating, shivering

Mechanism Regulated by Heat Mechanism Activated by heat  Interthreshold change


To Inc Heat Loss To dec Heat production o Narrow range of core temperature maintained by
Cutaneous Vasodilation Anorexia hypothalamus
Sweating Apathy o not triggered by autonomic response
Increase Respiration (common in Inertia
animals) PHASES OF TEMP INFORMATION PROCESSING
1. Cold receptor – C & A delta fibers;
Heat receptor – C fiber
 Heat loss from body when environmental Temp is greater
2. Afferent thermal sensing
than body temperature
o Heat Transfer is from high temp to low temp 3. Via Spinothalamic tract
o You tend to sweat to lower the body temp 4. Center: Preoptic area/ anterior hypothalamus
 Heat center
SWEATING Post hypothalamus
 A form of evaporative heat loss  Received impulses from anterio
 (+) Stimulation of PREOPTIC AREA of hypothalamus via hypothalamus and periphery
sympathetic preganglionic cholinergic fibers  sweating is  Cold center
stimulated  Heat producing and heat conserving
 Circulating NEpi & Epi
 2 components of sweat gland  Extremes of temperature will lead to pain sensation
1. Deep subdermal coiled portion  There is more cold receptors than heat receptors  more
o secretes sweat dangerous kasi ang hypothermia
2. Duct portion  Most of the deep body receptors are cold receptors
o passes outward thru the dermis & epidermis of
skin BODY TEMP DETECTION
 Primary Secretion is similar to plasma but modified (no  HYPOTHALAMUS TEMPERATURE CONTROL CENTER
proteins) as it passes up towards the skin where there is o Via Preoptic area of anterior hypothalamus
absorption of NaCl o Heat sensitive & cold sensitive neurons
 Light sweating  Dec Na secretion, narereabsorb kasi
 Increased or excessive sweating  Inc Na Secretion  SKIN & DEEP BODY TEMPERATURE
o Mainly detects cold & cool temperatures
 As sweat comes out: Na along with water is reabsorbed  o Function to prevent hypothermia
dec secretion of Na along with water in sweat
 During strenuous exercise sweat comes out fast so  POSTERIOR HYPOTHALAMUS
reabsorption does not take place (mabilis kasi) Inc NaCl as o Receives input from anterior hypothalamus &
well as water secretion peripheral temperature receptors to elicit mainly
 In case of excessive sweating there is a possibility of having heat producing & heat conserving reactions
dehydration
PATHOLOGIC CONDITIONS
TEMPERATURE CENTERS  Fever
 Posterior Hypothalamus o Oldest, universally known Hallmark of disease
 Preoptic area of hypothalamus ( Anterior Hypothalamus) o Temp above 100° F/ 38° C
o Thermostat is set at higher level activating
Temp Regulation thru feedback mechanism temperature raising mechanism
 Always via a negative feedback to counteract temperature o (+) Chills & shivers
 Working center/area i.e brain  FEVER – Resetting Set Point Temperature

Page 88 of 89 By: Ayesha Bea Federizo Batch 2017


GRAND PHYSIOLOGY B FINALS REVIEWER

Pyrogens Can Directly Reset Point Pyrogens Can Indirectly Reset Point HYPOTHERMIA
Temp Temp  Heat regulation is lost at 85°F/29°C
Bacteria  Liposaccharides are Interleukin 1 released from phagocytes  In a man exposed to ice water for 20-30 minutes Cause of
pyrogens following phagocytosis of blood borne death is cardiac standstill
Pyrogens from degenerating tissue pyrogens  Causes Frost bites,
o Due to formation of microcrystal in cells
Interleukin 1 reset set point of temp by o usually affects ear lobes
increasing production of Prostaglandin
E2 (PG E2)
o Gangrene may follow after thawing

Phatogenesis of Fever
 Tissue becomes infected with bacteria, viruses, etc
WBC/leukocyte responds to infection & releases a protein
called IL-1 or Interleukin 1  IL-1 stimulates the production
of PROSTAGLANDIN E1 and PROSTAGLANDIN E2
Prostaglandins cross the blood brain barriers and signals the
Preoptic Area or Ant Hypothalamus  Inc Set point  brain
perceives that body is cold  (+) chills and shivering Fever

Time Course of Fever

 If baby has cold


feet  (+) vasoconstriction as
a means of compensation 
prone to fever

 Aspirin prevents or
inhibits prostaglanding
production

Benefits of Fever
 Acts as warning signal for the presence of disease processes
 High Temp Inc in Antibody production
 Slows growth of some tumor

Limits of Survival
 Extreme Temp: Very High/Low FATAL
 Hyperthermia  may lead to heat sroke

HEAT STROKE
 Above critical temperature of 106-108° F / 41-42 C
 S/Sx:
o dizziness, abnormal distress, delirium & coma
 Death is usually due to circulatory collapse

 Remember: the Limit of temperature one can withstand


depends on the humidity of air
 Water – good medium for heat, it absorbs heat from
environment
HUMIDITY – amount of water in air

MALIGNANT HYPERTHERMIA
 Usually happens to young and newborn, but also happens to
adults
o Associated with congenital anomalies
 A rare case, Fatal if not recognized early
 Triggered by traumatic events, most commonly triggered by
drugs
 Familial
o include 2 or more members of the family
 (+) Disorder in Ca metabolism
o Ca from Sarcoplasmic Reticulum  release Ca 
actin/myosin  causes contraction Ca does not
go back to Sarcoplasmic Reticulum persistent
muscle contraction Inc muscle metabolism  Inc
heat production
 Problem in ryanodine receptors
 S/Sx: Severe fever, tachycardia, arrhythmia, rigidity, Inc muscle
metabolism
 ManagementDantrolene Sodium (muscle relaxant) very
expensive, has short life and short expiration
o Effective when given early

Page 89 of 89 By: Ayesha Bea Federizo Batch 2017

Das könnte Ihnen auch gefallen